MED/SURG FINAL (Lewis Chapters 14-16, 23-24, 41-50, 63-65)

Réussis tes devoirs et examens dès maintenant avec Quizwiz!

Elimination changes that result from inability of the bladder to empty properly may cause which of the following? (Select all that apply.) A) Incontinence B) Frequency C) Urgency D) Urinary retention E) Urinary tract infection

A) Incontinence B) Frequency C) Urgency D) Urinary retention E) Urinary tract infection

The nurse would assess a patient admitted with cellulitis for which of the following localized signs? A) Pain B) Fever C) Chills D) Malaise

A) Pain Pain, redness, heat, and swelling are all localized signs of cellulitis. Fever, chills, and malaise are generalized, systemic manifestations of inflammation and infection.

A 21-year-old female soccer player has injured her anterior crucial ligament (ACL) and is having reconstructive surgery. The nurse knows that the patient will need more teaching when the patient makes which statement? A "I probably won't be able to play soccer for 6 to 8 months." B "They will have me do range of motion with my knee soon after surgery." C "I can't wait to get this done now so I can play soccer for the next tournament." D "I will need to wear an immobilizer and progressively bear weight on my knee."

C "I can't wait to get this done now so I can play soccer for the next tournament." When the athlete has ACL reconstructive surgery, the patient does not understand the severity when planning to be back to playing soccer soon, as safe return will not occur for 6 to 8 months after initial range of motion, immobilization, and progressive weight bearing with physical therapy occurs.

29. A type 1 diabetic patient who was admitted with severe hypoglycemia and treated tells the nurse, "I did not have any of the usual symptoms of hypoglycemia." Which question by the nurse will help identify a possible reason for the patient's hypoglycemic unawareness? a. "Do you use any calcium-channel blocking drugs for blood pressure?" b. "Have you observed any recent skin changes?" c. "Do you notice any bloating feeling after eating?" d. "Have you noticed any painful new ulcerations or sores on your feet?"

C. "Do you notice any bloating feeling after eating?" Rationale: Hypoglycemic unawareness is caused by autonomic neuropathy, which would also cause delayed gastric emptying. Calcium-channel blockers are not associated with hypoglycemic unawareness, although -adrenergic blockers can prevent patients from having symptoms of hypoglycemia. Skin changes can occur with diabetes, but these are not associated with autonomic neuropathy. If the patient can feel painful areas on the feet, neuropathy has not occurred.

The patient is told that the adenoma tumor is not encapsulated but has normally differentiated cells and that surgery will be needed. The patient asks the nurse what this means. What should the nurse tell the patient? A. It will recur. B. It has metastasized. C. It is probably benign. D. It is probably malignant.

C. It is probably benign. Benign tumors usually are encapsulated and have normally differentiated cells. They do not metastasize and rarely recur as malignant tumors do.

Which information noted by the nurse when caring for a patient with a bladder infection is most important to report to the health care provider? a. Dysuria b. Hematuria c. Left-sided flank pain d. Temperature 100.1° F

C. Left-sided flank pain Flank pain indicates that the patient may have developed pyelonephritis as a complication of the bladder infection. The other clinical manifestations are consistent with a lower urinary tract infection (UTI). DIF: Cognitive Level: Application REF: 1132-1133

The nurse is admitting a patient who complains of a new onset of lower back pain. To differentiate between the pain of a lumbar herniated disc and lower back pain from other causes, what would be the best question for the nurse to ask the patient? A. "Is the pain worse in the morning or in the evening?" B. "Is the pain sharp or stabbing or burning or aching?" C. "Does the pain radiate down the buttock or into the leg?" D. "Is the pain totally relieved by analgesics, such as acetaminophen (Tylenol)?"

C. "Does the pain radiate down the buttock or into the leg?" Lower back pain associated with a herniated lumbar disc is accompanied by radiation along the sciatic nerve and can be commonly described as traveling through the buttock, to the posterior thigh, or down the leg. This is because the herniated disc causes compression on spinal nerves as they exit the spinal column. Time of occurrence, type of pain, and pain relief questions do not elicit differentiating data.

The nurse has reviewed proper body mechanics with a patient with a history of low back pain caused by a herniated lumbar disc. Which statement made by the patient indicates a need for further teaching? A. "I should sleep on my side or back with my hips and knees bent." B. "I should exercise at least 15 minutes every morning and evening." C. "I should pick up items by leaning forward without bending my knees." D. "I should try to keep one foot on a stool whenever I have to stand for a period of time."

C. "I should pick up items by leaning forward without bending my knees." The patient should avoid leaning forward without bending the knees. Bending the knees helps to prevent lower back strain and is part of proper body mechanics when lifting. Sleeping on the side or back with hips and knees bent and standing with a foot on a stool will decrease lower back strain. Back strengthening exercises are done twice a day once symptoms subside.

The nurse receives report from the licensed practical nurse about care provided to patients on the orthopedic surgical unit. It is most important for the nurse to follow up on which statement? A. "The patient who had a spinal fusion 12 hours ago has hypoactive bowel sounds and is not passing flatus." B. "The patient who had cervical spine surgery 2 days ago wants to wear her soft cervical collar when out of bed." C. "The patient who had spinal surgery 3 hours ago is complaining of a headache and has clear drainage on the dressing." D. "The patient who had a laminectomy 24 hours ago is using patient-controlled analgesia with morphine for pain management."

C. "The patient who had spinal surgery 3 hours ago is complaining of a headache and has clear drainage on the dressing." After spinal surgery there is potential for cerebrospinal fluid (CSF) leakage. Severe headache or leakage of CSF (clear or slightly yellow) on the dressing should be reported immediately. The drainage is CSF if a dipstick test is positive for glucose. Patients after spinal surgery may experience paralytic ileus and interference with bowel function for several days. Postoperatively most patients require opioids such as morphine IV for 24 to 48 hours. Patient-controlled analgesia is the preferred method for pain management during this time. After cervical spine surgery patients often wear a soft or hard cervical collar to immobilize the neck.

Which cellular dysfunction in the process of cancer development allows defective cell proliferation? A. Proto-oncogenes B. Cell differentiation C. Dynamic equilibrium D. Activation of oncogenes

C. Dynamic equilibrium Dynamic equilibrium is the regulation of proliferation that usually only occurs to equal cell degeneration or death or when the body has a physiologic need for more cells. Cell differentiation is the orderly process that progresses a cell from a state of immaturity to a state of differentiated maturity. Mutations that alter the expression of proto-oncogenes can activate them to function as oncogenes, which are tumor-inducing genes and alter their differentiation.

A 26-year-old patient with a history of polycystic kidney disease is admitted to the surgical unit after having knee surgery. Which of the routine postoperative orders is most important for the nurse to discuss with the health care provider? a. Infuse 5% dextrose in normal saline at 75 mL/hr. b. Order regular diet after patient is awake and alert. c. Give ketorolac (Toradol) 10 mg PO PRN for pain. d. Obtain blood urea nitrogen (BUN), creatinine, and electrolytes in 2 hours.

C. Give ketorolac (Toradol) 10 mg PO PRN for pain The NSAIDs should be avoided in patients with decreased renal function because nephrotoxicity is a potential adverse effect. The other orders do not need any clarification or change. DIF: Cognitive Level: Application REF: 1142-1143

23. A diabetic patient is admitted with ketoacidosis and the health care provider writes all of the following orders. Which order should the nurse implement first? a. Start an infusion of regular insulin at 50 U/hr. b. Give sodium bicarbonate 50 mEq IV push. c. Infuse 1 liter of normal saline per hour. d. Administer regular IV insulin 30 U.

C. Infuse 1 liter of normal saline per hour. Rationale: The most urgent patient problem is the hypovolemia associated with DKA, and the priority is to infuse IV fluids. The other actions can be accomplished after the infusion of normal saline is initiated.

17. When teaching a patient with type 2 diabetes about taking glipizide (Glucotrol), the nurse determines that additional teaching about the medication is needed when the patient says, a. "Since I can take oral drugs rather than insulin, my diabetes is not serious and won't cause many complications." b. "If I overeat at a meal, I will still take just the usual dose of medication." c. "If I become ill, I may have to take insulin to control my blood sugar." d. "I should check with my doctor before taking any other medications because there are many that will affect glucose levels."

A. "Since i can take oral drugs rather than insulin, my diabetes is not serious and won't cause many complications" Rationale: The patient should understand that type 2 diabetes places the patient at risk for many complications and that good glucose control is as important when taking oral agents as when using insulin. The other statements are accurate and indicate good understanding of the use of glipizide.

The nurse performs a physical assessment on a 74-year-old woman with possible endocrine dysfunction. The patient's weight was 142 pounds 6 months ago compared to a current weight of 125 pounds. What percent weight change will the nurse document in the patient's health record? A. 12% weight loss Correct B. 17% weight loss C. 25% weight loss D. 74% weight loss

A. 12% weight loss 142 pounds - 125 pounds = 17 pounds; (17/142) × 100 = 12%. Weight change (%) is calculated by dividing the current body weight change by the usual body weight and multiplying the result by 100. Weight change greater than 5% in 1 month, 7.5% in 3 months, or 10% in 6 months is considered severe.

A patient has been taking oral prednisone for the past several weeks after having a severe reaction to poison ivy. The nurse has explained the procedure for gradual reduction rather than sudden cessation of the drug. What is the rationale for this approach to drug administration? A. Prevention of hypothyroidism B. Prevention of diabetes insipid us C. Prevention of adrenal insufficiency D. Prevention of cardiovascular complications

C. Prevention of adrenal insufficiency Sudden cessation of corticosteroid therapy can precipitate life-threatening adrenal insufficiency. Diabetes insipidus, hypothyroidism, and cardiovascular complications are not common consequences of suddenly stopping corticosteroid therapy.

The nurse prepares to administer IV ibandronate (Boniva) to a 67-year-old woman with osteoporosis. What is a priority laboratory assessment to make before the administration of ibandronate? A. Serum calcium B. Serum creatinine C. Serum phosphate D. Serum alkaline phosphatase

C. Serum phosphate Ibandronate is a bisphosphonate that is administered IV every 3 months and is administered slowly over 15 to 30 seconds to prevent renal damage. Ibandronate should not be used by patients taking other nephrotoxic drugs or by those with severe renal impairment (defined as serum creatinine above 2.3 mg/dL or creatinine clearance less than 30 mL/min).

Which assessment finding for a patient who has just been admitted with acute pyelonephritis is most important for the nurse to report to the health care provider? a. Foul-smelling urine b. Complaint of flank pain c. Blood pressure 88/45 mm Hg d. Temperature 100.1° F (57.8° C)

C. blood pressure 88/45 mm Hg The low blood pressure indicates that urosepsis and septic shock may be occurring and should be immediately reported. The other findings are typical of pyelonephritis.

6. When the nurse is caring for a patient whose HIV status in unknown, which of these patient exposures is most likely to require postexposure prophylaxis? a. Needle stick with a needle and syringe used to draw blood b. Splash into the eyes when emptying a bedpan containing stool c. Contamination of open skin lesions with patient vaginal secretions d. Needle stick injury with a suture needle during a surgical procedure

A. Needle stick with a needle and shrine used to draw blood Puncture wounds are the most common means for workplace transmission of blood-borne diseases, and a needle with a hollow bore that had been contaminated with the patient's blood would be a high-risk situation. The other situations described would be much less likely to result in transmission of the virus.

Which nursing intervention is most appropriate when turning a patient following spinal surgery? A. Placing a pillow between the patient's legs and turning the body as a unit B. Having the patient turn to the side by grasping the side rails to help turn over C. Elevating the head of bed 30 degrees and having the patient extend the legs while turning D. Turning the patient's head and shoulders and then the hips, keeping the patient's body centered in the bed

A. Placing a pillow between the patient's legs and turning the body as a unit Placing a pillow between the legs and turning the patient as a unit (logrolling) helps to keep the spine in good alignment and reduces pain and discomfort following spinal surgery. Having the patient turn by grasping the side rail to help, elevating the head of the bed, and turning with extended legs or turning the patient's head and shoulders and then the hips will not maintain proper spine alignment and may cause damage.

The nurse is caring for a 56-year-old man receiving high-dose oral corticosteroid therapy to prevent organ rejection after a kidney transplant. What is most important for the nurse to observe related to this medication? A. Signs of infection B. Low blood pressure C. Increased urine output D. Decreased blood glucose

A. Signs of infection Side effects of corticosteroid therapy include increased susceptibility to infection, edema related to sodium and water retention (decreased urine output), hypertension, and hyperglycemia.

21. A 63-year-old patient is newly diagnosed with type 2 diabetes. When developing an education plan, the nurse's first action should be to a. assess the patient's perception of what it means to have type 2 diabetes. b. demonstrate how to check glucose using capillary blood glucose monitoring. c. ask the patient's family to participate in the diabetes education program. d. discuss the need for the patient to actively participate in diabetes management.

A. assess the patients perception of what it means to have type 2 diabetes Rationale: Before planning education, the nurse should assess the patient's interest in and ability to self-manage the diabetes. After assessing the patient, the other nursing actions may be appropriate, but planning needs to be individualized to each patient.

17. To evaluate the effectiveness of ART, the nurse will schedule the patient for a. viral load testing. b. enzyme immunoassay. c. rapid HIV antibody testing. d. immunofluorescence assay.

A. viral load testing The effectiveness of ART is measured by the decrease in the amount of virus detectable in the blood. The other tests are used to detect HIV antibodies, which remain positive even with effective ART.

27. A patient recovering from a gastrojejunostomy (Billroth II) for treatment of a duodenal ulcer develops dizziness, weakness, and palpitations about 20 minutes after eating. To avoid recurrence of these symptoms, the nurse teaches the patient to a. lie down for about 30 minutes after eating. b. choose foods that are high in carbohydrates. c. increase the amount of fluid intake with meals. d. drink sugared fluids or eat candy after each meal.

ANS: A The patient is experiencing symptoms of dumping syndrome, which may be reduced by lying down after eating. Increasing fluid intake and choosing high carbohydrate foods will increase the risk for dumping syndrome. Having a sweet drink or hard candy will correct the hypoglycemia that is associated with dumping syndrome but will not prevent dumping syndrome.

Before assisting a patient with ambulation on the day after a total hip replacement, which action is most important for the nurse to take? a. Administer the ordered oral opioid pain medication. b. Instruct the patient about the benefits of ambulation. c. Ensure that the incisional drain has been discontinued. d. Change the hip dressing and document the wound appearance.

ANS: A The patient should be adequately medicated for pain before any attempt to ambulate. Instructions about the benefits of ambulation may increase the patient's willingness to ambulate, but decreasing pain with ambulation is more important. The presence of an incisional drain or timing of dressing change will not affect ambulation.

The client scheduled for intravenous urography informs the nurse of the following allergies. Which one should the nurse report to the physician immediately? A. Seafood B. Penicillin C. Bee stings D. Red food dye

ANS: A Clients with seafood allergies often have severe allergic reactions to the standard dyes used during intravenous urography.

The client scheduled to have an intravenous urogram is a diabetic and taking the antidiabetic agent metformin. What should the nurse tell this client? A. "Call your diabetes doctor and tell him or her that you are having an intravenous urogram performed using dye." B. "Do not take your metformin the morning of the test because you are not going to be eating anything and could become hypoglycemic." C. "You must start on an antibiotic before this test because your risk of infection is greater as a result of your diabetes." D. "You must take your metformin immediately before the test is performed because the IV fluid and the dye contain a significant amount of sugar."

ANS: A Metformin can cause a lactic acidosis and renal impairment as an interaction with the dye. This drug must be discontinued for 48 hours before the procedure and not started again after the procedure until urine output is well established.

When planning care for a patient who has had hip replacement surgery, which nursing action can the nurse delegate to experienced nursing assistive personnel (NAP)? a. Teach quadriceps-setting exercises. b. Reposition the patient every 1 to 2 hours. c. Assess for skin irritation on the patient's back. d. Determine the patient's pain level and tolerance.

ANS: B Repositioning of patients is within the scope of practice of NAP (after they have been trained and evaluated in this skill). The other actions should be done by licensed nursing staff members.

When doing discharge teaching for a 19-year-old patient who has had a repair of a fractured mandible, the nurse will include information about a. administration of nasogastric tube feedings. b. how and when to cut the immobilizing wires. c. the importance of high-fiber foods in the diet. d. the use of sterile technique for dressing changes.

ANS: B The jaw will be wired for stabilization, and the patient should know what emergency situations require that the wires be cut to protect the airway. There are no dressing changes for this procedure. The diet is liquid, and patients are not able to chew high-fiber foods. Initially, the patient may receive nasogastric tube feedings, but by discharge, the patient will swallow liquid through a straw

The nurse supervises the care of a patient with a temporary radioactive cervical implant. Which action by unlicensed assistive personnel (UAP), if observed by the nurse, would require an intervention? a. The UAP flushes the toilet once after emptying the patient's bedpan. b. The UAP stands by the patient's bed for 30 minutes talking with the patient. c. The UAP places the patient's bedding in the laundry container in the hallway. d. The UAP gives the patient an alcohol-containing mouthwash to use for oral care.

ANS: B Because patients with temporary implants emit radioactivity while the implants are in place, exposure to the patient is limited. Laundry and urine/feces do not have any radioactivity and do not require special precautions. Cervical radiation will not affect the oral mucosa, and alcohol-based mouthwash is not contraindicated.

Which information obtained by the nurse about a 29-year-old patient with a lumbar vertebral compression fracture is most important to report to the health care provider? a. Patient refuses to be turned due to back pain. b. Patient has been incontinent of urine and stool. c. Patient reports lumbar area tenderness to palpation. d. Patient frequently uses oral corticosteroids to treat asthma.

ANS: B Changes in bowel or bladder function indicate possible spinal cord compression and should be reported immediately because surgical intervention may be needed. The other findings are also pertinent but are consistent with the patient's diagnosis and do not require immediate intervention.

The female client's urinalysis shows all the following characteristics. Which should the nurse document as abnormal? A. pH 5.6 B. Ketone bodies present C. Specific gravity is 1.030 D. Two white blood cells per high-power field

ANS: B Ketone bodies are byproducts of incomplete metabolism of fatty acids. Normally, there are no ketones in urine. Ketone bodies are produced when fat sources are used instead of glucose to provide cellular energy.

The client's urine specific gravity is 1.018. What is the nurse's best action? A. Ask the client for a 24-hour recall of liquid intake. B. Document the finding as the only action. C. Obtain a specimen for culture. D. Notify the physician.

ANS: B This specific gravity is within the normal range for urine.

Which of the following muscle actions results in voluntary urination? A. Detrusor contraction, external sphincter contraction B. Detrusor contraction, external sphincter relaxation C. Detrusor relaxation, external sphincter contraction D. Detrusor relaxation, external sphincter relaxation

ANS: B Voiding becomes a voluntary act as a result of learned responses controlled by the cerebral cortex that cause contraction of the bladder detrusor muscle and simultaneous relaxation of the external urethral sphincter muscle.

The nurse teaches a patient who is scheduled for a prostate needle biopsy about the procedure. Which statement, if made by the patient, indicates that teaching was effective? a. "The biopsy will remove the cancer in my prostate gland." b. "The biopsy will determine how much longer I have to live." c. "The biopsy will help decide the treatment for my enlarged prostate." d. "The biopsy will indicate whether the cancer has spread to other organs."

ANS: C A biopsy is used to determine whether the prostate enlargement is benign or malignant, and determines the type of treatment that will be needed. A biopsy does not give information about metastasis, life expectancy, or the impact of cancer on the patient's life.

Which action will the urgent care nurse take when caring for a patient who has a possible knee meniscus injury? a. Encourage bed rest for 24 to 48 hours. b. Avoid palpation or movement of the knee. c. Apply a knee immobilizer to the affected leg. d. Administer intravenous narcotics for pain relief.

ANS: C A knee immobilizer may be used for several days after a meniscus injury to stabilize the knee and minimize pain. Patients are encouraged to ambulate with crutches. The knee is assessed by flexing, internally rotating, and extending the knee (McMurray's test). The pain associated with a meniscus injury will not typically require IV opioid administration; nonsteroidal antiinflammatory drugs (NSAIDs) are usually recommended for pain relief.

A patient arrived at the emergency department after tripping over a rug and falling at home. Which finding is most important for the nurse to communicate to the health care provider? a. There is bruising at the shoulder area. b. The patient reports arm and shoulder pain. c. The right arm appears shorter than the left. d. There is decreased shoulder range of motion.

ANS: C A shorter limb after a fall indicates a possible dislocation, which is an orthopedic emergency. Bruising, pain, and decreased range of motion also should be reported, but these do not indicate that emergent treatment is needed to preserve function.

Which information will the nurse include when discharging a patient with a sprained wrist from the emergency department? a. Keep the wrist loosely wrapped with gauze. b. Apply a heating pad to reduce muscle spasms. c. Use pillows to elevate the arm above the heart. d. Gently move the wrist through the range of motion.

ANS: C Elevation of the arm will reduce the amount of swelling and pain. Compression bandages are used to decrease swelling. For the first 24 to 48 hours, cold packs are used to reduce swelling. The wrist should be rested and kept immobile to prevent further swelling or injury.

Which discharge instruction will the emergency department nurse include for a patient with a sprained ankle? a. Keep the ankle loosely wrapped with gauze. b. Apply a heating pad to reduce muscle spasms. c. Use pillows to elevate the ankle above the heart. d. Gently move the ankle through the range of motion.

ANS: C Elevation of the leg will reduce the amount of swelling and pain. Compression bandages are used to decrease swelling. For the first 24 to 48 hours, cold packs are used to reduce swelling. The ankle should be rested and kept immobile to prevent further swelling or injury.

14. The nurse will plan to teach the patient with newly diagnosed achalasia that a. a liquid or blenderized diet will be necessary. b. drinking fluids with meals should be avoided. c. endoscopic procedures may be used for treatment. d. lying down and resting after meals is recommended.

ANS: C Endoscopic and laparoscopic procedures are the most effective therapy for improving symptoms caused by achalasia. Patients are advised to drink fluid with meals. Keeping the head elevated after eating will improve esophageal emptying. A semisoft diet is recommended to improve esophageal emptying.

When examining an older patient in the home, the home health nurse notices irregular patterns of bruising at different stages of healing on the patient's body. Which action should the nurse take first? a. Discourage the use of throw rugs throughout the house. b. Ensure the patient has a pair of shoes with non-slip soles. c. Talk with the patient alone and ask about what caused the bruising. d. Notify the health care provider so that x-rays can be ordered as soon as possible.

ANS: C The nurse should note irregular patterns of bruising, especially in the shapes of hands or fingers, in different stages of resolution. These may be indications of other health problems or abuse, and should be further investigated. It is important that the nurse interview the patient alone because, if mistreatment is occurring, the patient may not disclose it in the presence of the person who may be the abuser. Throw rugs and shoes with slippery surfaces may contribute to falls. X-rays may be needed if the patient has fallen recently and also has complaints of pain or decreased mobility. However, the nurse's first nursing action is to further assess the patient

41. A patient with acute gastrointestinal (GI) bleeding is receiving normal saline IV at a rate of 500 mL/hr. Which assessment finding obtained by the nurse is most important to communicate immediately to the health care provider? a. The patient's blood pressure (BP) has increased to 142/94 mm Hg. b. The nasogastric (NG) suction is returning coffee-ground material. c. The patient's lungs have crackles audible to the midline. d. The bowel sounds are very hyperactive in all four quadrants.

ANS: C The patient's lung sounds indicate that pulmonary edema may be developing as a result of the rapid infusion of IV fluid and that the fluid infusion rate should be slowed. The return of coffee-ground material in an NG tube is expected for a patient with upper GI bleeding. The BP is slightly elevated but would not be an indication to contact the health care provider immediately. Hyperactive bowel sounds are common when a patient has GI bleeding.

When a patient arrives in the emergency department with a facial fracture, which action will the nurse take first? a. Assess for nasal bleeding and pain. b. Apply ice to the face to reduce swelling. c. Use a cervical collar to stabilize the spine. d. Check the patient's alertness and orientation.

ANS: C Patients who have facial fractures are at risk for cervical spine injury and should be treated as if they have a cervical spine injury until this is ruled out. The other actions are also necessary, but the most important action is to prevent cervical spine injury.

The nurse is caring for a patient receiving intravesical bladder chemotherapy. The nurse should monitor for which adverse effect? a. Nausea b. Alopecia c. Mucositis d. Hematuria

ANS: D The adverse effects of intravesical chemotherapy are confined to the bladder. The other adverse effects are associated with systemic chemotherapy.

The nurse is caring for a patient receiving intravesical bladder chemotherapy. The nurse should monitor for which adverse effect? a. Nausea b. Alopecia c. Mucositis d. Hematuria

ANS: D The adverse effects of intravesical chemotherapy are confined to the bladder. The other adverse effects are associated with systemic chemotherapy. DIF: Cognitive Level: Apply (application) REF: 261 TOP: Nursing Process: Evaluation MSC: NCLEX: Physiological Integrity

A woman calls the clinic because she is having an unusually heavy menstrual flow. She tells the nurse that she has saturated three tampons in the past 2 hours. The nurse estimates that the amount of blood loss over the past 2 hours is _____ mL. a. 20 to 30 b. 30 to 40 c. 40 to 60 d. 60 to 90

ANS: D The average tampon absorbs 20 to 30 mL.

31. A patient with a recent 20-pound unintended weight loss is diagnosed with stomach cancer. Which nursing action will be included in the plan of care? a. Refer the patient for hospice services. b. Infuse IV fluids through a central line. c. Teach the patient about antiemetic therapy. d. Offer supplemental feedings between meals.

ANS: D The patient data indicate a poor nutritional state and improvement in nutrition will be helpful in improving response to therapies such as surgery, chemotherapy, or radiation. Nausea and vomiting are not common clinical manifestations of stomach cancer. There is no indication that the patient requires hospice or IV fluid infusions.

The nurse is caring for a patient who is to be discharged from the hospital 5 days after insertion of a femoral head prosthesis using a posterior approach. Which statement by the patient indicates a need for additional instruction? a. "I should not cross my legs while sitting." b. "I will use a toilet elevator on the toilet seat." c. "I will have someone else put on my shoes and socks." d. "I can sleep in any position that is comfortable for me."

ANS: D The patient needs to sleep in a position that prevents excessive internal rotation or flexion of the hip. The other patient statements indicate that the patient has understood the teaching.

The day after a 60-year-old patient has an open reduction and internal fixation (ORIF) for an open, displaced tibial fracture, the priority nursing diagnosis is a. activity intolerance related to deconditioning. b. risk for constipation related to prolonged bed rest. c. risk for impaired skin integrity related to immobility. d. risk for infection related to disruption of skin integrity.

ANS: D A patient having an ORIF is at risk for problems such as wound infection and osteomyelitis. After an ORIF, patients typically are mobilized starting the first postoperative day, so problems caused by immobility are not as likely

33. Which assessment finding in a patient who had a total gastrectomy 12 hours previously is most important to report to the health care provider? a. Absent bowel sounds b. Scant nasogastric (NG) tube drainage c. Complaints of incisional pain d. Temperature 102.1° F (38.9° C)

ANS: D An elevation in temperature may indicate leakage at the anastomosis, which may require return to surgery or keeping the patient NPO. The other findings are expected in the immediate postoperative period for patients who have this surgery.

The nurse obtains information about a hospitalized patient who is receiving chemotherapy for colorectal cancer. Which information about the patient alerts the nurse to discuss a possible change in therapy with the health care provider? a. Poor oral intake b. Frequent loose stools c. Complaints of nausea and vomiting d. Increase in carcinoembryonic antigen (CEA)

ANS: D An increase in CEA indicates that the chemotherapy is not effective for the patient's cancer and may need to be modified. The other patient findings are common adverse effects of chemotherapy. The nurse may need to address these, but they would not necessarily indicate a need for a change in therapy.

The nurse assesses a patient who is receiving interleukin-2. Which finding should the nurse report immediately to the health care provider? a. Generalized muscle aches b. Complaints of nausea and anorexia c. Oral temperature of 100.6° F (38.1° C) d. Crackles heard at the lower scapular border

ANS: D Capillary leak syndrome and acute pulmonary edema are possible toxic effects of interleukin-2. The patient may need oxygen and the nurse should rapidly notify the health care provider. The other findings are common side effects of interleukin-2

The nurse will plan to teach a 51-year-old man who is scheduled for an annual physical exam about a(n) a. increased risk for testicular cancer. b. possible changes in erectile function. c. normal decreases in testosterone level. d. prostate specific antigen (PSA) testing.

ANS: D PSA testing may be recommended annually for men, starting at age 50. There is no indication that the other patient teaching topics are appropriate for this patient

Which information obtained by the emergency department nurse when admitting a patient with a left femur fracture is most important to report to the health care provider? a. Bruising of the left thigh b. Complaints of left thigh pain c. Outward pointing toes on the left foot d. Prolonged capillary refill of the left foot

ANS: D Prolonged capillary refill may indicate complications such as arterial damage or compartment syndrome. The other findings are typical with a left femur fracture.

An older adult patient who has colorectal cancer is receiving IV fluids at 175 mL/hour in conjunction with the prescribed chemotherapy. Which finding by the nurse is most important to report to the health care provider? a. Patient complains of severe fatigue. b. Patient needs to void every hour during the day. c. Patient takes only 50% of meals and refuses snacks. d. Patient has audible crackles to the midline posterior chest.

ANS: D Rapid fluid infusions may cause heart failure, especially in older patients. The other findings are common in patients who have cancer and/or are receiving chemotherapy

After change-of-shift report, which patient should the nurse assess first? a. Patient with a Colles' fracture who has right wrist swelling and deformity b. Patient with a intracapsular left hip fracture whose leg is externally rotated c. Patient with a repaired mandibular fracture who is complaining of facial pain d. Patient with right femoral shaft fracture whose thigh is swollen and ecchymotic

ANS: D Swelling and bruising after a femoral shaft fracture suggest hemorrhage and risk for compartment syndrome. The nurse should assess the patient rapidly and then notify the health care provider. The other patients have symptoms that are typical for their injuries, but do not require immediate intervention.

A 54-year-old patient admitted with diabetes mellitus, malnutrition, osteomyelitis, and alcohol abuse has a serum amylase level of 280 U/L and a serum lipase level of 310 U/L. To what diagnosis does the nurse attribute these findings? Malnutrition Osteomyelitis Alcohol abuse Diabetes mellitus

Alcohol abuse The patient with alcohol abuse could develop pancreatitis as a complication, which would increase the serum amylase (normal 30-122 U/L) and serum lipase (normal 31-186 U/L) levels as shown.

C

An abnormal finding noted during physical assessment of the male reproductive system is: A. descended testes B. symmetric scrotum C. slight clear urethral discharge D. the glans covered with a prepuce

B

An age-related finding noted by the nurse during assessment of the older woman's reproductive system is: A. dyspareunia B. vaginal dryness C. nipple retraction D. increased sensitivity of the labia

A patient with bladder cancer is scheduled for intravesical chemotherapy. In preparation for the treatment the nurse will teach the patient about a. premedicating to prevent nausea. b. where to obtain wigs and scarves. c. the importance of oral care during treatment. d. the need to empty the bladder before treatment.

D. the need to empty the bladder before treatment The patient will be asked to empty the bladder before instillation of the chemotherapy. Systemic side effects are not experienced with intravesical chemotherapy.

The condition of a patient who has cirrhosis of the liver has deteriorated. Which diagnostic study would help determine if the patient has developed liver cancer? Serum α-fetoprotein level Ventilation/perfusion scan Hepatic structure ultrasound Abdominal girth measurement

Hepatic structure ultrasound Hepatic structure ultrasound, CT, and MRI are used to screen and diagnose liver cancer. Serum α-fetoprotein level may be elevated with liver cancer or other liver problems. Ventilation/perfusion scans do not diagnose liver cancer. Abdominal girth measurement would not differentiate between cirrhosis and liver cancer.

The family of a patient newly diagnosed with hepatitis A asks the nurse what they can do to prevent becoming ill themselves. Which response by the nurse is most appropriate? "The hepatitis vaccine will provide immunity from this exposure and future exposures." "I am afraid there is nothing you can do since the patient was infectious before admission." "You will need to be tested first to make sure you don't have the virus before we can treat you." "An injection of immunoglobulin will need to be given to prevent or minimize the effects from this exposure."

"An injection of immunoglobulin will need to be given to prevent or minimize the effects from this exposure." Immunoglobulin provides temporary (1-2 months) passive immunity and is effective for preventing hepatitis A if given within 2 weeks after exposure. It may not prevent infection in all persons, but it will at least modify the illness to a subclinical infection. The hepatitis vaccine is only used for preexposure prophylaxis.

The patient states that she "loses urine" every time she laughs or coughs. The nurse teaches the patient measures to regain urinary control. The nurse recognizes the need for further teaching when the patient states: A) "I will perform my Kegel exercises every day." B) "I joined weight watchers." C) "I drink two glasses of wine with dinner." D) "I have tried urinating every 3 hours."

"I drink two glasses of wine with dinner."

2. A nurse is caring for a client who has chronic renal failure. Which of the following client statements indicates an understanding of the dietary needs for lifestyle management of this disease? -"I will limit my fluid intake" -"I will eat a diet high in potassium" -"I will eat a diet high in carbohydrates" -"I will eat a diet high in protein"

"I will limit my fluid intake"

The nurse instructs a 50-year-old woman about cholestyramine to reduce pruritis caused by gallbladder disease. Which statement by the patient to the nurse indicates she understands the instructions? "This medication will help me digest fats and fat-soluble vitamins." "I will apply the medicated lotion sparingly to the areas where I itch." "The medication is a powder and needs to be mixed with milk or juice." "I should take this medication on an empty stomach at the same time each day."

"The medication is a powder and needs to be mixed with milk or juice." For treatment of pruritus, cholestyramine may provide relief. This is a resin that binds bile salts in the intestine, increasing their excretion in the feces. Cholestyramine is in powder form and should be mixed with milk or juice before oral administration.

Prevention of AKI is important because of the high mortality rate. Which patients are at increased risk for AKI (select all that apply)? a. An 86-year-old woman scheduled for a cardiac catheterization b. A 48-year-old man with multiple injuries from a motor vehicle accident c. A 32-year-old woman following a C-section delivery for abruptio placentae d. A 64-year-old woman with chronic heart failure admitted with bloody stools e. A 58-year-old man with prostate cancer undergoing preoperative workup for prostatectomy

a, b, c, d, e. High-risk patients include those exposed to nephrotoxic agents and advanced age (a), massive trauma (b), prolonged hypovolemia or hypotension (possibly b and c), obstetric complications (c), cardiac failure (d), preexisting chronic kidney disease, extensive burns, or sepsis. Patients with prostate cancer may have obstruction of the outflow tract, which increases risk of postrenal AKI (e).

After a transurethral prostatectomy a patient returns to his room with a triple-lumen indwelling catheter and continuous bladder irrigation. The irrigation is normal saline at 150 mL/hr. The nurse empties the drainage bag for a total of 2520 mL after an 8-hour period. How much of the total is urine output?

1320

The nurse directs the NAP to remove a Foley catheter at 1300. The nurse would check if the patient has voided by: A) 1400. B) 1600 C) 1700. D) 2300.

1700.

A 22-year-old patient is being seen in the clinic with increased secretion of the anterior pituitary hormones. The nurse would expect the laboratory results to show a. increased urinary cortisol. b. decreased serum thyroxine. c. elevated serum aldosterone levels. d. low urinary catecholamines excretion.

A

The nurse is caring for a 63-year-old with a possible pituitary tumor who is scheduled for a computed tomography (CT) scan with contrast. Which information about the patient is most important to discuss with the health care provider before the test? a. History of renal insufficiency b. Complains of chronic headache c. Recent bilateral visual field loss d. Blood glucose level of 134 mg/dL

A

The nurse is caring for a group of patients. Which patient is at highest risk for pancreatic cancer? A 38-year-old Hispanic female who is obese and has hyperinsulinemia A 23-year-old who has cystic fibrosis-related pancreatic enzyme insufficiency A 72-year-old African American male who has smoked cigarettes for 50 years A 19-year-old who has a 5-year history of uncontrolled type 1 diabetes mellitus

A 72-year-old African American male who has smoked cigarettes for 50 years Risk factors for pancreatic cancer include chronic pancreatitis, diabetes mellitus, age, cigarette smoking, family history of pancreatic cancer, high-fat diet, and exposure to chemicals such as benzidine. African Americans have a higher incidence of pancreatic cancer than whites. The most firmly established environmental risk factor is cigarette smoking. Smokers are two or three times more likely to develop pancreatic cancer as compared with nonsmokers. The risk is related to duration and number of cigarettes smoked.

The patient had frostbite on the distal areas of the toes on both feet. The patient is scheduled for amputation of the damaged tissue. Which assessment finding or diagnostic study is the most objective indicator for locating the level of the patient's injury? A Arteriography showing blood vessels B Peripheral pulse assessment bilaterally C Patches of black, indurated, and cold tissue D Bilateral pale and cool skin below the ankles

A Arteriography showing blood vessels Arteriography is the most objective study to determine viable tissue for salvage based on perfusion because actual blood flow through the tissues is observed in real time. It is considered the gold standard for evaluating arterial perfusion. Bilateral peripheral pulse assessment, areas of black, indurated, and cold tissue, and bilateral pale and cool skin all identify the lack of tissue perfusion, but not the specific area where tissue perfusion stops and amputation needs to occur.

The nurse is caring for a 76-year-old man who has undergone left knee arthroplasty with prosthetic replacement of the knee joint to relieve the pain of severe osteoarthritis. Postoperatively the nurse expects what to be included in the care of the affected leg? A Progressive leg exercises to obtain 90-degree flexion B Early ambulation with full weight bearing on the left leg C Bed rest for 3 days with the left leg immobilized in extension D Immobilization of the left knee in 30-degree flexion for 2 weeks to prevent dislocation

A Progressive leg exercises to obtain 90-degree flexion Although early ambulation is not done, the patient is encouraged to engage in progressive leg exercises until 90-degree flexion is possible. Because this is painful after surgery, the patient requires good pain management and often the use of a CPM machine. The patient's knee is unlikely to dislocate.

Pheochromocytoma

A benign tumor of the adrenal medulla that causes the gland to produce excess epinephrine

c

A father who has an X-linked recessive disorder and a wife with a normal genotype will: A. pass the carrier state to his make child B. pass the carrier state to all of his children C. pass the carrier state to his female child D. not pass on the genetic mutation to any of is children

C

A normal reproductive function that may be altered in a patient who undergoes a prostatectomy is: A. sperm production B. production of testosterone C. production of seminal fluid D. release of sperm from the epididymis

D

A nursing intervention that is most appropriate to decrease postoperative edema and pain following an inguinal herniorraphy is: A. applying a truss to the hernia site B. allowing the patient to stand to void C. supporting the incision during coughing D. applying a scrotal support with ice bag

d. When administering immunotherapy, it is imperative to closely monitor the patient for any signs of an adverse reaction. The high risk and significant consequence of an adverse reaction supersede the need to assess the patient's fluid balance, whereas pain and changes in level of consciousness are not likely events when administering immunotherapy.

A patient has begun immunotherapy for the treatment of intractable environmental allergies. When administering the patient's immunotherapy, which of the following is the nurse's priority action? A. Monitoring the patient's fluid balance B. Assessing the patient's need for analgesia C. Assessing the patient for changes in level of consciousness D. Monitoring for signs and symptoms of an adverse reaction

c

A patient is undergoing plasmapheresis for treatment of systemic lupus erythematosus. The nurse explains that plasmapheresis is used in her treatment to: A. remove T lymphocytes in her blood that are producing antinuclear antibodies B. remove normal particles in her blood that are being damaged by autoantibodies C. exchange her plasma that contains antinuclear antibodies with a substitute fluid D. replace viral-damaged cellular components of her blood with replacement whole blood

C

A patient with metastatic colorectal cancer is scheduled for both chemotherapy and radiation. Patient teaching regarding these therapies for this patient would include an explanation that: A. chemotherapy can be used to cure colorectal cancer B. radiation is commonly used as adjuvant therapy following surgery C. both chemotherapy and radiation can be used as palliative treatments D. the patient should expect few if any side effects from the chemotherapeutic agents

b. Transfusion reactions are characterized as a type II (cytotoxic) reaction in which agglutination and cytolysis occur.

A patient's low hemoglobin and hematocrit have necessitated a transfusion of packed red blood cells (PRBCs). Shortly after the first unit of PRBCs is hung, the patient develops signs and symptoms of a transfusion reaction. Which of the following hypersensitivity reactions has the patient experienced? A. Type I B. Type II C. Type III D. Type IV

To evaluate the female patients breasts you would use the examination techniques of (select all that apply): A. palpation B. percussion C. inspection D. auscultation

A, C

11. The nurse has been teaching the patient to administer a dose of 10 units of regular insulin and 28 units of NPH insulin. The statement by the patient that indicates a need for additional instruction is, a. "I need to rotate injection sites among my arms, legs, and abdomen each day." b. "I will buy the 0.5-ml syringes because the line markings will be easier to see." c. "I should draw up the regular insulin first after injecting air into the NPH bottle." d. "I do not need to aspirate the plunger to check for blood before I inject the insulin."

A. "I need to rotate injection sites among my arms, legs, and abdomen each day." Rationale: Rotating sites is no longer necessary because all insulin is now purified human insulin, and the risk for lipodystrophy is low. The other patient statements are accurate and indicate that no additional instruction is needed.

Analyze the following diagnostic fings for your patient with type 2 diabetes. Which result will need further assessment? a) A1C 9% b) BP 126/80 mmHg c) FSBS 130 mg/dL d) LDL Cholesterol 100 mg/dL

A. A1C 9%

32. Amitriptyline (Elavil) is prescribed for a diabetic patient with peripheral neuropathy who has burning foot pain occurring mostly at night. Which information should the nurse include when teaching the patient about the new medication? a. Amitriptyline will help prevent the transmission of pain impulses to the brain. b. Amitriptyline will improve sleep and make you less aware of nighttime pain. c. Amitriptyline will decrease the depression caused by the pain. d. Amitriptyline will correct some of the blood vessel changes that cause pain.

A. Amitriptyline will help prevent the transmission of pain impulses to the brain. Rationale: Tricyclic antidepressants decrease the transmission of pain impulses to the spinal cord and brain. Tricyclics also improve sleep quality and are used for depression, but that is not the major purpose for their use in diabetic neuropathy. The blood vessel changes that contribute to neuropathy are not affected by tricyclics.

11. The nurse is reviewing the chart for a patient who is scheduled for an annual physical exam. Which information will be most important in determining whether the patient needs HIV testing? a. Patient age b. Patient lifestyle c. Patient symptoms d. Patient sexual orientation

A. Patient age The current CDC policy is to offer routine testing for HIV to all individuals age 13 to 64. Although lifestyle, symptoms, and sexual orientation may suggest increased risk for HIV infection, the goal is to test all individuals in this age range.

22. Cardiac monitoring is initiated for a patient in diabetic ketoacidosis (DKA). The nurse recognizes that this measure is important to identify a. electrocardiographic (ECG) changes and dysrhythmias related to hypokalemia. b. fluid overload resulting from aggressive fluid replacement. c. the presence of hypovolemic shock related to osmotic diuresis. d. cardiovascular collapse resulting from the effects of hyperglycemia.

A. electrocardiographic (ECG) changes and dysrhythmias related to hypokalemia. Rationale: The hypokalemia associated with metabolic acidosis can lead to potentially fatal dysrhythmias such as ventricular tachycardia and ventricular fibrillation, which would be detected with ECG monitoring. Fluid overload, hypovolemia, and cardiovascular collapse are possible complications of DKA, but cardiac monitoring would not detect theses.

26. While hospitalized and recovering from an episode of diabetic ketoacidosis, the patient calls the nurse and reports feeling anxious, nervous, and sweaty. Based on the patient's report, the nurse should a. obtain a glucose reading using a finger stick. b. administer 1 mg glucagon subcutaneously. c. have the patient eat a candy bar. d. have the patient drink 4 ounces of orange juice.

A. obtain a glucose reading using a finger stick. Rationale: The patient's clinical manifestations are consistent with hypoglycemia and the initial action should be to check the patient's glucose with a finger stick or order a stat blood glucose. If the glucose is low, the patient should ingest a rapid-acting carbohydrate, such as orange juice. Glucagon might be given if the patient's symptoms become worse or if the patient is unconscious. Candy bars contain fat, which would slow down the absorption of sugar and delay the response to treatment.

A patient in the hospital has a history of functional urinary incontinence. Which nursing action will be included in the plan of care? a. Place a bedside commode near the patient's bed. b. Demonstrate the use of the Credé maneuver to the patient. c. Use an ultrasound scanner to check postvoiding residuals. d. Teach the use of Kegel exercises to strengthen the pelvic floor.

A. place a bedside commode near the patient's bed Modifications in the environment make it easier to avoid functional incontinence. Checking for residual urine and performing the Credé maneuver are interventions for overflow incontinence. Kegel exercises are useful for stress incontinence.

When admitting a patient with acute glomerulonephritis, it is most important that the nurse ask the patient about a. recent sore throat and fever. b. history of high blood pressure. c. frequency of bladder infections. d. family history of kidney stones.

A. recent sore throat and fever Acute glomerulonephritis frequently occurs after a streptococcal infection such as strep throat. It is not caused by hypertension, urinary tract infection (UTI), or kidney stones.

Which are appropriate therapies for patients with diabetes mellitus? (Select all that apply) a) Use of statins to treat dyslipidemia b) Use of diuretics to treat nephropathy c) Use of ACE inhibitors to treat nephropathy d) Use of serotonin agonists to decrease appetite e) Use of laser photocoagulation to treat retinopathy

A. use of statins to treat dyslipidemia C. use of ACE inhibitors to treat nephropathy E. use of laser photocoagulation to treat retinopathy

In which order will the nurse take these actions when caring for a patient with left leg fractures after a motor vehicle accident? Put a comma and space between each answer choice (a, b, c, d, etc.) ____________________ a. Obtain x-rays. b. Check pedal pulses. c. Assess lung sounds. d. Take blood pressure. e. Apply splint to the leg. f. Administer tetanus prophylaxis.

ANS: C, D, B, E, A, F The initial actions should be to ensure that airway, breathing, and circulation are intact. This should be followed by checking the neurovascular status of the leg (before and after splint application). Application of a splint to immobilize the leg should be done before sending the patient for x-rays. The tetanus prophylaxis is the least urgent of the actions.

A patient undergoing external radiation has developed a dry desquamation of the skin in the treatment area. The nurse teaches the patient about management of the skin reaction. Which statement, if made by the patient, indicates the teaching was effective? a. "I can buy some aloe vera gel to use on the area." b. "I will expose the treatment area to a sun lamp daily." c. "I can use ice packs to relieve itching in the treatment area." d. "I will scrub the area with warm water to remove the scales."

ANS: A Aloe vera gel and cream may be used on the radiated skin area. Ice and sunlamps may injure the skin. Treatment areas should be cleaned gently to avoid further injury.

When performing a skin assessment, the nurse notes several angiomas on the chest of an older patient. Which action should the nurse take next? a. Assess the patient for evidence of liver disease. b. Discuss the adverse effects of sun exposure on the skin. c. Teach the patient about possible skin changes with aging. d. Suggest that the patient make an appointment with a dermatologist.

ANS: A Angiomas are a common occurrence as patients get older, but they may occur with systemic problems such as liver disease. The patient may want to see a dermatologist to have the angiomas removed, but this is not the initial action by the nurse. The nurse may need to teach the patient about the effects of aging on the skin and about the effects of sun exposure, but the initial action should be further assessment

13. A patient has just arrived on the postoperative unit after having a laparoscopic esophagectomy for treatment of esophageal cancer. Which nursing action should be included in the postoperative plan of care? a. Elevate the head of the bed to at least 30 degrees. b. Reposition the nasogastric (NG) tube if drainage stops or decreases. c. Notify the doctor immediately about bloody NG drainage. d. Start oral fluids when the patient has active bowel sounds.

ANS: A Elevation of the head of the bed decreases the risk for reflux and aspiration of gastric secretions. The NG tube should not be repositioned without consulting with the health care provider. Bloody NG drainage is expected for the first 8 to 12 hours. A swallowing study is needed before oral fluids are started.

19. A patient is hospitalized with vomiting of "coffee-ground" emesis. The nurse will anticipate preparing the patient for a. endoscopy. b. angiography. c. gastric analysis testing. d. barium contrast studies.

ANS: A Endoscopy is the primary tool for visualization and diagnosis of upper gastrointestinal (GI) bleeding. Angiography is used only when endoscopy cannot be done because it is more invasive and has more possible complications. Gastric analysis testing may help with determining the cause of gastric irritation, but it is not used for acute GI bleeding. Barium studies are helpful in determining the presence of gastric lesions, but not whether the lesions are actively bleeding.

Interleukin-2 (IL-2) is used as adjuvant therapy for a patient with metastatic renal cell carcinoma. Which information should the nurse include when explaining the purpose of this therapy to the patient? a. IL-2 enhances the immunologic response to tumor cells. b. IL-2 stimulates malignant cells in the resting phase to enter mitosis. c. IL-2 prevents the bone marrow depression caused by chemotherapy. d. IL-2 protects normal cells from the harmful effects of chemotherapy.

ANS: A IL-2 enhances the ability of the patient's own immune response to suppress tumor cells. IL-2 does not protect normal cells from damage caused by chemotherapy, stimulate malignant cells to enter mitosis, or prevent bone marrow depression.

18. A patient who has had several episodes of bloody diarrhea is admitted to the emergency department. Which action should the nurse anticipate taking? a. Obtain a stool specimen for culture. b. Administer antidiarrheal medications. c. Teach about adverse effects of nonsteroidal anti-inflammatory drugs (NSAIDs). d. Provide education about antibiotic therapy.

ANS: A Patients with bloody diarrhea should have a stool culture for E. coli O157:H7. NSAIDs may cause occult blood in the stools, but not diarrhea. Antidiarrheal medications usually are avoided for possible infectious diarrhea to avoid prolonging the infection. Antibiotic therapy in the treatment of infectious diarrhea is controversial because it may precipitate kidney complications.

Which nursing action for a patient who has had right hip replacement surgery can the nurse delegate to experienced unlicensed assistive personnel (UAP)? a. Reposition the patient every 1 to 2 hours. b. Assess for skin irritation on the patient's back. c. Teach the patient quadriceps-setting exercises. d. Determine the patient's pain level and tolerance.

ANS: A Repositioning of orthopedic patients is within the scope of practice of UAP (after they have been trained and evaluated in this skill). The other actions should be done by licensed nursing staff members.

The nurse assesses a patient with non-Hodgkin's lymphoma who is receiving an infusion of rituximab (Rituxan). Which assessment finding would require the most rapid action by the nurse? a. Shortness of breath b. Temperature 100.2° F (37.9° C) c. Shivering and complaint of chills d. Generalized muscle aches and pains

ANS: A Rituximab (Rituxan) is a monoclonal antibody. Shortness of breath should be investigated rapidly because anaphylaxis is a possible reaction to monoclonal antibody administration. The nurse will need to rapidly take actions such as stopping the infusion, assessing the patient further, and notifying the health care provider. The other findings will also require action by the nurse, but are not indicative of life-threatening complications

The nurse is assessing the sexual-reproductive functional health pattern of a 32-year-old woman. Which question is most useful in determining the patient's sexual orientation and risk factors? a. "Do you have sex with men, women, or both?" b. "Which gender do you prefer to have sex with?" c. "What types of sexual activities do you prefer?" d. "Are you heterosexual, homosexual, or bisexual?"

ANS: A This question is the most simply stated and will increase the likelihood of obtaining the relevant information about sexual orientation and possible risk factors associated with sexual activity. A patient who prefers sex with women may also have intercourse at times with men. The types of sexual activities engaged in may not indicate sexual orientation. Many patients who have sex with both men and women do not identify themselves as homosexual or bisexual.

What would be the response if a person's nephrons were not able to filter normally due to scarring of the proximal convoluted tubule leading to inhibition of reabsorption? A. Increased urine output, fluid volume deficit B. Decreased urine output, fluid volume deficit C. Increased urine output, fluid volume overload D. Decreased urine output, fluid volume overload

ANS: A The nephrons filter about 120 mL/min. Most of this filtrate is reabsorbed in the proximal convoluted tubule. If the tubule were not able to reabsorb the fluid that has been filtered, urine output would greatly increase, leading to rapid and severe dehydration.

Which activities can the nurse working in the outpatient clinic delegate to a licensed practical/vocational nurse (LPN/LVN) (select all that apply)? a. Administer patch testing to a patient with allergic dermatitis. b. Interview a new patient about chronic health problems and allergies. c. Apply a sterile dressing after the health care provider excises a mole. d. Teach a patient about site care after a punch biopsy of an upper arm lesion. e. Explain potassium hydroxide testing to a patient with a superficial skin infection.

ANS: A, C Skills such as administration of patch testing and sterile dressing technique are included in LPN/LVN education and scope of practice. Obtaining a health history and patient education require more critical thinking and registered nurse (RN) level education and scope of practice

After scheduling a patient with a possible ovarian cyst for ultrasound, the nurse will teach the patient that she should a. expect to receive IV contrast during the procedure. b. drink several glasses of fluids before the procedure. c. experience mild abdominal cramps after the procedure. d. discontinue taking aspirin for 7 days before the procedure.

ANS: B A full bladder is needed for many ultrasound procedures, so the nurse will have the patient drink fluids before arriving for the ultrasound. The other instructions are not accurate for this procedure

34. Which information about a patient who has just been admitted to the hospital with nausea and vomiting will require the most rapid intervention by the nurse? a. The patient has taken only sips of water. b. The patient is lethargic and difficult to arouse. c. The patient's chart indicates a recent resection of the small intestine. d. The patient has been vomiting several times a day for the last 4 days.

ANS: B A lethargic patient is at risk for aspiration, and the nurse will need to position the patient to decrease aspiration risk. The other information also is important to collect, but it does not require as quick action as the risk for aspiration.

2. A patient who has been NPO during treatment for nausea and vomiting caused by gastric irritation is to start oral intake. Which of these should the nurse offer to the patient? a. A glass of orange juice b. A dish of lemon gelatin c. A cup of coffee with cream d. A bowl of hot chicken broth

ANS: B Clear liquids are usually the first foods started after a patient has been nauseated. Acidic foods such as orange juice, very hot foods, and coffee are poorly tolerated when patients have been nauseated.

A patient reports chronic itching of the ankles and continuously scratches the area. Which assessment finding will the nurse expect? a. Hypertrophied scars on both ankles b. Thickening of the skin around the ankles c. Yellowish-brown skin around both ankles d. Complete absence of melanin in both ankles

ANS: B Lichenification is likely to occur in areas where the patient scratches the skin frequently. Lichenification results in thickening of the skin with accentuated normal skin markings. Vitiligo is the complete absence of melanin in the skin. Keloids are hypertrophied scars. Yellowish-brown skin indicates jaundice. Vitiligo, keloids, and jaundice do not usually occur as a result of scratching the skin

A patient who is to have no weight bearing on the left leg is learning to walk using crutches. Which observation by the nurse indicates that the patient can safely ambulate independently? a. The patient moves the right crutch with the right leg and then the left crutch with the left leg. b. The patient advances the left leg and both crutches together and then advances the right leg. c. The patient uses the bedside chair to assist in balance as needed when ambulating in the room. d. The patient keeps the padded area of the crutch firmly in the axillary area when ambulating.

ANS: B Patients are usually taught to move the crutches and the injured leg forward at the same time and then to move the unaffected leg. Patients are discouraged from using furniture to assist with ambulation. The patient is taught to place weight on the hands, not in the axilla, to avoid nerve damage. If the 2- or 4-point gaits are to be used, the crutch and leg on opposite sides move forward, not the crutch and same-side leg

The day after a having a right below-the-knee amputation, a patient complains of pain in the right foot. Which action is best for the nurse to take? a. Explain the reasons for the phantom limb pain. b. Administer prescribed analgesics to relieve the pain. c. Loosen the compression bandage to decrease incisional pressure. d. Inform the patient that this phantom pain will diminish over time.

ANS: B Phantom limb sensation is treated like any other type of postoperative pain would be treated. Explanations of the reason for the pain may be given, but the nurse should still medicate the patient. The compression bandage is left in place except during physical therapy or bathing. Although the pain may decrease over time, it still requires treatment now.

A woman in the emergency department after a motor vehicle crash is scheduled for chest and abdominal x-rays. Which information is most important to report to the health care provider before the x-rays are obtained? a. Severity of abdominal pain b. Positive result of hCG test c. Blood pressure 172/88 mm Hg d. Temperature 102.1° F (38.9° C)

ANS: B Positive hCG testing indicates that the patient is pregnant and that unnecessary abdominal x-rays should be avoided. The other information is also important to report, but it will not affect whether the x-rays should be done

The nurse administers an IV vesicant chemotherapeutic agent to a patient. Which action is most important for the nurse to take? a. Infuse the medication over a short period of time. b. Stop the infusion if swelling is observed at the site. c. Administer the chemotherapy through a small-bore catheter. d. Hold the medication unless a central venous line is available.

ANS: B Swelling at the site may indicate extravasation, and the IV should be stopped immediately. The medication generally should be given slowly to avoid irritation of the vein. The size of the catheter is not as important as administration of vesicants into a running IV line to allow dilution of the chemotherapeutic drug. These medications can be given through peripheral lines, although central vascular access devices (CVADs) are preferred.

A patient undergoes a right above-the-knee amputation with an immediate prosthetic fitting. When the patient first arrives on the orthopedic unit after surgery, the nurse should a. place the patient in a prone position. b. check the surgical site for hemorrhage. c. remove the prosthesis and wrap the site. d. keep the residual leg elevated on a pillow.

ANS: B The nurse should monitor for hemorrhage after the surgery. The prosthesis will not be removed. To avoid flexion contracture of the hip, the leg will not be elevated on a pillow. The patient is placed in a prone position after amputation to prevent hip flexion, but this would not be done during the immediate postoperative period.

A patient has a long-arm plaster cast applied for immobilization of a fractured left radius. Until the cast has completely dried, the nurse should a. keep the left arm in a dependent position. b. handle the cast with the palms of the hands. c. place gauze around the cast edge to pad any roughness. d. cover the cast with a small blanket to absorb the dampness.

ANS: B Until a plaster cast has dried, placing pressure on the cast should be avoided to prevent creating areas inside the cast that could place pressure on the arm. The left arm should be elevated to prevent swelling. The edges of the cast may be petaled once the cast is dry, but padding the edges before that may cause the cast to be misshapen. The cast should not be covered until it is dry because heat builds up during drying.

Which of the following observations made by the nurse who is evaluating the crutch-walking technique of a patient who is to have no weight bearing on the right leg indicates that the patient can safely ambulate independently? a. The patient keeps the padded area of the crutch firmly in the axillary area when ambulating. b. The patient advances the right leg and both crutches together and then advances the left leg. c. The patient moves the left crutch with the left leg and then the right crutch with the right leg. d. The patient uses the bedside chair to assist in balance as needed when ambulating in the room.

ANS: B When using crutches, patients are usually taught to move the assistive device and the injured leg forward at the same time and then to move the unaffected leg. Patients are discouraged from using furniture to assist with ambulation. The patient is taught to place weight on the hands, not in the axilla, to avoid nerve damage. If the 2- or 4-point gaits are to be used, the crutch and leg on opposite sides move forward, not the crutch and same-side leg.

The nurse teaches a patient with cancer of the liver about high-protein, high-calorie diet choices. Which snack choice by the patient indicates that the teaching has been effective? a. Lime sherbet b. Blueberry yogurt c. Cream cheese bagel d. Fresh strawberries and bananas

ANS: B Yogurt has high biologic value because of the protein and fat content. Fruit salad does not have high amounts of protein or fat. Lime sherbet is lower in fat and protein than yogurt. Cream cheese is low in protein.

The client is taking a medication for an endocrine problem that inhibits aldosterone secretion and release. To what complications of this therapy should the nurse be alert? A. Dehydration, hypokalemia B. Dehydration, hyperkalemia C. Overhydration, hyponatremia D. Overhydration, hypernatremia

ANS: B Aldosterone is a mineralocorticoid that increases the reabsorption of water and sodium in the kidney at the same time that it promotes excretion of potassium. Any drug or condition that disrupts aldosterone secretion or release increases the client's risk for excessive water loss and potassium reabsorption.

Which condition would trigger the release of antidiuretic hormone (ADH)? A. Plasma osmolarity decreased secondary to overhydration. B. Plasma osmolarity increased secondary to dehydration. C. Plasma volume decreased secondary to hemorrhage. D. Plasma volume increased with edema formation.

ANS: B Antidiuretic hormone is triggered by a rising ECF osmolarity, especially hypernatremia.

Which assessment maneuvers should the nurse perform first when assessing the renal system at the same time as the abdomen? A. Abdominal percussion B. Abdominal auscultation C. Abdominal palpation D. Renal palpation

ANS: B Auscultation precedes percussion and palpation because the nurse needs to auscultate for abdominal bruits before palpation or percussion of the abdominal and renal components of a physical assessment.

The nurse is providing teaching by telephone to a patient who is scheduled for a pelvic examination and Pap test next week. The nurse instructs the patient that she should a. shower, but not take a tub bath, before the examination. b. not have sexual intercourse the day before the Pap test. c. avoid douching for at least 24 hours before the examination. d. schedule to have the Pap test just after her menstrual period.

ANS: C Because the results of a Pap test may be affected by douching, the patient should not douche before the examination. The exam may be scheduled without regard to the menstrual period. The patient may shower or bathe before the examination. Sexual intercourse does not affect the results of the examination or Pap test

Which finding from the nurse's physical assessment of a 42-year-old male patient should be reported to the health care provider? a. One testis hangs lower than the other. b. Genital hair distribution is diamond shaped. c. Clear discharge is present at the penile meatus. d. Inguinal lymph nodes are nonpalpable bilaterally.

ANS: C Clear penile discharge may be indicative of a sexually transmitted infection (STI). The other findings are normal and do not need to be reported

A dark-skinned patient has been admitted to the hospital with chronic heart failure. How would the nurse best assess this patient for cyanosis? a. Assess the skin color of the earlobes. b. Apply pressure to the palms of the hands. c. Check the lips and oral mucous membranes. d. Examine capillary refill time of the nail beds.

ANS: C Cyanosis in dark-skinned individuals is more easily seen in the mucous membranes. Earlobe color may change in light-skinned individuals, but this change in skin color is difficult to detect on darker skin. Application of pressure to the palms of the hands and nail bed assessment would check for adequate circulation but not for skin color

When giving home care instructions to a patient who has comminuted forearm fractures and a long-arm cast on the left arm, which information should the nurse include? a. Keep the left shoulder elevated on a pillow or cushion. b. Keep the hand immobile to prevent soft tissue swelling. c. Call the health care provider for increased swelling or numbness of the hand. d. Avoid nonsteroidal antiinflammatory drugs (NSAIDs) for 24 hours after the injury.

ANS: C Increased swelling or numbness may indicate increased pressure at the injury, and the health care provider should be notified immediately to avoid damage to nerves and other tissues. The patient should be encouraged to move the joints above and below the cast to avoid stiffness. There is no need to elevate the shoulder, although the forearm should be elevated to reduce swelling. NSAIDs are appropriate to treat pain after a fracture.

A 49-year-old man who has type 2 diabetes, high blood pressure, hyperlipidemia, and gastroesophageal reflux tells the nurse that he has had recent difficulty in achieving an erection. Which of the following drugs from his current medications list may cause erectile dysfunction (ED)? a. Ranitidine (Zantac) b. Atorvastatin (Lipitor) c. Propranolol (Inderal) d. Metformin (Glucophage)

ANS: C Some antihypertensives may cause erectile dysfunction, and the nurse should anticipate a change in antihypertensive therapy. The other medications will not affect erectile function

The nurse is caring for a patient who smokes 2 packs/day. To reduce the patient's risk of lung cancer, which action by the nurse is best? a. Teach the patient about the seven warning signs of cancer. b. Plan to monitor the patient's carcinoembryonic antigen (CEA) level. c. Discuss the risks associated with cigarettes during every patient encounter. d. Teach the patient about the use of annual chest x-rays for lung cancer screening.

ANS: C Teaching about the risks associated with cigarette smoking is recommended at every patient encounter because cigarette smoking is associated with multiple health problems. A tumor must be at least 0.5 cm large before it is detectable by current screening methods and may already have metastasized by that time. Oncofetal antigens such as CEA may be used to monitor therapy or detect tumor reoccurrence, but are not helpful in screening for cancer. The seven warning signs of cancer are actually associated with fairly advanced disease. DIF: Cognitive Level: Apply (application) REF: 255-256 TOP: Nursing Process: Implementation MSC: NCLEX: Health Promotion and Maintenance

A patient who arrives at the emergency department experiencing severe left knee pain is diagnosed with a patellar dislocation. The initial patient teaching by the nurse will focus on the need for a. a knee immobilizer. b. gentle knee flexion. c. monitored anesthesia care. d. physical activity restrictions.

ANS: C The first goal of collaborative management is realignment of the knee to its original anatomic position, which will require anesthesia or monitored anesthesia care (MAC), formerly called conscious sedation. Immobilization, gentle range-of-motion (ROM) exercises, and discussion about activity restrictions will be implemented after the knee is realigned.

A patient who is scheduled for a right breast biopsy asks the nurse the difference between a benign tumor and a malignant tumor. Which answer by the nurse is correct? a. "Benign tumors do not cause damage to other tissues." b. "Benign tumors are likely to recur in the same location." c. "Malignant tumors may spread to other tissues or organs." d. "Malignant cells reproduce more rapidly than normal cells."

ANS: C The major difference between benign and malignant tumors is that malignant tumors invade adjacent tissues and spread to distant tissues and benign tumors never metastasize. The other statements are inaccurate. Both types of tumors may cause damage to adjacent tissues. Malignant cells do not reproduce more rapidly than normal cells. Benign tumors do not usually recur. DIF: Cognitive Level: Understand (comprehension) REF: 253 TOP: Nursing Process: Implementation MSC: NCLEX: Physiological Integrity

The client has an elevated blood urea nitrogen (BUN) level and an increased ratio of blood urea nitrogen to creatinine. What is the nurse's interpretation of these laboratory results? A. The client probably has a urinary tract infection. B. The client may be overhydrated. C. The kidney may be hypoperfused. D. The kidney may be damaged.

ANS: C When dehydration or renal hypoperfusion exist, the BUN level rises more rapidly than the serum creatinine level, causing the ratio to be increased, even when no renal dysfunction is present.

A patient with a large stomach tumor that is attached to the liver is scheduled to have a debulking procedure. Which information should the nurse teach the patient about the outcome of this procedure? a. Pain will be relieved by cutting sensory nerves in the stomach. b. Relief of pressure in the stomach will promote better nutrition. c. Tumor growth will be controlled by the removal of malignant tissue. d. Tumor size will decrease and this will improve the effects of other therapy.

ANS: D A debulking surgery reduces the size of the tumor and makes radiation and chemotherapy more effective. Debulking surgeries do not control tumor growth. The tumor is debulked because it is attached to the liver, a vital organ (not to relieve pressure on the stomach). Debulking does not sever the sensory nerves, although pain may be lessened by the reduction in pressure on the abdominal organs.

In developing a care plan for a patient with an open reduction and internal fixation (ORIF) of an open, displaced fracture of the tibia, the priority nursing diagnosis is a. activity intolerance related to deconditioning. b. risk for constipation related to prolonged bed rest. c. risk for impaired skin integrity related to immobility. d. risk for infection related to disruption of skin integrity.

ANS: D A patient having an ORIF is at risk for problems such as wound infection and osteomyelitis. After an ORIF, patients typically are mobilized starting the first postoperative day, so problems caused by immobility are not as likely.

Which action will the nurse include in the plan of care for a patient who has had a total right knee arthroplasty? a. Avoid extension of the right knee beyond 120 degrees. b. Use a compression bandage to keep the right knee flexed. c. Teach about the need to avoid weight bearing for 4 weeks. d. Start progressive knee exercises to obtain 90-degree flexion.

ANS: D After knee arthroplasty, active or passive flexion exercises are used to obtain a 90-degree flexion of the knee. The goal for extension of the knee will be 180 degrees. A compression bandage is used to hold the knee in an extended position after surgery. Full weight bearing is expected before discharge.

10. A patient who recently has been experiencing frequent heartburn is seen in the clinic. The nurse will anticipate teaching the patient about a. barium swallow. b. radionuclide tests. c. endoscopy procedures. d. proton pump inhibitors.

ANS: D Because diagnostic testing for heartburn that is probably caused by gastroesophageal reflux disease (GERD) is expensive and uncomfortable, proton pump inhibitors are frequently used for a short period as the first step in the diagnosis of GERD. The other tests may be used but are not usually the first step in diagnosis.

A 44-year-old patient in the sexually transmitted infection clinic has a positive Venereal Disease Research Laboratory (VDRL) test, but no chancre is visible on assessment. The nurse will plan to send specimens for a. gram stain. b. cytologic studies. c. rapid plasma reagin (RPR) agglutination. d. fluorescent treponemal antibody absorption (FTA-Abs).

ANS: D Because false positives are common with VDRL and RPR testing, FTA-Abs testing is recommended to confirm a diagnosis of syphilis. Gram staining is used for other sexually transmitted infections (STIs) such as gonorrhea and Chlamydia and cytologic studies are used to detect abnormal cells (such as neoplastic cells).

A patient has been assigned the nursing diagnosis of imbalanced nutrition: less than body requirements related to painful oral ulcers. Which nursing action will be most effective in improving oral intake? a. Offer the patient frequent small snacks between meals. b. Assist the patient to choose favorite foods from the menu. c. Provide teaching about the importance of nutritional intake. d. Apply the ordered anesthetic gel to oral lesions before meals.

ANS: D Because the etiology of the patient's poor nutrition is the painful oral ulcers, the best intervention is to apply anesthetic gel to the lesions before the patient eats. The other actions might be helpful for other patients with impaired nutrition, but would not be as helpful for this patient.

40. All of the following orders are received for a patient who has vomited 1500 mL of bright red blood. Which order will the nurse implement first? a. Insert a nasogastric (NG) tube and connect to suction. b. Administer intravenous (IV) famotidine (Pepcid) 40 mg. c. Draw blood for typing and crossmatching. d. Infuse 1000 mL of lactated Ringer's solution.

ANS: D Because the patient has vomited a large amount of blood, correction of hypovolemia and prevention of hypovolemic shock are the priorities. The other actions also are important to implement quickly but are not the highest priorities.

Which action will the nurse take in order to evaluate the effectiveness of Buck's traction for a patient who has an intracapsular fracture of the left femur? a. Assess for hip contractures. b. Monitor for hip dislocation. c. Check the peripheral pulses. d. Ask about left hip pain level.

ANS: D Buck's traction keeps the leg immobilized and reduces painful muscle spasm. Hip contractures and dislocation are unlikely to occur in this situation. The peripheral pulses will be assessed, but this does not help in evaluating the effectiveness of Buck's traction.

The nurse explains to a 37-year-old patient being prepared for colposcopy with a cervical biopsy that the procedure a. involves dilation of the cervix and biopsy of the tissue lining the uterus. b. will take place in a same-day surgery center so that local anesthesia can be used. c. requires that the patient have nothing to eat or drink for 6 hours before the procedure. d. is similar to a speculum examination of the cervix and should result in little discomfort.

ANS: D Colposcopy involves visualization of the cervix with a binocular microscope and is similar to a speculum examination. Anesthesia is not required and fasting is not necessary. A cervical biopsy may cause a minimal amount of pain

12. Which information will the nurse include when teaching a patient with newly diagnosed gastroesophageal reflux disease (GERD)? a. "Peppermint tea may be helpful in reducing your symptoms." b. "You should avoid eating between meals to reduce acid secretion." c. "Vigorous physical activities may increase the incidence of reflux." d. "It will be helpful to keep the head of your bed elevated on blocks."

ANS: D Elevating the head of the bed will reduce the incidence of reflux while the patient is sleeping. Peppermint will lower LES pressure and increase the chance for reflux. Small, frequent meals are recommended to avoid abdominal distention. There is no need to make changes in physical activities because of GERD.

9. After the nurse teaches a patient with gastroesophageal reflux disease (GERD) about recommended dietary modifications, which diet choice for a snack 2 hours before bedtime indicates that the teaching has been effective? a. Chocolate pudding b. Glass of low-fat milk c. Peanut butter sandwich d. Cherry gelatin and fruit

ANS: D Gelatin and fruit are low fat and will not decrease lower esophageal sphincter (LES) pressure. Foods like chocolate are avoided because they lower LES pressure. Milk products increase gastric acid secretion. High-fat foods such as peanut butter decrease both gastric emptying and LES pressure.

39. A patient who is vomiting bright red blood is admitted to the emergency department. Which assessment should the nurse perform first? a. Checking the level of consciousness b. Measuring the quantity of any emesis c. Auscultating the chest for breath sounds d. Taking the blood pressure (BP) and pulse

ANS: D The nurse is concerned about blood loss and possible hypovolemic shock in a patient with acute gastrointestinal (GI) bleeding; BP and pulse are the best indicators of these complications. The other information also is important to obtain, but BP and pulse rate are the best indicators for hypoperfusion.

A patient with a comminuted fracture of the right femur has Buck's traction in place while waiting for surgery. To assess for pressure areas on the patient's back and sacral area and to provide skin care, the nurse should a. loosen the traction and have the patient turn onto the unaffected side. b. place a pillow between the patient's legs and turn gently to each side. c. turn the patient partially to each side with the assistance of another nurse. d. have the patient lift the buttocks by bending and pushing with the left leg.

ANS: D The patient can lift the buttocks off the bed by using the left leg without changing the right-leg alignment. Turning the patient will tend to move the leg out of alignment. Disconnecting the traction will interrupt the weight needed to immobilize and align the fracture.

The second day after admission with a fractured pelvis, a patient develops acute onset confusion. Which action should the nurse take first? a. Take the blood pressure. b. Assess patient orientation. c. Check pupil reaction to light. d. Assess the oxygen saturation.

ANS: D The patient's history and clinical manifestations suggest a fat embolus. The most important assessment is oxygenation. The other actions also are appropriate but will be done after the nurse assesses gas exchange.

15. A patient who is nauseated and vomiting up blood-streaked fluid is admitted to the hospital with acute gastritis. To determine possible risk factors for gastritis, the nurse will ask the patient about a. the amount of fat in the diet. b. history of recent weight gain or loss. c. any family history of gastric problems. d. use of nonsteroidal anti-inflammatory drugs (NSAIDs).

ANS: D Use of an NSAID is associated with damage to the gastric mucosa, which can result in acute gastritis. Family history, recent weight gain or loss, and fatty foods are not risk factors for acute gastritis.

Which action should the nurse take when caring for a patient who is receiving chemotherapy and complains of problems with concentration? a. Teach the patient to rest the brain by avoiding new activities. b. Teach that "chemo-brain" is a short-term effect of chemotherapy. c. Report patient symptoms immediately to the health care provider. d. Suggest use of a daily planner and encourage adequate rest and sleep.

ANS: D Use of tools to enhance memory and concentration such as a daily planner and adequate rest are helpful for patients who develop "chemo-brain" while receiving chemotherapy. Patients should be encouraged to exercise the brain through new activities. Chemo-brain may be short- or long-term. There is no urgent need to report common chemotherapy side effects to the provider.

The client is scheduled to have a renogram (kidney scan). She is concerned about discomfort during the procedure. What is the nurse's best response? A. "Before the test you will be given a sedative to reduce any pain." B. "A local anesthetic agent will be used, so you might feel a little pressure but no pain." C. "Although this test is very sensitive, there is no more discomfort than you would have with an ordinary x-ray." D. "The only pain associated with this procedure is a small needle stick when you are given the radioisotope

ANS: D The test involves an intravenous injection of the radioisotope and the subsequent recording of the emission by a scintillator.

33. A patient hospitalized for IV corticosteroid therapy to treat polymyositis has joint pain, an erythematosus facial rash with eyelid edema, and a weak, hoarse voice. The priority nursing diagnosis for the patient is a. risk for aspiration related to dysphagia. b. acute pain related to inflammation. c. risk for impaired skin integrity related to scratching. d. disturbed visual perception related to eyelid swelling.

Answer: A Rationale: The patient's vocal weakness and hoarseness indicate weakness of the pharyngeal muscles and a high risk for aspiration. The other nursing diagnoses are also appropriate but are not as high a priority as the maintenance of the patient's airway. Cognitive Level: Application Text Reference: p. 1725 Nursing Process: Diagnosis NCLEX: Physiological Integrity

MULTIPLE RESPONSE 1. During assessment of the patient with fibromyalgia syndrome (FMS), the nurse would expect the patient to report (Select all that apply.) a. sleep disturbances. b. multiple tender points. c. urinary frequency and urgency. d. cardiac palpitations and dizziness. e. multijoint pain with inflammation and swelling. f. widespread bilateral, burning musculoskeletal pain.

Answer: A, B, C, F Rationale: These symptoms are commonly described by patients with FMS. Cardiac involvement and joint inflammation are not typical of FMS. Cognitive Level: Comprehension Text Reference: p. 1727 Nursing Process: Assessment NCLEX: Physiological Integrity

29. Following instruction for a patient with newly diagnosed systemic lupus erythematosus (SLE), the nurse determines that teaching about the disease has been effective when the patient says, a. "I should expect to have a low fever all the time with this disease." b. "I need to restrict my exposure to sunlight to prevent an acute onset of symptoms." c. "I should try to ignore my symptoms as much as possible and have a positive outlook." d. "I can expect a temporary improvement in my symptoms if I become pregnant."

Answer: B Rationale: Sun exposure is associated with SLE exacerbation, and patients should use sunscreen with an SPF of at least 15 and stay out of the sun between 11:00 AM and 3:00 PM. Low-grade fever may occur with an exacerbation but should not be expected all the time. A positive attitude may decrease the incidence of SLE exacerbations, but patients are taught to self-monitor for symptoms that might indicate changes in the disease process. Symptoms may worsen during pregnancy and especially during the postpartum period. Cognitive Level: Application Text Reference: pp. 1717, 1720 Nursing Process: Evaluation NCLEX: Physiological Integrity

1. A 60-year-old patient has osteoarthritis (OA) of the left knee. A finding that the nurse would expect to be present on examination of the patient's knee is a. Heberden's nodules. b. redness and swelling of the knee joint. c. pain upon joint movement. d. stiffness that increases with movement.

Answer: C Rationale: Initial symptoms of OA include pain with joint movement. Heberden's nodules occur on the fingers. Redness of the joint is more strongly associated with rheumatoid arthritis (RA), and stiffness in OA is worse right after the patient rests and decreases with joint movement. Cognitive Level: Comprehension Text Reference: p. 1694 Nursing Process: Assessment NCLEX: Physiological Integrity

The patient is incontinent, and a condom catheter is placed. The nurse should take which action? A) Secure the condom with adhesive tape B) Change the condom every 48 hours C) Assess the patient for skin irritation D) Use sterile technique for placement

Assess the patient for skin irritation

d

Association between HLA antigens and disease is most commonly found in what disease condition? A. malignancies B. infectious disease C. neurologic diseases D. autoimmune disorders

A patient who is given a bisacodyl (Dulcolax) suppository asks the nurse how long it will take to work. The nurse replies that the patient will probably need to use the bedpan or commode within which time frame after administration? A. 2-5 minutes B. 15-60 minutes C. 2-4 hours D. 6-8 hours

B Bisacodyl suppositories usually are effective within 15 to 60 minutes of administration, so the nurse should plan accordingly to assist the patient to use the bedpan or commode.

While completing an admission history for a 73-year-old man with osteoarthritis admitted for knee arthroplasty, the nurse asks about the patient's perception of the reason for admission. The nurse expects the patient to relate which response to this question? A Recent knee trauma B Debilitating joint pain C Repeated knee infections D Onset of "frozen" knee joint

B Debilitating joint pain The most common reason for knee arthroplasty is debilitating joint pain despite attempts to manage it with exercise and drug therapy. Recent knee trauma, repeated knee infections, and onset of "frozen" knee joint are not primary indicators for a knee arthroplasty.

The nurse would recognize which of the following patients as likely to have the poorest prognosis? A) A 60-year-old diagnosed with nodular ulcerative basal cell carcinoma B) A 59-year-old man who is being treated for stage IV malignant melanoma C) A 70-year-old woman who has been diagnosed with late squamous cell carcinoma D) A 51-year-old woman whose biopsy has revealed superficial squamous cell carcinoma

B) A 59-year-old man who is being treated for stage IV malignant melanoma Late detection of malignant melanoma is associated with a poor outcome. Basal cell carcinomas often have very effective treatment success rates. Although late SCC has worse outcomes than superficial SCC, these are both exceeded in mortality by late-stage malignant melanoma.

The nurse identifies that a patient with a diagnosis of which of the following disorders is most at risk for spreading the disease? A) tinea pedis B) impetigo on the face C) candidiasis of the nails D) psoriasis on the palms and soles

B) impetigo on the face

13. A patient receives a daily injection of 70/30 NPH/regular insulin premix at 7:00 AM. The nurse expects that a hypoglycemic reaction is most likely to occur between a. 8:00 and 10:00 AM. b. 4:00 and 6:00 PM. c. 7:00 and 9:00 PM. d. 10:00 PM and 12:00 AM.

B. 4:00 and 6:00 PM. Rationale: The greatest insulin effect with this combination occurs mid afternoon. The patient is not at a high risk at the other listed times, although hypoglycemia may occur.

The nurse is caring for a 40-year-old man who has begun taking levothyroxine (Synthroid) for recently diagnosed hypothyroidism. What information reported by the patient is most important for the nurse to further assess? A. Weight gain or weight loss B. Chest pain and palpitations C. Muscle weakness and fatigue D. Decreased appetite and constipation

B. Chest pain and palpitations Levothyroxine (Synthroid) is used to treat hypothyroidism. Any chest pain or heart palpitations or heart rate greater than 100 beats/minute experienced by a patient starting thyroid replacement should be reported immediately, and an electrocardiogram (ECG) and serum cardiac enzyme tests should be performed.

The nurse is planning health promotion teaching for a 45-year-old patient with asthma, low back pain from herniated lumbar disc, and schizophrenia. What does the nurse determine would be the best exercise to include in an individualized exercise plan for the patient? A. Yoga B. Walking C. Calisthenics D. Weight lifting

B. Walking The patient would benefit from an aerobic exercise that takes into account the patient's health status and fits the patient's lifestyle. The best exercise is walking, which builds strength in the back and leg muscles without putting undue pressure or strain on the spine. Yoga, calisthenics, and weight lifting would all put pressure on or strain the spine.

A common site for the lesions associated with atopic dermatitis is the A) buttocks B) temporal area C) antecubital space D) plantar surface of the feet

C) antecubital space

The nurse is caring for patients in a primary care clinic. Which individual is most at risk to develop osteomyelitis caused by Staphylococcus aureus? A. 22-year-old female with gonorrhea who is an IV drug user B. 48-year-old male with muscular dystrophy and acute bronchitis C. 32-year-old male with type 1 diabetes mellitus and a stage IV pressure ulcer D. 68-year-old female with hypertension who had a knee arthroplasty 3 years ago

C. 32-year-old male with type 1 diabetes mellitus and a stage IV pressure ulcer Osteomyelitis caused by Staphylococcus aureus is usually associated with a pressure ulcer or vascular insufficiency related to diabetes mellitus. Osteomyelitis caused by Staphylococcus epidermidis is usually associated with indwelling prosthetic devices such as joint replacements. Osteomyelitis caused by Neisseria gonorrhoeae is usually associated with gonorrhea. Osteomyelitis caused by Pseudomonas is usually associated with IV drug use. Muscular dystrophy is not associated with osteomyelitis.

The patient with breast cancer is having teletherapy radiation treatments after her surgery. What should the nurse teach the patient about the care of her skin? A. Use Dial soap to feel clean and fresh. B. Scented lotion can be used on the area. C. Avoid heat and cold to the treatment area. D. Wear the new bra to comfort and support the area.

C. Avoid heat and cold to the treatment area. Avoiding heat and cold in the treatment area will protect it. Only mild soap and unscented, nonmedicated lotions may be used to prevent skin damage. The patient will want to avoid wearing tight-fitting clothing such as a bra over the treatment field and will want to expose the area to air as often as possible.

A 33-year-old patient has recently been diagnosed with stage II cervical cancer. What should the nurse understand about the patient's cancer? A. It is in situ. B. It has metastasized. C. It has spread locally. D. It has spread extensively.

C. It has spread locally. Stage II cancer is associated with limited local spread. Stage 0 denotes cancer in situ; stage I denotes tumor limited to the tissue of origin with localized tumor growth. Stage III denotes extensive local and regional spread. Stage IV denotes metastasis.

What is the priority action for the nurse to take if the patient with type 2 diabetes complains of blurred vision and irritability? a) Call the physician b) Administer insulin as ordered c) Check the patient's blood glucose level d) Assess for other neurologic symptoms.

C. check the patient's blood glucose levels

6. During a clinic visit 3 months following a diagnosis of type 2 diabetes, the patient reports following a reduced-calorie diet. The patient has not lost any weight and did not bring the glucose-monitoring record. The nurse will plan to obtain a(n) a. fasting blood glucose level. b. urine dipstick for glucose. c. glycosylated hemoglobin level. d. oral glucose tolerance test.

C. glycosylated hemoglobin level Rationale: The glycosylated hemoglobin (Hb A1C) test shows the overall control of glucose over 90 to 120 days. A fasting blood level indicates only the glucose level at one time. Urine glucose testing is not an accurate reflection of blood glucose level and does not reflect the glucose over a prolonged time. Oral glucose tolerance testing is done to diagnose diabetes, but is not used for monitoring glucose control once diabetes has been diagnosed.

6. A nurse is caring for a client who is suspected to have a UTI. The provider prescribes a urine specimen. Which of the following findings should confirm to the nurse that an upper UTI involving the kidneys is present? -Bacteria -WBC -Casts -Ketones

Casts

Since removal of the patient's Foley catheter, the patient has voided 50 to 100 mL every 2 to 3 hours. Which action should the nurse take first? A) Check for bladder distention B) Encourage fluid intake C) Obtain an order to recatheterize the patient D) Document the amount of each voiding for 24 hours

Check for bladder distention

The patient with right upper quadrant abdominal pain has an abdominal ultrasound that reveals cholelithiasis. What should the nurse expect to do for this patient? Prevent all oral intake. Control abdominal pain. Provide enteral feedings. Avoid dietary cholesterol.

Control abdominal pain. Patients with cholelithiasis can have severe pain, so controlling pain is important until the problem can be treated. NPO status may be needed if the patient will have surgery but will not be used for all patients with cholelithiasis. Enteral feedings should not be needed, and avoiding dietary cholesterol is not used to treat cholelithiasis.

What is the main underlying risk factor for metabolic syndrome? a. Age b. Heart disease c. Insulin resistance d. High cholesterol levels

Correct answer: c Rationale: Insulin resistance is the main underlying risk factor for metabolic syndrome. Aging is associated with metabolic syndrome. High cholesterol, hypertension, and increased clotting risk are characteristics of metabolic syndrome.

Priority Decision: During care of the severely obese patient, what is most important for the nurse to do? a. Avoid reference to the patient's weight to avoid embarrassing the patient. b. Emphasize to the patient how important it is to lose weight to maintain health. c. Plan for necessary modifications in equipment and nursing techniques before initiating care. d. Recognize that a full assessment of each body system might not be possible because of numerous layers of skinfolds.

Correct answer: c Rationale: Special considerations are needed for the care of the severely obese patient because most hospital units are not prepared with beds, chairs, BP cuffs, and other equipment that will need to be used with the very obese patient. Consideration of all aspects of care should be made before implementing care for the patient, including extra time and perhaps assistance for positioning, physical assessment, and transferring the patient.

A severely obese patient has undergone Roux-en-Y gastric bypass surgery. In planning postoperative care, the nurse anticipates that the patient a. may have severe diarrhea early in the postoperative period. b. will not be allowed to ambulate for 1 to 2 days postoperatively. c. will require nasogastric suction until the incision heals. d. may have only liquids orally, and in very limited amounts, during the postoperative period.

Correct answer: d Rationale: During the immediate postoperative period, water and sugar-free clear liquids are given (30 mL every 2 hours while the patient is awake).

Normally, which hormones and peptides affect appetite (select all that apply)? a. Leptin b. Insulin c. Ghrelin d. Peptide YY e. Neuropeptide Y f. Cholecystokinin

Correct answers: a, b, c, d, e, f Rationale: Normally ghrelin and neuropeptide Y stimulate appetite. Leptin suppresses appetite and hunger. Insulin decreases appetite. Peptide YY and cholecystokinin inhibit appetite by slowing gastric emptying and sending satiety signals to the hypothalamus.

What characteristics describe adjustable gastric banding (select all that apply)? a. 85% of the stomach is removed. b. Stomach restriction can be reversed. c. Eliminates hormones that stimulate hunger. d. Malabsorption of fat-soluble vitamins occurs. e. Inflatable band allows for modification of gastric stoma size. f. Stomach with a gastric pouch surgically anastomosed to the jejunum.

Correct answers: b, e Rationale: The adjustable gastric banding procedure is reversible and allows a change in gastric stoma size by inflation or deflation of the band around the funds of the stomach. The vertical sleeve gastrectomy removes 85% of the stomach and eliminates the hormones produced in the stomach that stimulate hunger. The biliopancreatic diversion is a maladaptive surgery that prevents absorption of nutrients, including fat-soluble vitamins. The Roux-en-Y gastric bypass reduces the stomach size with a gastric pouch anastomosed to the small intestine, so it is both restrictive and malabsorptive.

A 61-year-old female patient admitted with pneumonia has a total serum calcium level of 13.3 mg/dL (3.3 mmol/L). The nurse will anticipate the need to teach the patient about testing for _____ levels. a. calcitonin b. catecholamine c. thyroid hormone d. parathyroid hormone

D

An 18-year-old male patient with a small stature is scheduled for a growth hormone stimulation test. In preparation for the test, the nurse will obtain a. ice in a basin. b. glargine insulin. c. a cardiac monitor. d. 50% dextrose solution.

D

When teaching the patient about the diet for diverticular disease, which foods should the nurse recommend? A. White bread, cheese, and green beans B. Fresh tomatoes, pears, and corn flakes C. Oranges, baked potatoes, and raw carrots D. Dried beans, All Bran (100%) cereal, and raspberries

D A high fiber diet is recommended for diverticular disease. Dried beans, All Bran (100%) cereal, and raspberries all have higher amounts of fiber than white bread, cheese, green beans, fresh tomatoes, pears, corn flakes, oranges, baked potatoes, and raw carrots.

The nurse is preparing to administer a dose of bisacodyl (Dulcolax). In explaining the medication to the patient, the nurse would explain that it acts in what way? A. Increases bulk in the stool B. Lubricates the intestinal tract to soften feces C. Increases fluid retention in the intestinal tract D. Increases peristalsis by stimulating nerves in the colon wall

D Bisacodyl is a stimulant laxative that aids in producing a bowel movement by irritating the colon wall and stimulating enteric nerves. It is available in oral and suppository forms. Fiber and bulk forming drugs increase bulk in the stool; water and stool softeners soften feces, and saline and osmotic solutions cause fluid retention in the intestinal tract.

What should the nurse instruct the patient to do to best enhance the effectiveness of a daily dose of docusate sodium (Colace)? A. Take a dose of mineral oil at the same time. B. Add extra salt to food on at least one meal tray. C. Ensure dietary intake of 10 g of fiber each day. D. Take each dose with a full glass of water or other liquid.

D Docusate lowers the surface tension of stool, permitting water and fats to penetrate and soften the stool for easier passage. The patient should take the dose with a full glass of water and should increase overall fluid intake, if able, to enhance effectiveness of the medication. Dietary fiber intake should be a minimum of 20 g daily to prevent constipation. Mineral oil and extra salt are not recommended.

The nurse is completing a neurovascular assessment on the patient with a tibial fracture and a cast. The feet are pulseless, pale, and cool. The patient says they are numb. What should the nurse suspect is occurring? A Paresthesia B Pitting edema C Poor venous return D Compartment syndrome

D Compartment syndrome The nurse should suspect compartment syndrome with one or more of the following six Ps: paresthesia, pallor, pulselessness, pain distal to the injury and unrelieved with opioids, pressure increases in the compartment, and paralysis. Although paresthesia and poor venous return are evident, these are just some of the manifestations of compartment syndrome.

The nurse is completing discharge teaching with an 80-year-old male patient who underwent right total hip arthroplasty. The nurse identifies a need for further instruction if the patient states the need to A avoid crossing his legs. B use a toilet elevator on toilet seat. C notify future caregivers about the prosthesis. D maintain hip in adduction and internal rotation.

D maintain hip in adduction and internal rotation. The patient should not force hip into adduction or force hip into internal rotation as these movements could displace the hip replacement. Avoiding crossing the legs, using a toilet elevator on a toilet seat, and notifying future caregivers about the prosthesis indicate understanding of discharge teaching.

The nurse should monitor for increases in which laboratory value for the patient as a result of being treated with dexamethasone (Decadron)? A. Sodium B. Calcium C. Potassium D. Blood glucose

D. Blood glucose Hyperglycemia or increased blood glucose level is an adverse effect of corticosteroid therapy. Sodium, calcium, and potassium levels are not directly affected by dexamethasone.

8. Antiretroviral therapy (ART) is being considered for an HIV-infected patient who has a CD4+ cell count of 400/µl. Which factor is most important to consider when determining whether ART will be started for this patient? a. Patient social support system b. HIV genotype and phenotype c. Potential medication side effects d. Patient ability to comply with ART schedule

D. Patient ability to comply with ART schedule Drug resistance develops quickly unless the patient takes ART medications on a stringent schedule, and this endangers both the patient and the community. The other information is also important to consider, but patients who are unable to manage and follow a complex drug treatment regimen should not be considered for ART.

37. The nurse teaches the diabetic patient who rides a bicycle to work every day to administer morning insulin into the a. thigh. b. buttock. c. arm. d. abdomen.

D. abdomen Rationale: Patients should be taught not to administer insulin into a site that will be exercised because exercise will increase the rate of absorption. The thigh, buttock, and arm are all exercised by riding a bicycle.

prevent the recurrence of renal calculi, the nurse teaches the patient to a. use a filter to strain all urine. b. avoid dietary sources of calcium. c. drink diuretic fluids such as coffee. d. have 2000 to 3000 mL of fluid a day.

D. have 2000 to 3000 mL of fluid a day A fluid intake of 2000 to 3000 mL daily is recommended to help flush out minerals before stones can form. Avoidance of calcium is not usually recommended for patients with renal calculi. Coffee tends to increase stone recurrence. There is no need for a patient to strain all urine routinely after a stone has passed, and this will not prevent stones.

2. A patient screened for diabetes at a clinic has a fasting plasma glucose level of 120 mg/dl (6.7 mmol/L). The nurse will plan to teach the patient about a. use of low doses of regular insulin. b. self-monitoring of blood glucose. c. oral hypoglycemic medications. d. maintenance of a healthy weight.

D. maintenance of a healthy weight Rationale: The patient's impaired fasting glucose indicates prediabetes and the patient should be counseled about lifestyle changes to prevent the development of type 2 diabetes.

The health care provider orders lactulose for a patient with hepatic encephalopathy. The nurse will monitor for effectiveness of this medication for this patient by assessing what? Relief of constipation Relief of abdominal pain Decreased liver enzymes Decreased ammonia levels

Decreased ammonia levels Hepatic encephalopathy is a complication of liver disease and is associated with elevated serum ammonia levels. Lactulose traps ammonia in the intestinal tract. Its laxative effect then expels the ammonia from the colon, resulting in decreased serum ammonia levels and correction of hepatic encephalopathy.

19. A nurse is collecting a 24-hr creatinine clearance. During the collection, the client accidentally discards a specimen. Which of the following is an appropriate action by the nurse? -Continue the collection, noting the loss on the lab slip -Add 1 hr to the collection time -Discard the previously collected urine and start the collection again -Discontinue the collection and draw a serum creatinine

Discard the previously collected urine and start the collection again

D

During the assessment of a patient with acute abdominal pain, the nurse should: A. perform deep palpation before auscultation B. obtain pulse rate and blood pressure to determine hypovolemic changes C. auscultate bowel sounds because hyperactive bowel sounds suggest paralytic ileus D. measure body temperature because an elevated temp may indicate an inflammatory or infectious process

water deprivation test

Easy and reliable diag test for DI. Degydration is induced by withholding fluids. Urine output is measured and tested hourly

C

Estrogen production by the mature ovarian follicle causes: A. decreased secretion of FSH and LH B. increased production of GnRH and FSH C. release of GnRH and increased secretion of LH D. decreased release of FSH and decreased progesterone production

A patient who has hepatitis B surface antigen (HBsAg) in the serum is being discharged with pain medication after knee surgery. Which medication order should the nurse question because it is most likely to cause hepatic complications? Tramadol (Ultram) Hydromorphone (Dilaudid) Oxycodone with aspirin (Percodan) Hydrocodone with acetaminophen (Vicodin)

Hydrocodone with acetaminophen (Vicodin) The analgesic with acetaminophen should be questioned because this patient is a chronic carrier of hepatitis B and is likely to have impaired liver function. Acetaminophen is not suitable for this patient because it is converted to a toxic metabolite in the liver after absorption, increasing the risk of hepatocellular damage.

8. A nurse is reinforcing teaching to a client who is scheduled for a vasectomy about the procedure. Which of the following client statements indicates an understanding of the procedure? -I should avoid having sex for at least 2 weeks after surgery -I will no longer be capable of producing sperm -If I reverse this surgery, I will be as fertile as before -I need to have 2 follow-up negative sperm counts

I need to have 2 follow-up negative sperm counts

d

If a person is heterozygous for a given gene, it means that the person: A. is a carrier for a genetic disorder B. is affected by the genetic disorder C. has two identical allels for the gene D.has two different allels for the gene

b, e

In a person having an acute rejection of a transplanted kidney, which of the following would help the nurse understand the course of events (select all that apply): A. a new transplant could be considered B. acute rejection can be treated with OKT3 C. acute rejection usually leads to chronic rejection D. corticosteroids are the most successful drug used to treat acute rejection E. Acute rejection is common after a transplant and can be treated with drug therapy

d

In a type 1 hypersensitivity reaction, the primary immunologic disorder appears to be: A. binding of IgG to an antigen on the cell surface B. deposit of antigen-antibody complexes in small vessels C. release of cytokines to interact with specific antigens D. release of chemical mediators from IgE-bound mast cells and basophils

B

In contrast to diverticulitis, the patient with diverticulosis: A. has rectal bleeding B. often has no symptoms C. has localized cramping pain D. frequently develops peritonitis

C

In planning care for the patient with Crohn's disease, the nurse recognizes that a major difference between ulcerative colitis and Crohn's disease is that Crohn's disease: A. frequently results in toxic megacolon B. causes fewer nutritional deficiencies than does ulcerative colitis C. often recurs after surgery, whereas ulcerative colitis is curable with a colectomy D. is manifested by rectal bleeding and anemia more frequently than is ulcerative colitis

When planning care for a patient with cirrhosis, the nurse will give highest priority to which nursing diagnosis? Impaired skin integrity related to edema, ascites, and pruritus Imbalanced nutrition: less than body requirements related to anorexia Excess fluid volume related to portal hypertension and hyperaldosteronism Ineffective breathing pattern related to pressure on diaphragm and reduced lung volume

Ineffective breathing pattern related to pressure on diaphragm and reduced lung volume Although all of these nursing diagnoses are appropriate and important in the care of a patient with cirrhosis, airway and breathing are always the highest priorities.

13. A nurse is caring for a client who is receiving peritoneal dialysis. Which of the following is a complication of this procedure? -Hyponatremia -Metabolic acidosis -Hypoglycemia -Infection

Infection

A patient with a Foley catheter carries the collection bag at waist level when ambulating. The nurse tells the patient that he or she is at risk for: (Select all that apply.) A) Infection. B) Retention. C) Stagnant urine. D) Reflux of urine.

Infection. Reflux of urine.

An older male patient states that he is having problems starting and stopping his stream of urine and he feels the urgency to void. The best way to assist this patient is to: A) Help him stand to void. B) Place a condom catheter. C) Have him practice Credé's method. D) Initiate Kegel exercises.

Initiate Kegel exercises.

The patient with sudden pain in the left upper quadrant radiating to the back and vomiting was diagnosed with acute pancreatitis. What intervention(s) should the nurse expect to include in the patient's plan of care? Immediately start enteral feeding to prevent malnutrition. Insert an NG and maintain NPO status to allow pancreas to rest. Initiate early prophylactic antibiotic therapy to prevent infection. Administer acetaminophen (Tylenol) every 4 hours for pain relief.

Insert an NG and maintain NPO status to allow pancreas to rest. Correct Initial treatment with acute pancreatitis will include an NG tube if there is vomiting and being NPO to decrease pancreatic enzyme stimulation and allow the pancreas to rest and heal. Fluid will be administered to treat or prevent shock. The pain will be treated with IV morphine because of the NPO status. Enteral feedings will only be used for the patient with severe acute pancreatitis in whom oral intake is not resumed. Antibiotic therapy is only needed with acute necrotizing pancreatitis and signs of infection.

The nurse is caring for a 55-year-old man patient with acute pancreatitis resulting from gallstones. Which clinical manifestation would the nurse expect the patient to exhibit? Hematochezia Left upper abdominal pain Ascites and peripheral edema Temperature over 102o F (38.9o C)

Left upper abdominal pain Abdominal pain (usually in the left upper quadrant) is the predominant manifestation of acute pancreatitis. Other manifestations of acute pancreatitis include nausea and vomiting, low-grade fever, leukocytosis, hypotension, tachycardia, and jaundice. Abdominal tenderness with muscle guarding is common. Bowel sounds may be decreased or absent. Ileus may occur and causes marked abdominal distention. Areas of cyanosis or greenish to yellow-brown discoloration of the abdominal wall may occur. Other areas of ecchymoses are the flanks (Grey Turner's spots or sign, a bluish flank discoloration) and the periumbilical area (Cullen's sign, a bluish periumbilical discoloration).

5. A nurse is caring for a young adult client who sustained massive damage to the bladder. An emergency cystectomy and ileal conduit are performed. After viewing the appliance for the first time, the client tells the nurse, "Well I guess my sex life is over now." The most therapeutic response from the nurse would be which of the following? -It will just take time to adjust -Your partner will understand -Let's take about why you feel that way -The doctors did what they had to save your life.

Let's talk about why you feel that way

To minimize the patient experiencing nocturia, the nurse would teach him or her to: A) Perform perineal hygiene after urinating. B) Set up a toileting schedule. C) Double void. D) Limit fluids before bedtime.

Limit fluids before bedtime

patient with type 2 diabetes and cirrhosis asks the nurse if it would be okay to take silymarin (milk thistle) to help minimize liver damage. The nurse responds based on what knowledge? Milk thistle may affect liver enzymes and thus alter drug metabolism. Milk thistle is generally safe in recommended doses for up to 10 years. There is unclear scientific evidence for the use of milk thistle in treating cirrhosis. Milk thistle may elevate the serum glucose levels and is thus contraindicated in diabetes.

Milk thistle may affect liver enzymes and thus alter drug metabolism. There is good scientific evidence that there is no real benefit from using milk thistle to protect the liver cells from toxic damage in the treatment of cirrhosis. Milk thistle does affect liver enzymes and thus could alter drug metabolism. Therefore patients will need to be monitored for drug interactions. It is noted to be safe for up to 6 years, not 10 years, and it may lower, not elevate, blood glucose levels.

The patient is to have an intravenous pyelogram (IVP). Which of the following apply to this procedure? (Select all that apply.) A) Note any allergies. B) Monitor intake and output. C) Provide for perineal hygiene. D) Assess vital signs. E) Encourage fluids after the procedure.

Note any allergies. Encourage fluids after the procedure.

c

One function of cell-mediated immunity is: A. formation of antibodies B. activation of the complement system C. surveillance for malignant cell changes D. opsonization of antigens to allow phagocytosis by neutrophils

9. A nurse is caring for a client who just had a TURP. Which of the following should the nurse remind the client to report to the provider? -Pink tinged urine -Painful urine -Stress incontinence -Retrograde ejactulation

Painful urination

1. A nurse is caring for a client who is to undergo a cystoscopy. When reinforcing teaching to the client on post-procedure expectation, which of the following should the nurse state? -"It will be necessary to keep the sutures clean." -"You will be placed in a dorsal recumbent position" -"Expect to be on bed rest for 24 hours" -"Pink-tinged urine and burning while urinating can be expected."

Pink tinged urine and burning while urinating can be expected

17. A nurse is caring for a client who has CKD. The nurse anticipates that the provider will prescribe a diet that has which of the following restrictions? -Protein -Fats -Carbohydrates -Fiber

Protein

20. A nurse is caring for a client who has a diagnosis of renal calculi. The client reports severe right flank pain and nausea. Which of the following is a priority nursing action? -Relieve pain -Push fluids -Monitor I&O -Strain urine

Relieve pain

C

Significant information about a patient's past health history related to the reproductive system should include: A. extent of sexual activity B. general satisfaction with sexuality C. previous STDs D. self image and relationship with others

d. The patient's symptoms are characteristic of graft-versus-host-disease (GVHD) in which transplanted cells mount an immune response to the host's tissue. GVHD is not a type I allergic response or an atopic reaction, and it differs from transplant rejection in that the graft rejects the host rather than the host rejecting the graft.

Ten days after receiving a bone marrow transplant, a patient has developed a skin rash on his palms and soles, jaundice, and diarrhea. What is the most likely etiology of these clinical manifestations? A. The patient is experiencing a type I allergic reaction. B. An atopic reaction is causing the patient's symptoms. C. The patient is experiencing rejection of the bone marrow. D. Cells in the transplanted bone marrow are rejecting the host tissue.

10. A nurse is reinforcing education about prostate health to a client. Which of the following is an appropriate statement for the nurse to make in regard to a PSA test? -You should fast for 8 hours prior to having a PSA specimen obtained -Yearly PSA screening should begin at age 40 in all men -Normal PSA values decrease as you get older -The PSA test should not be performed for 48 hours following a DRE

The PSA test should not be performed for 48 hours following a DRE

A

The appropriate collaborative therapy for the patient with acute diarrhea caused by a viral infection is to: A. increased fluid intake B. administer an antibiotic C. administer antimotility drugs D. quarantine the patient to prevent spread of the virus

The patient with a history of lung cancer and hepatitis C has developed liver failure and is considering liver transplantation. After the comprehensive evaluation, the nurse knows that which factor discovered may be a contraindication for liver transplantation? Has completed a college education Has been able to stop smoking cigarettes Has well-controlled type 1 diabetes mellitus The chest x-ray showed another lung cancer lesion.

The chest x-ray showed another lung cancer lesion. Correct Contraindications for liver transplant include severe extrahepatic disease, advanced hepatocellular carcinoma or other cancer, ongoing drug and/or alcohol abuse, and the inability to comprehend or comply with the rigorous post-transplant course.

d

The function of monocytes in immunity is related to their ability to: A. stimulate the production of T and B lymphocytes B. produce antibodies on exposure to foreign substances C. bind antigens and stimulate natural killer cell activation D. capture antigens by phagocytosis and present them to lymphocytes

a

The most common cause of secondary immunodeficiencies is: A. drugs B. stress C. malnutrition D. human immunodeficiency virus

d

The nurse advises a friend who asks him to administer his allergy shots that: A. it is illegal for nurses to administer injections outside of a medical setting B. he is qualified to do it if the friend has epinephrine in an injectible syringe provided with his extract C. avoiding the allergens is a much more effective way of controlling allergens, and allergy shots are not usually effective D. immunotherapy should only be administered in a setting where emergency equipment and drugs are available

A

The nurse determines that the goals of dietary teaching have been met when the patient with celiac disease selects from the menu: A. scrambled eggs and sausage B. buckwheat pancake and syrup C. oatmeal, skim milk, and OJ D. yogurt, strawberries and rye toast with butter

A

The nurse explains to the patient undergoing ostomy surgery that the procedure that maintain the most normal functioning of the bowel is: A. a sigmoid colostomy B. a transverse colostomy C. a descending colostomy D. an ascending colostomy

a

The nurse is altered to possible anaphylactic shock immediately after a patient has received intramuscular penicillin by the development of: A. edema and itching at the injection site B. sneezing and itching of the nose and eyes C. a wheal-and-flare reaction at the injection site D. chest tightness and production of thick sputum

A, C

The nurse performs a detailed assessment of the abdomen of a patient with a possible bowel obstruction, knowing that a manifestation of an obstruction in the large intestine is (select all that apply): A. a largely distended abdomen B. diarrhea that is loose or liquid C. persistent, colicky abdominal pain D. profuse vomiting that relieves abdominal pain

B

The nurse would increase the comfort of a patient with appendicitis by: A. having the patient lie prone B. flexing the patient's right knee C. sitting the patient upright in a chair D. turning the patient onto his left side

a

The reason newborns are protected for the first 6 months of life from bacterial infection is because of the maternal transmission of: A. IgG B. IgA C. IgM D. IgE

D

The screening criteria for assessing prostate cancer include a: A. baseline ultrasound of the prostate at age 40 B. baseline ultrasound of the prostate at age 50 C. yearly DRE for men age 30 and older D. yearly DRE for men age 50 and older

The patient with cirrhosis has an increased abdominal girth from ascites. The nurse should know that this fluid gathers in the abdomen for which reasons (select all that apply)? There is decreased colloid oncotic pressure from the liver's inability to synthesize albumin. Hyperaldosteronism related to damaged hepatocytes increases sodium and fluid retention. Portal hypertension pushes proteins from the blood vessels, causing leaking into the peritoneal cavity. Osmoreceptors in the hypothalamus stimulate thirst, which causes the stimulation to take in fluids orally. Overactivity of the enlarged spleen results in increased removal of blood cells from the circulation, which decreases the vascular pressure.

There is decreased colloid oncotic pressure from the liver's inability to synthesize albumin. Correct Hyperaldosteronism related to damaged hepatocytes increases sodium and fluid retention. Correct Portal hypertension pushes proteins from the blood vessels, causing leaking into the peritoneal cavity. Correct The ascites related to cirrhosis are caused by decreased colloid oncotic pressure from the lack of albumin from liver inability to synthesize it and the portal hypertension that shifts the protein from the blood vessels to the peritoneal cavity, and hyperaldosteronism which increases sodium and fluid retention. The intake of fluids orally and the removal of blood cells by the spleen do not directly contribute to ascites.

A, C

To evaluate the female patients breasts you would use the examination techniques of (select all that apply): A. palpation B. percussion C. inspection D. auscultation

The postoperative patient has difficulty voiding after surgery and is feeling "uncomfortable" in the lower abdomen. Which action should the nurse implement first? A) Encourage fluid intake B) Administer pain medication C) Catheterize the patient D) Turn on the bathroom faucet as he tries to void

Turn on the bathroom faucet as he tries to void

The nurse assesses that the patient has a full bladder, and the patient states that he or she is having difficulty voiding. The nurse would teach the patient to: A) Use the double-voiding technique. B) Perform Kegel exercises. C) Use Credé's method. D) Keep a voiding diary.

Use Credé's method.

When caring for a patient with a biliary obstruction, the nurse will anticipate administering which vitamin supplements (select all that apply)? Vitamin A Vitamin D Vitamin E Vitamin K Vitamin B

Vitamin A Correct Vitamin D Correct Vitamin E Correct Vitamin K Correct Biliary obstruction prevents bile from entering the small intestine and thus prevents the absorption of fat-soluble vitamins. Vitamins A, D, E, and K are all fat-soluble and thus would need to be supplemented in a patient with biliary obstruction.

a, c, d, e

When caring for a patient with a known latex allergy, the nurse would monitor the patient closely for a cross-sensitivity to which of the following foods (select all that apply)? A. Grapes B. Oranges C. Bananas D. Potatoes E. Tomatoes

d

Which of the following accurately describes rejection following transplantation? A. hyperacute rejection can be treated with OKT3 B. acute rejections can be treated with sirolimus or tacrolimus C. chronic rejection can be treated with tacrolimus or cyclosporine D. hyperacute rejection can usually be avoided is crossmatching is done before the transplantation

D

Which of the following should a patient be taught after a hemorrhoidectomy? A. take mineral oil prior to bedtime B eat a low fiber diet to rest the colon C. administer oil retention enema to empty the colon D. use prescribed pain medication before a bowel movement

12. A nurse is caring for a client with acute pyelonephritis. Which of the following is an appropriate response by the nurse regarding home care? -You should complete the entire cycle of antibiotic therapy -You should maintain complete bed rest until manifestations decrease -You should drink 1,000 mL of fluid per day -You should weigh yourself daily.

You should complete the entire cycle of antibiotic therapy

In planning care for a patient with metastatic liver cancer, the nurse should include interventions that a. focus primarily on symptomatic and comfort measures. b. reassure the patient that chemotherapy offers a good prognosis. c. promote the patient's confidence that surgical excision of the tumor will be successful. d. provide information necessary for the patient to make decisions regarding liver transplantation. (Lewis 1042)

a Rationale: Nursing intervention for a patient with liver cancer focuses on keeping the patient as comfortable as possible. The prognosis for patients with liver cancer is poor. The cancer grows rapidly, and death may occur within 4 to 7 months as a result of hepatic encephalopathy or massive blood loss from gastrointestinal (GI) bleeding.

Which drugs will be used to treat the patient with CKD for mineral and bone disorder (select all that apply)? a. Cinacalcet (Sensipar) b. Sevelamer (Renagel) c. IV glucose and insulin d. Calcium acetate (PhosLo) e. IV 10% calcium gluconate

a, b, d. Cinacalcet (Sensipar), a calcimimetic agent to control secondary hyperparathyroidism; sevelamer (Renagel), a noncalcium phosphate binder; and calcium acetate (PhosLo), a calcium-based phosphate binder are used to treat mineral and bone disorder in CKD. IV glucose and insulin and IV 10% calcium gluconate along with sodium polystyrene sulfonate (Kayexalate) are used to treat the hyperkalemia of CKD.

Number the following in the order of the phases of exchange in PD. Begin with 1 and end with 3. a. Drain b. Dwell c. Inflow

a. 3; b. 2; c. 1

When the nurse assesses the patient that has pancreatitis, what function may be altered related to the endocrine function of the pancreas? a. Blood glucose regulation b. Increased response to stress c. Fluid and electrolyte regulation d. Regulates metabolic rate of cells

a. Blood glucose regulation The endocrine functions of the pancreas are regulated by α cells that produce and secrete glucagon, β cells that produce and secrete insulin and amylin, delta cells that produce and secrete somatostatin, and F cells that secrete pancreatic polypeptide. Glucagon, insulin, and amylin, and somatostatin all affect blood glucose. Pancreatic polypeptide regulates appetite. Increased response to stress occurs from epinephrine secreted by the adrenal medulla. Fluid and electrolyte regulation occurs in response to several hormones (mineralocorticoids, antidiuretic hormone, parathyroid hormone, calcitonin) from several organs (adrenal cortex, posterior pituitary, parathyroid, thyroid). The metabolic rate of cells is regulated by triiodothyronine (T3) from the thyroid

The surgeon was unable to save a patient's parathyroid gland during a radical thyroidectomy. The nurse should consequently pay particular attention to which laboratory value? a. Calcium levels b. Potassium levels c. Blood glucose levels d. Sodium and chloride levels

a. Calcium levels The parathyroid gland plays a key role in maintaining calcium levels. Potassium, sodium, glucose, and chloride are not directly influenced by the loss of the parathyroid gland.

5. An African American woman with a history of breast cancer has panhypopituitarism from radiation therapy for primary pituitary tumors. Which medications should the nurse teach her about needing for the rest of her life (select all that apply)? a. Cortisol b. Vasopressin c. Sex hormones d. Levothyroxine (Synthyroid) e. Growth hormone (somatropin [Omnitrope]) f. Dopamine agonists (bromocriptine [Parlodel])

a. Cortisol b. Vasopressin d. Levothyroxine (Synthyroid) e. Growth hormone (somatropin [Omnitrope]) With panhypopituitarism, lifetime hormone replacement is needed for cortisol, vasopressin, thyroid, and GH. Sex hormones will not be replaced because of the patient's history of breast cancer. Dopamine agonists will not be used because they reduce secretion of GH, which has already been achieved with the radiation.

Priority Decision: A patient on a medical unit has a potassium level of 6.8 mEq/L. What is the priority action that the nurse should take? a. Place the patient on a cardiac monitor. b. Check the patient's blood pressure (BP). c. Instruct the patient to avoid high-potassium foods. d. Call the lab and request a redraw of the lab to verify results.

a. Dysrhythmias may occur with an elevated potassium level and are potentially lethal. Monitor the rhythm while contacting the physician or calling the rapid response team. Vital signs should be checked. Depending on the patient's history and cause of increased potassium, instruct the patient about dietary sources of potassium; however, this would not help at this point. The nurse may want to recheck the value but until then the heart rhythm needs to be monitored.

Metabolic acidosis occurs in the oliguric phase of AKI as a result of impairment of a. ammonia synthesis. b. excretion of sodium. c. excretion of bicarbonate. d. conservation of potassium.

a. Metabolic acidosis occurs in AKI because the kidneys cannot synthesize ammonia or excrete acid products of metabolism, resulting in an increased acid load. Sodium is lost in urine because the kidneys cannot conserve sodium. Impaired excretion of potassium results in hyperkalemia. Bicarbonate is normally generated and reabsorbed by the functioning kidney to maintain acidbase balance.

Which characteristics describe the use of RAI (select all that apply)? a. Often causes hypothyroidism over time b. Decreases release of thyroid hormones c. Blocks peripheral conversion of T4 to T3 d. Treatment of choice in nonpregnant adults e. Decreases thyroid secretion by damaging thyroid gland f. Often used with iodine to produce euthyroid before surgery

a. Often cause hypothyroidism over time d. Treatment of choice in non pregnant adults e. Decreases thyroid secretion by damaging thyroid gland RAI causes hypothyroidism over time by damaging thyroid tissue and is the treatment of choice for nonpregnant adults. Potassium iodide decreases the release of thyroidhormones and decreases the size of the thyroid gland preoperatively. Propylthiouracil (PTU) blocks peripheral conversion of T4 to T3 and may be used with iodine to produce a euthyroid state before surgery.

A patient has sought care because of a loss of 25 lb over the past 6 months, during which the patient claims to have made no significant dietary changes. What potential problem should the nurse assess the patient for? a. Thyroid disorders b. Diabetes insipidus c. Pituitary dysfunction d. Parathyroid dysfunction

a. Thyroid disorders Hyperthyroidism is associated with weight loss. Alterations in pituitary function, such as diabetes insipidus, and parathyroid dysfunction are not commonly associated with this phenomenon.

*6. A patient is admitted to the hospital with severe renal colic. The nurse's first priority in management of the patient is to* a. administer opioids as prescribed. b. obtain supplies for straining all urine c. encourage fluid intake of 3-4L/day d. keep the patient NPO in preparation for surgery

a. administer opioids as prescribed.

*4.One of the nruse's most important roles in relation to acute poststreptococcal golmerulonephritis is to* a. promote early diagnosis and treatment of sore throats and skin lesions b.encourage patients to obtain antibiotic therapy for upper respiratory tract infections c.teach patients with APSGN that long term prophylactic antibiotic therapy is necessary to prevent recurrence d.monitor patients for respiratory symptoms that indicate the disease is affecting the alveolar basement membrane

a. promote early diagnosis and treatment of sore throats and skin lesions

*9. In planning nursing interventions to increase bladder control in the patient with urinary incontinence, the nurse includes:* a. teaching the patient to use Kegel exercises b.clamping and releasing a catheter to increase bladder tone c.teaching the patient biofeedback mechanisms to suppress the urge to void d.counseling the patient concerning choices of incontinence containment devices

a. teaching the patient to use Kegel exercises

34. During discharge teaching for the patient with Addison's disease, which statement by the patient indicates that the nurse needs to do additional teaching? a. "I should always call the doctor if I develop vomiting or diarrhea." b. "If my weight goes down, my dosage of steroid is probably too high." c. "I should double or triple my steroid dose if I undergo rigorous physical exercise." d. "I need to carry an emergency kit with injectable hydrocortisone in case I can't take my medication by mouth."

b. "if my weight goes down, my dosage of steroid is probably too high" A weight reduction in the patient with Addison's disease may indicate a fluid loss and a dose of replacement therapy that is too low rather than too high. Because vomiting and diarrhea are early signs of crisis and because fluid and electrolytes must be replaced, patients should notify their health care provider if these symptoms occur. Patients with Addison's disease are taught to take two to three times their usual dose of steroids if they become ill, have teeth extracted, or engage in rigorous physical activity and should always have injectablehydrocortisone available if oral doses cannot be taken.

What is the primary way that a nurse will evaluate the patency of an AVF? a. Palpate for pulses distal to the graft site. b. Auscultate for the presence of a bruit at the site. c. Evaluate the color and temperature of the extremity. d. Assess for the presence of numbness and tingling distal to the site.

b. A patent arteriovenous fistula (AVF) creates turbulent blood flow that can be assessed by listening for a bruit or palpated for a thrill as the blood passes through the graft. Assessment of neurovascular status in the extremity distal to the graft site is important to determine that the graft does not impair circulation to the extremity but the neurovascular status does not indicate whether the graft is open.

17. What medication is used with thyrotoxicosis to block the effects of the sympathetic nervous stimulation of the thyroid hormones? a. Potassium iodide b. Atenolol (Tenormin) c. Propylthiouracil (PTU) d. Radioactive iodine (RAI)

b. Atenolol (Tenormin) The β-adrenergic blocker atenolol is used to block the sympathetic nervous system stimulation by thyroid hormones. Potassium iodide is used to prepare the patient for thyroidectomy or for treatment of thyrotoxic crisis to inhibit the synthesis of thyroid hormones. Antithyroid medications inhibit the synthesis of thyroid hormones. Radioactive iodine (RAI) therapy destroys thyroid tissue, which limits thyroid hormone secretion.

In which type of dialysis does the patient dialyze during sleep and leave the fluid in the abdomen during the day? a. Long nocturnal hemodialysis b. Automated peritoneal dialysis (APD) c. Continuous venovenous hemofiltration (CVVH) d. Continuous ambulatory peritoneal dialysis (CAPD)

b. Automated peritoneal dialysis (APD) is the type of dialysis in which the patient dialyzes during sleep and leaves the fluid in the abdomen during the day. Long nocturnal hemodialysis occurs while the patient is sleeping and is done up to six times per week. Continuous venovenous hemofiltration (CVVH) is a type of continuous renal replacement therapy used to treat AKI. Continuous ambulatory peritoneal dialysis (CAPD) is dialysis that is done with exchanges of 1.5 to 3 L of dialysate at least four times daily.

29. When the patient with parathyroid disease experiences symptoms of hypocalcemia, what is a measure that can be used to temporarily raise serum calcium levels? a. Administer IV normal saline. b. Have patient rebreathe in a paper bag. c. Administer furosemide (Lasix) as ordered. d. Administer oral phosphorus supplements.

b. Have patient rebreathe in a paper bag. Rebreathing in a paper bag promotes carbon dioxide retention in the blood, which lowers pH and creates an acidosis. An acidemia enhances the solubility and ionizationof calcium, increasing the proportion of total body calcium available in physiologically active form and relieving the symptoms of hypocalcemia. Saline promotes calcium excretion, as does furosemide. Phosphate levels in the blood are reciprocal to calcium and an increase in phosphate promotes calcium excretion.

What is the most serious electrolyte disorder associated with kidney disease? a. Hypocalcemia b. Hyperkalemia c. Hyponatremia d. Hypermagnesemia

b. Hyperkalemia can lead to life-threatening dysrhythmias. Hypocalcemia leads to an accelerated rate of bone remodeling and potentially to tetany. Hyponatremia may lead to confusion. Elevated sodium levels lead to edema, hypertension, and heart failure. Hypermagnesemia may decrease reflexes, mental status, and blood pressure.

33. A patient with Addison's disease comes to the emergency department with complaints of nausea, vomiting, diarrhea, and fever. What collaborative care should the nurse expect? a. IV administration of vasopressors b. IV administration of hydrocortisone c. IV administration of D5W with 20 mEq KCl d. Parenteral injections of adrenocorticotropic hormone (ACTH)

b. IV administration of hydrocortisone Vomiting and diarrhea are early indicators of Addisonian crisis and fever indicates an infection, which is causing additional stress for the patient. Treatment of a crisis requires immediate glucocorticoid replacement and IV hydrocortisone, fluids, sodium, and glucose are necessary for 24 hours. Addison's disease is a primary insufficiency of the adrenal gland and adrenocorticotropic hormone (ACTH) is not effective, nor would vasopressors be effective with the fluid deficiency of Addison's disease. Potassium levels are increased in Addison's disease and KCl would be contraindicated.

38. Priority Decision: What is the priority nursing intervention during the management of the patient with pheochromocytoma? a. Administering IV fluids b. Monitoring blood pressure c. Administering β-adrenergic blockers d. Monitoring intake and output and daily weights

b. Monitoring blood pressure Pheochromocytoma is a catecholamine-producing tumor of the adrenal medulla, which may cause severe, episodic hypertension; severe, pounding headache; and profuse sweating. Monitoring for a dangerously high BP before surgery is critical, as is monitoring for BP fluctuations during medical and surgical treatment.

The patient with CKD asks why she is receiving nifedipine (Procardia) and furosemide (Lasix). The nurse understands that these drugs are being used to treat the patient's a. anemia. b. hypertension. c. hyperkalemia. d. mineral and bone disorder.

b. Nifedipine (Procardia) is a calcium channel blocker and furosemide (Lasix) is a loop diuretic. Both are used to treat hypertension.

In replying to a patient's questions about the seriousness of her chronic kidney disease (CKD), the nurse knows that the stage of CKD is based on what? a. Total daily urine output b. Glomerular filtration rate c. Degree of altered mental status d. Serum creatinine and urea levels

b. Stages of chronic kidney disease are based on the GFR. No specific markers of urinary output, mental status, or azotemia classify the degree of chronic kidney disease (CKD).

12. When caring for a patient with nephrogenic diabetes insipidus, what should the nurse expect the treatment to include? a. Fluid restriction b. Thiazide diuretics c. A high-sodium diet d. Chlorpropamide (Diabinese)

b. Thiazide diuretics In nephrogenic diabetes insipidus, the kidney is unable to respond to ADH, so vasopressin or hormone analogs are not effective. Thiazide diuretics slow the glomerular filtration rate (GFR) in the kidney and produce a decrease in urine output. Low-sodium diets (<3 g/day) are also thought to decrease urine output. Fluids are not restricted because the patient could easily become dehydrated.

A 68-year-old man with a history of heart failure resulting from hypertension has AKI as a result of the effects of nephrotoxic diuretics. Currently his serum potassium is 6.2 mEq/L (6.2 mmol/L) with cardiac changes, his BUN is 108 mg/dL (38.6 mmol/L), his serum creatinine is 4.1 mg/dL (362 mmol/L), and his serum HCO3 − is 14 mEq/L (14 mmol/L). He is somnolent and disoriented. Which treatment should the nurse expect to be used for him? a. Loop diuretics b. Renal replacement therapy c. Insulin and sodium bicarbonate d. Sodium polystyrene sulfonate (Kayexalate)

b. This patient has at least three of the six common indications for renal replacement therapy (RRT), including (1) high potassium level, (2) metabolic acidosis, and (3) changed mental status. The other indications are (4) volume overload, resulting in compromised cardiac status (this patient has a history of hypertension), (5) BUN greater than 120 mg/dL, and (6) pericarditis, pericardial effusion, or cardiac tamponade. Although the other treatments may be used, they will not be as effective as RRT for this older patient. Loop diuretics and increased fluid are used if the patient is dehydrated. Insulin and sodium bicarbonate can be used to temporarily drive the potassium into the cells. Sodium polystyrene sulfonate (Kayexalate) is used to actually decrease the amount of potassium in the body.

*3. The immunologic mechanisms involved in acute poststreptococal glomerulonephritis include:* a. tubular blocking by precipitates of bacteria and antibody reactions b. deposition of immune complexes and complement along the GBM c. thickening of the GBM from autoimmune microangiopathic changes d. destruction of glomeruli by proteolytic enzymes contained in the GBM

b. deposition of immune complexes and complement along the GBM

A patient has been diagnosed with osteosarcoma of the humerus. He shows an understanding of his treatment options when he states a."I accept that I have to lose my arm with surgery." b."The chemotherapy before surgery will shrink the tumor." c."This tumor is related to the melanoma I had 3 years ago." d."I'm glad they can take out the cancer with such a small scar."

b."The chemotherapy before surgery will shrink the tumor." A patient with osteosarcoma usually has preoperative chemotherapy to decrease tumor size before surgery. As a result, limb-salvage procedures, including a wide surgical resection of the tumor, are being used more often. Osteosarcoma is a primary bone tumor that is extremely aggressive and rapidly metastasizes to distant sites.

You are teaching a patient with osteopenia. What is important to include in the teaching plan? a.Lose weight. b.Stop smoking. c.Eat a high-protein diet. d.Start swimming for exercise

b.Stop smoking. Patients with osteopenia should be instructed to quit smoking in order to decrease loss of bone mass.

*10. A patient with ureterolithotomy returns from surgery with a nephrostomy tube in place. Postoperative nursing care of the patient includes:* a.encourage the patient to drink fruit juices and milk b.encouraging fluids of at least 2-3 L/day after nausea has subsided c. irrigating the nephrostomy tube with 10ml of NS solution as needed d. notifying the physician if nephrostomy tube drainage is more than 30ml/hr

b.encouraging fluids of at least 2-3 L/day after nausea has subsided

An 83-year-old female patient was found lying on the bathroom floor. She said she fell 2 days ago and has not been able to take her heart medicine or eat or drink anything since then. What conditions could be causing prerenal AKI in this patient (select all that apply)? a. Anaphylaxis b. Renal calculi c. Hypovolemia d. Nephrotoxic drugs e. Decreased cardiac output

c, e. Because the patient has had nothing to eat or drink for 2 days, she is probably dehydrated and hypovolemic. Decreased cardiac output (CO) is most likely because she is older and takes heart medicine, which is probably for heart failure or hypertension. Both hypovolemia and decreased CO cause prerenal AKI. Anaphylaxis is also a cause of prerenal AKI but is not likely in this situation. Nephrotoxic drugs would contribute to intrarenal causes of AKI and renal calculi would be a postrenal cause of AKI.

The patient with chronic kidney disease is considering whether to use peritoneal dialysis (PD) or hemodialysis (HD). What are advantages of PD when compared to HD (select all that apply)? a. Less protein loss b. Rapid fluid removal c. Less cardiovascular stress d. Decreased hyperlipidemia e. Requires fewer dietary restrictions

c, e. Peritoneal dialysis is less stressful for the cardiovascular system and requires fewer dietary restrictions. Peritoneal dialysis actually contributes to more protein loss and increased hyperlipidemia. The fluid and creatinine removal are slower with peritoneal dialysis than hemodialysis.

A patient on hemodialysis develops a thrombus of a subcutaneous arteriovenous (AV) graft, requiring its removal. While waiting for a replacement graft or fistula, the patient is most likely to have what done for treatment? a. Peritoneal dialysis b. Peripheral vascular access using radial artery c. Silastic catheter tunneled subcutaneously to the jugular vein d. Peripherally inserted central catheter (PICC) line inserted into subclavian vein

c. A more permanent, soft, flexible Silastic double-lumen catheter is used for long-term access when other forms of vascular access have failed. These catheters are tunneled subcutaneously and have Dacron cuffs that prevent infection from tracking along the catheter.

35. A patient who is on corticosteroid therapy for treatment of an autoimmune disorder has the following additional drugs ordered. Which one is used to prevent corticosteroid-induced osteoporosis? a. Potassium b. Furosemide (Lasix) c. Alendronate (Fosamax) d. Pantoprazole (Protonix)

c. Alendronate (Fosamax) Alendronate (Fosamax) is used to prevent corticosteroidinduced osteoporosis. Potassium is used to prevent the mineralocorticoid effect of hypokalemia. Furosemide (Lasix) is used to decrease sodium and fluid retention from the mineralocorticoid effect. Pantoprazole (Protonix) is used to prevent gastrointestinal (GI) irritation from an increase in secretion of pepsin and hydrochloric acid.

30. A patient with hypoparathyroidism resulting from surgical treatment of hyperparathyroidism is preparing for discharge. What should the nurse teach the patient? a. Milk and milk products should be increased in the diet. b. Parenteral replacement of parathyroid hormone will be required for life. c. Calcium supplements with vitamin D can effectively maintain calcium balance. d. Bran and whole-grain foods should be used to prevent GI effects of replacement therapy.

c. Calcium supplements with vitamin D can effectively maintain calcium balance The hypocalcemia that results from PTH deficiency is controlled with calcium and vitamin D supplementation and possibly oral phosphate binders. Replacement with PTH is not used because of antibody formation to PTH, the need for parenteral administration, and cost. Milk products, although good sources of calcium, also have high levels of phosphate, which reduce calcium absorption. Whole grains and foods containing oxalic acid also impair calcium absorption.

A patient with AKI is a candidate for continuous renal replacement therapy (CRRT). What is the most common indication for use of CRRT? a. Azotemia b. Pericarditis c. Fluid overload d. Hyperkalemia

c. Continuous renal replacement therapy (CRRT) is indicated for the patient with AKI as an alternative or adjunct to hemodialysis to slowly remove solutes and fluid in the hemodynamically unstable patient. It is especially useful for treatment of fluid overload, but hemodialysis is indicated for treatment of hyperkalemia, pericarditis, or other serious effects of uremia.

What does the dialysate for PD routinely contain? a. Calcium in a lower concentration than in the blood b. Sodium in a higher concentration than in the blood c. Dextrose in a higher concentration than in the blood d. Electrolytes in an equal concentration to that of the blood

c. Dextrose or icodextrin or amino acid is added to dialysate fluid to create an osmotic gradient across the membrane to remove excess fluid from the blood. The dialysate fluid has no potassium so that potassium will diffuse into the dialysate from the blood. Dialysate also usually contains higher calcium to promote its movement into the blood. Dialysate sodium is usually less than or equal to that of blood to prevent sodium and fluid retention.

16. A patient is admitted to the hospital with thyrotoxicosis. On physical assessment of the patient, what should the nurse expect to find? a. Hoarseness and laryngeal stridor b. Bulging eyeballs and dysrhythmias c. Elevated temperature and signs of heart failure d. Lethargy progressing suddenly to impairment of consciousness

c. Elevated temperature and signs of heart failure A hyperthyroid crisis results in marked manifestations of hyperthyroidism, with severe tachycardia, heart failure, shock, hyperthermia, restlessness, irritability, abdominal pain, vomiting, diarrhea, delirium, and coma. Although exophthalmos may be present in the patient with Graves' disease, it is not a significant factor in hyperthyroid crisis. Hoarseness and laryngeal stridor are characteristic of the tetany of hypoparathyroidism and lethargy progressing to coma is characteristic of myxedema coma, a complication of hypothyroidism.

32. A patient is scheduled for a bilateral adrenalectomy. During the postoperative period, what should the nurse expect related to the administration of corticosteroids? a. Reduced to promote wound healing b. Withheld until symptoms of hypocortisolism appear c. Increased to promote an adequate response to the stress of surgery d. Reduced because excessive hormones are released during surgical manipulation of adrenal glands

c. Increased to promote an adequate response to the stress of surgery Although the patient with Cushing syndrome has excess corticosteroids, removal of the glands and the stress of surgery require that high doses of corticosteroids (cortisone) be administered postoperatively for several days before weaning the dose. The nurse should monitor the patient's vital signs postoperatively to detect whether large amounts of hormones were released during surgical manipulation, obtain morning urine specimens for cortisol measurement to evaluate the effectiveness of the surgery, and provide dressing changes with aseptic technique to avoid infection as usual inflammatory responses are suppressed.

A patient with mild iatrogenic Cushing syndrome is on an alternate-day regimen of corticosteroid therapy. What does the nurse explain to the patient about this regimen? a. It maintains normal adrenal hormone balance. b. It prevents ACTH release from the pituitary gland. c. It minimizes hypothalamic-pituitary-adrenal suppression. d. It provides a more effective therapeutic effect of the drug.

c. It minimizes hypothalamic-pituitary-adrenal suppresion Taking corticosteroids on an alternate-day schedule for pharmacologic purposes is less likely to suppress ACTH production from the pituitary and prevent adrenal atrophy. Normal adrenal hormone balance is not maintained during glucocorticoid therapy because excessive exogenous hormone is used.

For a patient with CKD the nurse identifies a nursing diagnosis of risk for injury: fracture related to alterations in calcium and phosphorus metabolism. What is the pathologic process directly related to the increased risk for fractures? a. Loss of aluminum through the impaired kidneys b. Deposition of calcium phosphate in soft tissues of the body c. Impaired vitamin D activation resulting in decreased GI absorption of calcium d. Increased release of parathyroid hormone in response to decreased calcium levels

c. The calcium-phosphorus imbalances that occur in CKD result in hypocalcemia, from a deficiency of active vitamin D and increased phosphorus levels. This leads to an increased rate of bone remodeling with a weakened bone matrix. Aluminum accumulation is also believed to contribute to the osteomalacia. Osteitis fibrosa involves replacement of calcium in the bone with fibrous tissue and is primarily a result of elevated levels of parathyroid hormone resulting from hypocalcemia.

During the nursing assessment of the patient with renal insufficiency, the nurse asks the patient specifically about a history of a. angina. b. asthma. c. hypertension. d. rheumatoid arthritis.

c. The most common causes of CKD in the United States are diabetes mellitus and hypertension. The nurse should obtain information on long-term health problems that are related to kidney disease. The other disorders are not closely associated with renal disease.

1. When instructing a patient regarding a urine study for free cortisol, what is most important for the nurse to tell the patient? a. Save the first voided urine in the morning. b. Maintain a high-sodium diet 3 days before collection. c. Try to avoid stressful situations during the collection period. d. Complete at least 30 minutes of exercise before collecting the urine sample.

c. Try to avoid stressful situations during the collection period. A urine study for free cortisol requires a 24-hour urine collection. The patient should be instructed to avoid stressful situations and excessive physical exercise that could unduly increase cortisol levels. The patient should also maintain a low-sodium diet before and during the urine collection period.

The patient with diabetes insipidus is brought to the emergency department with confusion and dehydration after excretion of a large volume of urine today even though several liters of fluid were drunk. What is a diagnostic test that the nurse should expect to be done to help make a diagnosis? a. Blood glucose b. Serum sodium level c. Urine specific gravity d. Computed tomography (CT) of the head

c. Urine specific gravity Patients with diabetes insipidus (DI) excrete large amounts of urine with a specific gravity of less than 1.005. Blood glucose would be tested to diagnose diabetes mellitus. The serum sodium level is expected to be low with DI but is not diagnostic. To diagnose central DI a water deprivation test is required. Then a CT of the head may be done to determine the cause. Nephrogenic DI is differentiated from central DI with determination of the level of ADH after an analog of ADH is given.

11. A patient with diabetes insipidus is treated with nasal desmopressin acetate (DDAVP). The nurse determines that the drug is not having an adequate therapeutic effect when the patient experiences a. headache and weight gain. b. nasal irritation and nausea. c. a urine specific gravity of 1.002. d. an oral intake greater than urinary output.

c. a urine specific gravity of 1.002 Normal urine specific gravity is 1.005 to 1.025 and urine with a specific gravity of 1.002 is very dilute, indicating that there continues to be excessive loss of water and that treatment of diabetes insipidus is inadequate. Headache, weight gain, and oral intake greater than urinary output are signs of volume excess that occur with overmedication. Nasal irritation and nausea may also indicate overdosage.

A patient with cholelithiasis needs to have the gallbladder removed. Which patient assessment is a contraindication for a cholecystectomy? Low-grade fever of 100° F and dehydration Abscess in the right upper quadrant of the abdomen Activated partial thromboplastin time (aPTT) of 54 seconds

ctivated partial thromboplastin time (aPTT) of 54 seconds Multiple obstructions in the cystic and common bile duct An aPTT of 54 seconds is above normal and indicates insufficient clotting ability. If the patient had surgery, significant bleeding complications postoperatively are very likely. Fluids can be given to eliminate the dehydration; the abscess can be assessed, and the obstructions in the cystic and common bile duct would be relieved with the cholecystectomy.

The nursing management of the patient with cholecystitis associated with cholelithiasis is based on the knowledge that a. shock-wave therapy should be tried initially. b. once gallstones are removed, they tend not to recur. c. the disorder can be successfully treated with oral bile salts that dissolve gallstones. d. laparoscopic cholecystectomy is the treatment of choice in most patients who are symptomatic. (Lewis 1042)

d Rationale: Laparoscopic cholecystectomy is the treatment of choice for symptomatic cholelithiasis.

What are intrarenal causes of acute kidney injury (AKI) (select all that apply)? a. Anaphylaxis b. Renal stones c. Bladder cancer d. Nephrotoxic drugs e. Acute glomerulonephritis f. Tubular obstruction by myoglobin

d, e, f. Intrarenal causes of acute kidney injury (AKI) include conditions that cause direct damage to the kidney tissue, including nephrotoxic drugs, acute glomerulonephritis, and tubular obstruction by myoglobin, or prolonged ischemia. Anaphylaxis and other prerenal problems are frequently the initial cause of AKI. Renal stones and bladder cancer are among the postrenal causes of AKI.

15. A patient with Graves' disease asks the nurse what caused the disorder. What is the best response by the nurse? a. "The cause of Graves' disease is not known, although it is thought to be genetic." b. "It is usually associated with goiter formation from an iodine deficiency over a long period of time." c. "Antibodies develop against thyroid tissue and destroy it, causing a deficiency of thyroid hormones." d. "In genetically susceptible persons, antibodies are formed that cause excessive thyroid hormone secretion."

d. "In genetically susceptible persons, antibodies are formed that cause excessive thyroid hormone secretion" In Graves' disease, antibodies to the TSH receptor are formed, attach to the receptors, and stimulate the thyroid gland to release triiodothyronine (T3), thyroxine (T4), or both, creating hyperthyroidism. The disease is not directly genetic but individuals appear to have a genetic susceptibility to develop autoimmune antibodies. Goiter formation from insufficient iodine intake is usually associated with hypothyroidism.

Priority Decision: What is the most appropriate snack for the nurse to offer a patient with stage 4 CKD? a. Raisins b. Ice cream c. Dill pickles d. Hard candy

d. A patient with CKD may have unlimited intake of sugars and starches (unless the patient is diabetic) and hard candy is an appropriate snack and may help to relieve the metallic and urine taste that is common in the mouth. Raisins are a high-potassium food. Ice cream contains protein and phosphate and counts as fluid. Pickled foods have high sodium content. Lewis, Sharon L.; Dirksen, Shannon Ruff; Bucher, Linda (2014-03-14). Study Guide for Medical-Surgical Nursing: Assessment and Management of Clinical Problems (Study Guide for Medical-Surgical Nursing: Assessment & Management of Clinical Problem) (Page 413). Elsevier Health Sciences. Kindle Edition.

Acute tubular necrosis (ATN) is the most common cause of intrarenal AKI. Which patient is most likely to develop ATN? a. Patient with diabetes mellitus b. Patient with hypertensive crisis c. Patient who tried to overdose on acetaminophen d. Patient with major surgery who required a blood transfusion

d. Acute tubular necrosis (ATN) is primarily the result of ischemia, nephrotoxins, or sepsis. Major surgery is most likely to cause severe kidney ischemia in the patient requiring a blood transfusion. A blood transfusion hemolytic reaction produces nephrotoxic injury if it occurs. Diabetes mellitus, hypertension, and acetaminophen overdose will not contribute to ATN.

The hypothalamus secretes releasing hormones and inhibiting hormones. What is the target tissue of these releasing hormones and inhibiting hormones? a. Pineal b. Adrenal cortex c. Posterior pituitary d. Anterior pituitary

d. Anterior pituitary The anterior pituitary is the target tissue of the releasing hormones (corticotropin releasing hormone, thyrotropin releasing hormone, growth hormone releasing factor, gonadotropin releasing hormone, prolactin releasing factor) and the inhibiting hormones (somatostatin, prolactin inhibiting factor). These hormones release or inhibit other hormones that affect the thyroid, adrenal cortex, pancreas, reproductive organs, and all body cells. The pineal gland is not directly affected by the releasing and inhibiting hormones from the hypothalamus. The posterior pituitary releases antidiuretic hormone (ADH) in response to plasma osmolality changes that is not directly affected by the hypothalamus hormones.

Which serum laboratory value indicates to the nurse that the patient's CKD is getting worse? a. Decreased BUN b. Decreased sodium c. Decreased creatinine d. Decreased calculated glomerular filtration rate (GFR)

d. As GFR decreases, BUN and serum creatinine levels increase. Although elevated BUN and creatinine indicate that waste products are accumulating, the calculated GFR is considered a more accurate indicator of kidney function than BUN or serum creatinine.

22. What is a cause of primary hypothyroidism in adults? a. Malignant or benign thyroid nodules b. Surgical removal or failure of the pituitary gland c. Surgical removal or radiation of the thyroid gland d. Autoimmune-induced atrophy of the thyroid gland

d. Autoimmune-induced atrophy of the thyroid gland Both Graves' disease and Hashimoto's thyroiditis are autoimmune disorders that eventually destroy the thyroid gland, leading to primary hypothyroidism. Thyroid tumors most often result in hyperthyroidism. Secondary hypothyroidism occurs as a result of pituitary failure and iatrogenic hypothyroidism results from thyroidectomy or radiation of the thyroid gland.

While caring for the patient in the oliguric phase of AKI, the nurse monitors the patient for associated collaborative problems. When should the nurse notify the health care provider? a. Urine output is 300 mL/day. b. Edema occurs in the feet, legs, and sacral area. c. Cardiac monitor reveals a depressed T wave and elevated ST segment. d. The patient experiences increasing muscle weakness and abdominal cramping.

d. Hyperkalemia is a potentially life-threatening complication of AKI in the oliguric phase. Muscle weakness and abdominal cramping are signs of the neuromuscular impairment that occurs with hyperkalemia. In addition, hyperkalemia can cause the cardiac conduction abnormalities of peaked T wave, prolonged PR interval, prolonged QRS interval, and depressed ST segment. Urine output of 300 mL/day is expected during the oliguric phase, as is the development of peripheral edema.

26. A patient who recently had a calcium oxalate renal stone had a bone density study, which showed a decrease in her bone density. What endocrine problem could this patient have? a. SIADH b. Hypothyroidism c. Cushing syndrome d. Hyperparathyroidism

d. Hyperparathyroidism The patient with hyperparathyroidism may have calcium nephrolithiasis, skeletal pain, decreased bone density, psychomotor retardation, or cardiac dysrhythmias. The other endocrine problems would not be related to calcium kidney stones or decreased bone density.

What indicates to the nurse that a patient with oliguria has prerenal oliguria? a. Urine testing reveals a low specific gravity. b. Causative factor is malignant hypertension. c. Urine testing reveals a high sodium concentration. d. Reversal of oliguria occurs with fluid replacement.

d. In prerenal oliguria, the oliguria is caused by a decrease in circulating blood volume and there is no damage yet to the renal tissue. It can be reversed by correcting the precipitating factor, such as fluid replacement for hypovolemia. Prerenal oliguria is characterized by urine with a high specific gravity and a low sodium concentration, whereas oliguria of intrarenal failure is characterized by urine with a low specific gravity and a high sodium concentration. Malignant hypertension causes damage to renal tissue and intrarenal oliguria.

What accurately describes the care of the patient with CKD? a. A nutrient that is commonly supplemented for the patient on dialysis because it is dialyzable is iron. b. The syndrome that includes all of the signs and symptoms seen in the various body systems in CKD is azotemia. c. The use of morphine is contraindicated in the patient with CKD because accumulation of its metabolites may cause seizures. d. The use of calcium-based phosphate binders in the patient with CKD is contraindicated when serum calcium levels are increased.

d. In the patient with CKD, when serum calcium levels are increased, calcium-based phosphate binders are not used. The nutrient supplemented for patients on dialysis is folic acid. The various body system manifestations occur with uremia, which includes azotemia. Meperidine is contraindicated in patients with CKD related to possible seizures.

10. In a patient with central diabetes insipidus, what will the administration of ADH during a water deprivation test result in? a. Decrease in body weight b. Increase in urinary output c. Decrease in blood pressure d. Increase in urine osmolality

d. Increase in urine osmolality A patient with central diabetes insipidus has a deficiency of ADH with excessive loss of water from the kidney, hypovolemia, hypernatremia, and dilute urine with a low specific gravity. When vasopressin is administered, the symptoms are reversed, with water retention, decreased urinary output that increases urine osmolality, and an increase in BP.

In caring for the patient with AKI, what should the nurse be aware of? a. The most common cause of death in AKI is irreversible metabolic acidosis. b. During the oliguric phase of AKI, daily fluid intake is limited to 1000 mL plus the prior day's measured fluid loss. c. Dietary sodium and potassium during the oliguric phase of AKI are managed according to the patient's urinary output. d. One of the most important nursing measures in managing fluid balance in the patient with AKI is taking accurate daily weights.

d. Measuring daily weights with the same scale at the same time each day allows for the evaluation and detection of excessive body fluid gains or losses. Infection is the leading cause of death in AKI, so meticulous aseptic technique is critical. The fluid limitation in the oliguric phase is 600 mL plus the prior day's measured fluid loss. Dietary sodium and potassium intake are managed according to the plasma levels.

21. When providing discharge instructions to a patient who had a subtotal thyroidectomy for hyperthyroidism, what should the nurse teach the patient? a. Never miss a daily dose of thyroid replacement therapy. b. Avoid regular exercise until thyroid function is normalized. c. Use warm saltwater gargles several times a day to relieve throat pain. d. Substantially reduce caloric intake compared to what was eaten before surgery.

d. Substantially reduce caloric intake compared to what was eaten before surgery With the decrease in thyroid hormone postoperatively, calories need to be reduced substantially to prevent weight gain. When a patient has had a subtotal thyroidectomy, thyroid replacement therapy is not given because exogenous hormone inhibits pituitary production of TSH and delays or prevents the restoration of thyroid tissue regeneration. Regular exercise stimulates the thyroid gland and is encouraged. Saltwater gargles are used for dryness and irritation of the mouth and throat following radioactive iodine therapy.

What indicates to the nurse that a patient with AKI is in the recovery phase? a. A return to normal weight b. A urine output of 3700 mL/day c. Decreasing sodium and potassium levels d. Decreasing blood urea nitrogen (BUN) and creatinine levels

d. The blood urea nitrogen (BUN) and creatinine levels remain high during the oliguric and diuretic phases of AKI. The recovery phase begins when the glomerular filtration returns to a rate at which BUN and creatinine stabilize and then decrease. Urinary output of 3 to 5 L/ day, decreasing sodium and potassium levels, and fluid weight loss are characteristic of the diuretic phase of AKI.

A patient's recent medical history is indicative of diabetes insipidus. The nurse would perform patient teaching related to which diagnostic test? a. Thyroid scan b. Fasting glucose test c. Oral glucose tolerance d. Water deprivation test

d. Water deprivation test A water deprivation test is used to diagnose the polyuria that accompanies diabetes insipidus. Glucose tests and thyroid tests are not directly related to the diagnosis of diabetes insipidus.

*1. In teaching a patient with pyelonephritis about the disorder, the nurse informs the paitent that the organisms that cause pyelonephritis most commonly reach the kidneys through* a. the bloodstream b.the lymphatic system c. a descending infection d. an ascending infection

d. an ascending infection

*11. A patient has has a cystectomy and ileal conduit diversion performed. Four days postoperatively, mucous shred are seen in the drainage bag. The nurse should.* a. notify the physician b.notify the charge nurse c. irrigate the drainage tube d. chart it as a normal observation

d. chart it as a normal observation

A patient suspected of having acromegaly has an elevated plasma growth hormone (GH) level. In acromegaly, what would the nurse also expect the patient's diagnostic results to indicate? a. Hyperinsulinemia b. Plasma glucose of <70 mg/dL (3.9 mmol/L) c. Decreased GH levels with an oral glucose challenge test d. Elevated levels of plasma insulin-like growth factor-1 (IGF-1)

d. elevated levels of plasma insulin-like growth factor-1 (IGF-1) A normal response to growth hormone (GH) secretion is stimulation of the liver to produce somatomedin C, or insulin-like growth factor-1 (IGF-1), which stimulates growth of bones and soft tissues. The increased levels of somatomedin C normally inhibit GH but in acromegaly the pituitary gland secretes GH despite elevated IGF-1 levels. When both GH and IGF-1 levels are increased, overproduction of GH is confirmed. GH also causes elevation of blood glucose and normally GH levels fall during an oral glucose challenge but not in acromegaly.

4. What findings are commonly found in a patient with a prolactinoma? a. Gynecomastia in men b. Profuse menstruation in women c. Excess follicle-stimulating hormone (FSH) and luteinizing hormone (LH) d. Signs of increased intracranial pressure, including headache, nausea, and vomiting

d. signs of increased intracranial pressure, including headache, nausea, and vomiting. Compression of the optic chiasm can cause visual problems as well as signs of increased intracranial pressure, including headache, nausea, and vomiting. About 30% of prolactinomas will have excess prolactin secretion with manifestations of impotence in men, galactorrhea or amenorrhea in women without relationship to pregnancy, and decreased libido in both men and women. There is decreased follicle-stimulating hormone (FSH) and luteinizing hormone (LH).

*7. The nurse recommends genetic counseling for the children of a patient with* a.nephrotic syndrome b.chronic pyelonephritis c. malignant nephrosclerosis d.adult onset polycystic kidney disease

d.adult onset polycystic kidney disease

*5.The edema that occurs in nephrotic syndrome is due to* a. increased hydrostatic pressure caused by sodium retention. b. decreased aldosterone secretion from adrenal insufficiency. c. increased fluid retention caused by decreased glomerular filtration d.decreased colloidal osmotic pressure caused by loss of serum albumin

d.decreased colloidal osmotic pressure caused by loss of serum albumin

*8. The nurse identifies a risk factor for kidney and bladder cancer in a patient who relates a history of* a.aspirin use b.tobacco use c.chronic alcohol abuse d.use of artificial sweeteners

b.tobacco use

*2. The nurse teaches the female paitent who has frequent UTIs that she should* a. take tub baths with bubble bath b.urinate before and after sexual intercourse c.take prophylactic sufonamides for the rest of her life d. restrict fluid intake to prevent the need for frequent voiding

b.urinate before and after sexual intercourse

The patient with CKD is brought to the emergency department with Kussmaul respirations. What does the nurse know about CKD that could cause this patient's Kussmaul respirations? a. Uremic pleuritis is occurring. b. There is decreased pulmonary macrophage activity. c. They are caused by respiratory compensation for metabolic acidosis. d. Pulmonary edema from heart failure and fluid overload is occurring.

c. Kussmaul respirations occur with severe metabolic acidosis when the respiratory system is attempting to compensate by removing carbon dioxide with exhalations. Uremic pleuritis would cause a pleural friction rub. Decreased pulmonary macrophage activity increases the risk of pulmonary infection. Dyspnea would occur with pulmonary edema.

37. When caring for a patient with primary hyperaldosteronism, the nurse would question a health care provider's prescription for which drug? a. Furosemide (Lasix) b. Amiloride (Midamor) c. Spironolactone (Aldactone) d. Aminoglutethimide (Cytadren)

a. Furosemide (Lasix) Hyperaldosteronism is an excess of aldosterone, which is manifested by sodium and water retention and potassium excretion. Furosemide is a potassium-wasting diuretic that would increase the potassium deficiency. Aminoglutethimide blocks aldosterone synthesis. Spironolactone and amiloride are potassium-sparing diuretics.

7. During care of the patient with SIADH, what should the nurse do? a. Monitor neurologic status at least every 2 hours. b. Teach the patient receiving treatment with diuretics to restrict sodium intake. c. Keep the head of the bed elevated to prevent antidiuretic hormone (ADH) release. d. Notify the health care provider if the patient's blood pressure decreases more than 20 mm Hg from baseline.

a. Monitor neurologis status at least every 2 hours The patient with syndrome of inappropriate antidiuretic hormone (SIADH) has marked dilutional hyponatremia and should be monitored for decreased neurologic function and seizures every 2 hours. Sodium intake is supplemented because of the hyponatremia and sodium loss caused by diuretics. ADH release is reduced by keeping the head of the bed flat to increase left atrial filling pressure. A reduction in blood pressure (BP) indicates a reduction in total fluid volume and is an expected outcome of treatment.

The nurse is conducting discharge teaching for a patient with metastatic lung cancer who was admitted with a bowel impaction. Which instructions would be most helpful to prevent further episodes of constipation? A. Maintain a high intake of fluid and fiber in the diet. B. Reduce intake of medications causing constipation. C. Eat several small meals per day to maintain bowel motility. D. Sit upright during meals to increase bowel motility by gravity.

A Increased fluid intake and a high-fiber diet reduce the incidence of constipation caused by immobility, medications, and other factors. Fluid and fiber provide bulk that in turn increases peristalsis and bowel motility. Analgesics taken for lung cancer probably cannot be reduced. Other medications may decrease constipation, but it is best to avoid laxatives. Eating several small meals per day and position do not facilitate bowel motility. Defecation is easiest when the person sits on the commode with the knees higher than the hips.

The wound, ostomy, and continence (WOC) nurse selects the site where the ostomy will be placed. What should be included in the consideration for the site? A. The patient must be able to see the site. B. Outside the rectus muscle area is the best site. C. It is easier to seal the drainage bag to a protruding area. D. The ostomy will need irrigation, so area should not be tender.

A In selection of the ostomy site, the WOC nurse will want a site visible to the patient so the patient can take care of it, within the rectus muscle to avoid hernias, and on a flat surface to more easily create a good seal with the drainage bag.

Which of the following patients would be more likely to have the highest risk of developing malignant melanoma? A) A fair-skinned woman who uses a tanning booth regularly B) An African American patient with a family history of cancer C) A Hispanic male with a history of psoriasis and eczema that responded poorly to treatment D) An adult who required phototherapy as an infant for the treatment of hyperbilirubinemia

A) A fair-skinned woman who uses a tanning booth regularly Risk factors for malignant melanoma include a fair complexion and exposure to ultraviolet light. Psoriasis, eczema, short-duration phototherapy and a family history of other cancers are less likely to be linked to malignant melanoma.

Which of the following interventions would be most helpful in managing a patient newly admitted with cellulitis of the right foot? A) Applying warm, moist heat B) Limiting ambulation to three times daily C) Keeping the foot at or below heart level D) Wrapping the foot snugly in warm blankets

A) Applying warm, moist heat The application of warm, moist heat speeds the resolution of inflammation and infection when accompanied by appropriate antibiotic therapy. It does this by increasing local circulation to the affected area to bring macrophages to the area and carry off cellular debris.

A female patient reports that she is experiencing burning on urination, frequency, and urgency. The nurse notes that a clean-voided urine specimen is markedly cloudy. The probable cause of these symptoms and findings is: A) Cystitis. B) Hematuria. C) Pyelonephritis. D) Dysuria.

A) Cystitis.

20. A patient with type 1 diabetes has been using self-monitoring of blood glucose (SMBG) as part of diabetes management. During evaluation of the patient's technique of SMBG, the nurse identifies a need for additional teaching when the patient a. chooses a puncture site in the center of the finger pad. b. washes the puncture site using soap and water. c. says the result of 130 mg indicates good blood sugar control. d. hangs the arm down for a minute before puncturing the site.

A. chooses a puncture site in the center of the finger pad Rationale: The patient is taught to choose a puncture site at the side of the finger pad. The other patient actions indicate that teaching has been effective.

When doing discharge teaching for a patient who has had a repair of a fractured mandible, the nurse will include information about a. when and how to cut the immobilizing wires. b. self-administration of nasogastric tube feedings. c. the use of sterile technique for dressing changes. d. the importance of including high-fiber foods in the diet.

ANS: A The jaw will be wired for stabilization, and the patient should know what emergency situations require that the wires be cut to protect the airway. There are no dressing changes for this procedure. The diet is liquid, and patients are not able to chew high fiber foods. Initially, the patient may receive nasogastric tube feedings, but by discharge the patient will swallow liquid through a straw.

The nurse is caring for a patient with colon cancer who is scheduled for external radiation therapy to the abdomen. Which information obtained by the nurse would indicate a need for patient teaching? a. The patient swims a mile 3 days a week. b. The patient snacks frequently during the day. c. The patient showers everyday with a mild soap. d. The patient has a history of dental caries with amalgam fillings.

ANS: A The patient is instructed to avoid swimming in salt water or chlorinated pools during the treatment period. The patient does not need to change habits of eating frequently or showering with a mild soap. A history of dental caries will not impact the patient who is scheduled for abdominal radiation.

Which statement by a patient who has had an above-the-knee amputation indicates that the nurse's discharge teaching has been effective? a. "I should lay on my abdomen for 30 minutes 3 or 4 times a day." b. "I should elevate my residual limb on a pillow 2 or 3 times a day." c. "I should change the limb sock when it becomes soiled or stretched out." d. "I should use lotion on the stump to prevent drying and cracking of the skin."

ANS: A The patient lies in the prone position several times daily to prevent flexion contractures of the hip. The limb sock should be changed daily. Lotion should not be used on the stump. The residual limb should not be elevated because this would encourage flexion contracture.

On the first postoperative day, a patient with a below-the-knee amputation complains of pain in the amputated limb. Which action is best for the nurse to take? a. Explain the reasons for the phantom limb pain. b. Administer prescribed analgesics to relieve the pain. c. Loosen the compression bandage to decrease incisional pressure. d. Remind the patient that this phantom pain will diminish over time.

ANS: B Phantom limb pain is treated like any other type of postoperative pain would be treated. Explanations of the reason for the pain may be given, but the nurse should still medicate the patient. The compression bandage is left in place except during physical therapy or bathing. Although the pain may decrease over time, it still requires treatment now.

Which nursing action will the nurse include in the plan of care for a patient who has had a total knee arthroplasty? a. Avoid extension of the knee beyond 120 degrees. b. Use a compression bandage to keep the knee flexed. c. Start progressive knee exercises to obtain 90-degree flexion. d. Teach about the need to avoid weight bearing for 4 weeks.

ANS: C After knee arthroplasty, active or passive flexion exercises are used to obtain a 90-degree flexion of the knee. The goal for extension of the knee will be 180 degrees. A compression bandage is used to hold the knee in an extended position after surgery. Full weight bearing is expected before discharge

A patient in the dermatology clinic is scheduled for removal of a 15-mm multicolored and irregular mole from the upper back. The nurse should prepare the patient for which type of biopsy? a. Shave biopsy b. Punch biopsy c. Incisional biopsy d. Excisional biopsy

ANS: C An incisional biopsy would remove the entire mole and the tissue borders. The appearance of the mole indicates that it may be malignant. A shave biopsy would not remove the entire mole. The mole is too large to be removed with punch biopsy. Excisional biopsies are done for smaller lesions and where a good cosmetic effect is desired, such as on the face

After a patient with a left femur fracture has a hip spica cast applied, which nursing intervention will be included in the plan of care? a. Avoid placing the patient in the prone position. b. Use the cast support bar to reposition the patient. c. Ask the patient about any abdominal discomfort or nausea. d. Discuss the reasons for remaining on bed rest for several weeks.

ANS: C Assessment of bowel tones, abdominal pain, and nausea and vomiting will detect the development of cast syndrome. To avoid breakage, the support bar should not be used for repositioning. After the cast dries, the patient can begin ambulating with the assistance of physical therapy personnel and may be turned to the prone position.

7. When admitting a patient with a stroke who is unconscious and unresponsive to stimuli, the nurse learns from the patient's family that the patient has a history of gastroesophageal reflux disease (GERD). The nurse will plan to do frequent assessments of the patient's a. apical pulse. b. bowel sounds. c. breath sounds. d. abdominal girth.

ANS: C Because GERD may cause aspiration, the unconscious patient is at risk for developing aspiration pneumonia. Bowel sounds, abdominal girth, and apical pulse will not be affected by the patient's stroke or GERD and do not require more frequent monitoring than the routine.

A 19-year-old patient calls the school clinic and tells the nurse, "My menstrual period is very heavy this time. I have to change my tampon every 4 hours." Which action should the nurse take next? a. Tell the patient that her flow is not unusually heavy. b. Schedule the patient for an appointment later that day. c. Ask the patient how heavy her usual menstrual flow is. d. Have the patient call again if the heavy flow continues.

ANS: C Because a heavy menstrual flow is usually indicated by saturating a pad or tampon in 1 to 2 hours, the nurse should first assess how heavy the patient's usual flow is. There is no need to schedule the patient for an appointment that day. The patient may need to call again, but this is not the first action that the nurse should take. Telling the patient that she does not have a heavy flow implies that the patient's concern is not important

The nurse receives change-of-shift report on the oncology unit. Which patient should the nurse assess first? a. 35-year-old patient who has wet desquamation associated with abdominal radiation b. 42-year-old patient who is sobbing after receiving a new diagnosis of ovarian cancer c. 24-year-old patient who received neck radiation and has blood oozing from the neck d. 56-year-old patient who developed a new pericardial friction rub after chest radiation

ANS: C Because neck bleeding may indicate possible carotid artery rupture in a patient who is receiving radiation to the neck, this patient should be seen first. The diagnoses and clinical manifestations for the other patients are not immediately life threatening.

The following patients call the outpatient clinic. Which phone call should the nurse return first? a. A 44-year-old patient who has bloody discharge after a hysteroscopy earlier today b. A 64-year-old patient who is experiencing shoulder pain after a laparoscopy yesterday c. A 34-year-old patient who is short of breath after pelvic computed tomography (CT) with contrast d. A 54-year-old patient who has severe breast tenderness following a needle aspiration breast biopsy

ANS: C The patient's dyspnea suggests a delayed reaction to the iodine dye used for the CT scan. The other patient's symptoms are not unusual after the procedures they had done.

37. After receiving change-of-shift report, which patient should the nurse assess first? a. A patient who was admitted yesterday with gastrointestinal (GI) bleeding and has melena b. A patient who is crying after receiving a diagnosis of esophageal cancer c. A patient with esophageal varices who has a blood pressure of 96/54 mm Hg d. A patient with nausea who has a dose of metoclopramide (Reglan) scheduled

ANS: C The patient's history and blood pressure indicate possible hemodynamic instability caused by GI bleeding. The data about the other patients do not indicate acutely life-threatening complications.

The second day after admission with a fractured pelvis, a 64-year-old patient suddenly develops confusion. Which action should the nurse take first? a. Take the blood pressure. b. Assess patient orientation. c. Check the oxygen saturation. d. Observe for facial asymmetry.

ANS: C The patient's history and clinical manifestations suggest a fat embolus. The most important assessment is oxygenation. The other actions are also appropriate but will be done after the nurse assesses gas exchange

38. Which of these assessment findings in a patient with a hiatal hernia who returned from a laparoscopic Nissen fundoplication 4 hours ago is most important for the nurse to address immediately? a. The patient is experiencing intermittent waves of nausea. b. The patient has absent breath sounds throughout the left lung. c. The patient has decreased bowel sounds in all four quadrants. d. The patient complains of 6/10 (0 to 10 scale) abdominal pain.

ANS: B Decreased breath sounds on one side may indicate a pneumothorax, which requires rapid diagnosis and treatment. The abdominal pain and nausea also should be addressed but they are not as high priority as the patient's respiratory status. The patient's decreased bowel sounds are expected after surgery and require ongoing monitoring but no other action.

A 22-year-old patient reports her concern about not having a menstrual period for the past 7 months. Which statement by the patient indicates a possible related factor to the amenorrhea? a. "I drink at least 3 glasses of nonfat milk every day." b. "I run 7 to 8 miles every day to keep my weight down." c. "I was treated for a sexually transmitted infection 2 years ago." d. "I am not sexually active but currently I have an IUD."

ANS: B Excessive exercise can cause amenorrhea. The other statements by the patient do not suggest any urgent teaching needs

22. The family member of a patient who has suffered massive abdominal trauma in an automobile accident asks the nurse why the patient is receiving famotidine (Pepcid). The nurse will explain that the medication will a. prevent aspiration of gastric contents. b. inhibit the development of stress ulcers. c. lower the chance for H. pylori infection. d. decrease the risk for nausea and vomiting.

ANS: B Famotidine is administered to prevent the development of physiologic stress ulcers, which are associated with a major physiologic insult such as massive trauma. Famotidine does not decrease nausea or vomiting, prevent aspiration, or prevent H. pylori infection.

The home health nurse cares for a patient who has been receiving interferon therapy for treatment of cancer. Which statement by the patient indicates a need for further assessment? a. "I have frequent muscle aches and pains." b. "I rarely have the energy to get out of bed." c. "I experience chills after I inject the interferon." d. "I take acetaminophen (Tylenol) every 4 hours."

ANS: B Fatigue can be a dose-limiting toxicity for use of biologic therapies. Flulike symptoms, such as muscle aches and chills, are common side effects with interferon use. Patients are advised to use acetaminophen every 4 hours.

A couple is scheduled to have a Huhner test for infertility. In preparation for the test, the nurse will instruct the couple about a. being sedated during the procedure. b. determining the estimated time of ovulation. c. experiencing shoulder pain after the procedure. d. refraining from intercourse before the appointment.

ANS: B For the Huhner test, the couple should have intercourse at the estimated time of ovulation and then arrive for the test 2 to 8 hours after intercourse. The other instructions would be used for other types of fertility testing

3. A nurse is caring for a client with suspected acute renal failure who is to undergo a renal biopsy. Which of the following positions illustrated below should the nurse assist the client into? -A -B -C -D

B

39. The home health nurse is doing a follow-up visit to a patient with recently diagnosed rheumatoid arthritis (RA). Which assessment made by the nurse indicates that more patient teaching is needed? a. The patient sleeps with two pillows under the head. b. The patient has been taking 16 aspirins daily. c. The patient requires a 2 hour midday nap. d. The patient sits on a stool when preparing meals.

Answer: A Rationale: The joints should be maintained in an extended position to avoid contractures, so patients should use a small, flat pillow for sleeping. The other information is appropriate for a patient with RA and indicates that teaching has been effective. Cognitive Level: Application Text Reference: pp. 1708-1709 Nursing Process: Evaluation NCLEX: Physiological Integrity

15. When helping a patient with rheumatoid arthritis (RA) plan a daily routine, the nurse informs the patient that it is most helpful to start the day with a. a warm bath followed by a short rest. b. a 10-minute routine of isometric exercises. c. stretching exercises to relieve joint stiffness. d. active range-of-motion (ROM) exercises.

Answer: A Rationale: Taking a warm shower or bath is recommended to relieve joint stiffness, which is worse in the morning. Isometric exercises would place stress on joints and would not be recommended. Stretching and ROM should be done later in the day, when joint stiffness is decreased. Cognitive Level: Comprehension Text Reference: p. 1708 Nursing Process: Implementation NCLEX: Physiological Integrity

41. When the nurse is reviewing laboratory results for a patient with systemic lupus erythematosus (SLE), which result is most important to communicate to the health care provider? a. Elevated blood urea nitrogen (BUN) and creatinine b. Positive lupus erythematosus cell prep c. Positive antinuclear antibodies (ANA) d. Decreased C-reactive protein (CRP)

Answer: A Rationale: The elevated BUN and creatinine levels indicate possible lupus nephritis and a need for a change in therapy to avoid further renal damage. The positive LE cell prep and ANA would be expected in a patient with SLE. A drop in CRP shows an improvement in the inflammatory process. Cognitive Level: Application Text Reference: p. 1718 Nursing Process: Assessment NCLEX: Physiological Integrity

A 61-year-old patient with suspected bowel obstruction had a nasogastric tube inserted at 4:00 AM. The nurse shares in the morning report that the day shift staff should check the tube for patency at what times? A. 7:00 AM, 10:00 AM, and 1:00 PM B. 8:00 AM, 12:00 PM, and 4:00 PM C. 9:00 AM and 3:00 PM D. 9:00 AM, 12:00 PM, and 3:00 PM

B A nasogastric tube should be checked for patency routinely at 4-hour intervals. Thus if the tube were inserted at 4:00 AM, it would be due to be checked at 8:00 AM, 12:00 PM, and 4:00 PM.

The nurse should administer an as-needed dose of magnesium hydroxide (MOM) after noting what information while reviewing a patient's medical record? A. Abdominal pain and bloating B. No bowel movement for 3 days C. A decrease in appetite by 50% over 24 hours D. Muscle tremors and other signs of hypomagnesemia

B MOM is an osmotic laxative that produces a soft, semisolid stool usually within 15 minutes to 3 hours. This medication would benefit the patient who has not had a bowel movement for 3 days. MOM would not be given for abdominal pain and bloating, decreased appetite, or signs of hypomagnesemia.

19. A patient who had arthroscopic surgery of the left knee 5 days previously is admitted with a red, swollen, and hot-to-touch knee. Which of these assessment data obtained by the nurse should be reported to the health care provider immediately? a. The white blood cell count is 14,200/μl. b. The patient rates the knee pain at 9 on a 10-point pain scale. c. The patient has recently taken ibuprofen (Motrin). d. The oral temperature is 104.1° F degrees.

Answer: D Rationale: The elevated temperature suggests that the patient may have an infection and be in danger of developing septicemia as a complication of septic arthritis. Immediate blood cultures and initiation of antibiotic therapy are indicated. The other information is typical of septic arthritis and should also be reported to the health care provider, but it does not indicate any immediately life-threatening problems. Cognitive Level: Application Text Reference: p. 1713 Nursing Process: Assessment NCLEX: Physiological Integrity

7. A nurse is caring for a client who is receiving hemodialysis via the left arteriovenous fistula for management of chronic renal disease. Which of the following teaching points should the nurse reinforce? -Check the site hourly for patency -Maintain skin integrity through frequent cleansing and application of lotion to the site -Avoid tight clothing around the site -Sleep on the left site to maintain patency of access site

Avoid tight clothing around the access site

A 30-year-old patient seen in the emergency department for severe headache and acute confusion is found to have a serum sodium level of 118 mEq/L. The nurse will anticipate the need for which diagnostic test? a. Urinary 17-ketosteroids b. Antidiuretic hormone level c. Growth hormone stimulation test d. Adrenocorticotropic hormone level

B

A 35-year-old female patient with a possible pituitary adenoma is scheduled for a computed tomography (CT) scan with contrast media. Which patient information is most important for the nurse to communicate to the health care provider before the test? a. Bilateral poor peripheral vision b. Allergies to iodine and shellfish c. Recent weight loss of 20 pounds d. Complaint of ongoing headaches

B

A 40-year-old male patient has been newly diagnosed with type 2 diabetes mellitus. Which information about the patient will be most useful to the nurse who is helping the patient develop strategies for successful adaptation to this disease? a. Ideal weight b. Value system c. Activity level d. Visual changes

B

A nurse will teach a patient who is scheduled to complete a 24-hour urine collection for 17-ketosteroids to a. insert and maintain a retention catheter. b. keep the specimen refrigerated or on ice. c. drink at least 3 L of fluid during the 24 hours. d. void and save that specimen to start the collection.

B

An age-related finding noted by the nurse during assessment of the older woman's reproductive system is: A. dyspareunia B. vaginal dryness C. nipple retraction D. increased sensitivity of the labia

B

During the physical examination of a 36-year-old female, the nurse finds that the patient's thyroid gland cannot be palpated. The most appropriate action by the nurse is to a. palpate the patient's neck more deeply. b. document that the thyroid was nonpalpable. c. notify the health care provider immediately. d. teach the patient about thyroid hormone testing.

B

The nurse is caring for a 45-year-old male patient during a water deprivation test. Which finding is most important for the nurse to communicate to the health care provider? a. The patient complains of intense thirst. b. The patient has a 5-lb (2.3 kg) weight loss. c. The patient's urine osmolality does not increase. d. The patient feels dizzy when sitting on the edge of the bed.

B

Which action by a new registered nurse (RN) caring for a patient with a goiter and possible hyperthyroidism indicates that the charge nurse needs to do more teaching? a. The RN checks the blood pressure on both arms. b. The RN palpates the neck thoroughly to check thyroid size. c. The RN lowers the thermostat to decrease the temperature in the room. d. The RN orders nonmedicated eye drops to lubricate the patient's bulging eyes.

B

Which information about a 30-year-old patient who is scheduled for an oral glucose tolerance test should be reported to the health care provider before starting the test? a. The patient reports having occasional orthostatic dizziness. b. The patient takes oral corticosteroids for rheumatoid arthritis. c. The patient has had a 10-pound weight gain in the last month. d. The patient drank several glasses of water an hour previously.

B

A colectomy is scheduled for a 38-year-old woman with ulcerative colitis. The nurse should plan to include what prescribed measure in the preoperative preparation of this patient? A. Instruction on irrigating a colostomy B. Administration of a cleansing enema C. A high-fiber diet the day before surgery D. Administration of IV antibiotics for bowel preparation

B Preoperative preparation for bowel surgery typically includes bowel cleansing with antibiotics, such as oral neomycin and cleansing enemas, including Fleet enemas. Instructions to irrigate the colostomy will be done postoperatively. Oral antibiotics are given preoperatively, and an IV antibiotic may be used in the OR. A clear liquid diet will be used the day before surgery with the bowel cleansing.

Which of the following assessment findings of a 70-year-old male patient's skin should the nurse prioritize? A) The patient's complaint of dry skin that is frequently itchy. B) The presence of an irregularly shaped mole that the patient states is new. C) The presence of veins on the back of the patient's leg that are blue and tortuous. D) The presence of a rash on the patient's hand and forearm to which the patient applies a corticosteroid ointment.

B) The presence of an irregularly shaped mole that the patient states is new. Although all of the noted assessment findings are significant, the presence of an irregular mole that is new is suggestive of a neoplasm and warrants immediate follow-up.

During assessment of a patient, you note an area of red, sharply defined plaques covered with silvery scales that are mildly itchy on the patient's knees and elbows. You recognize this finding as: A) lentigo B) psoriasis C) actinic keratosis D) seborrheic keratosis

B) psoriasis

In teaching a patient who is using topical corticosteroids to treat acute dermatitis, the nurse should tell the patient that (select all that apply) A) the cream form is the most efficient system of delivery B) short-term use of topical corticosteroids usually does not cause systemic side effects C) creams and ointments should be applied with a glove in a small amounts to prevent further infection D) abruptly discontinuing the use of topical corticosteroids may cause a reappearance of the detmatitis E) systemic side effects may be experienced from topical corticosteroids if the person is malnourished.

B) short-term use of topical corticosteroids usually does not cause systemic side effects D) abruptly discontinuing the use of topical corticosteroids may cause a reappearance of the detmatitis

The nurse is caring for a group of older patients in a long-term care setting. Which physical changes in the patients should the nurse investigate as signs of possible endocrine dysfunction? A. Absent reflexes, diarrhea, and hearing loss B. Hypoglycemia, delirium, and incontinence Incorrect C. Fatigue, constipation, and mental impairment Correct D. Hypotension, heat intolerance, and bradycardia

B. Hypoglycemia, delirium, and incontinence Changes of aging often mimic clinical manifestations of endocrine disorders. Clinical manifestations of endocrine dysfunction such as fatigue, constipation, or mental impairment in the older adult are often missed because they are attributed solely to aging.

34. The health care provider orders oral glucose tolerance testing for a patient seen in the clinic. Which information from the patient's health history is most important for the nurse to communicate to the health care provider? a. The patient had a viral illness 2 months ago. b. The patient uses oral contraceptives. c. The patient runs several days a week. d. The patient has a family history of diabetes.

B. The patient uses oral contraceptives Rationale: Oral contraceptive use may falsely elevate oral glucose tolerance test (OGTT) values. A viral 2 months previously illness may be associated with the onset of type 1 diabetes but will not falsely impact the OGTT. Exercise and a family history of diabetes both can affect blood glucose but will not lead to misleading information from the OGTT.

A 29-year-old patient in the outpatient clinic will be scheduled for blood cortisol testing. Which instruction will the nurse provide? a. "Avoid adding any salt to your foods for 24 hours before the test." b. "You will need to lie down for 30 minutes before the blood is drawn." c. "Come to the laboratory to have the blood drawn early in the morning." d. "Do not have anything to eat or drink before the blood test is obtained."

C

A 44-year-old patient is admitted with tetany. Which laboratory value should the nurse monitor? a. Total protein b. Blood glucose c. Ionized calcium d. Serum phosphate

C

The nurse reviews a patient's glycosylated hemoglobin (Hb A1C) results to evaluate a. fasting preprandial glucose levels. b. glucose levels 2 hours after a meal. c. glucose control over the past 90 days. d. hypoglycemic episodes in the past 3 months.

C

The nurse will teach a patient to plan to minimize physical and emotional stress while the patient is undergoing a. a water deprivation test. b. testing for serum T3 and T4 levels. c. a 24-hour urine test for free cortisol. d. a radioactive iodine (I-131) uptake test.

C

Which additional information will the nurse need to consider when reviewing the laboratory results for a patient's total calcium level? a. The blood glucose is elevated. b. The phosphate level is normal. c. The serum albumin level is low. d. The magnesium level is normal.

C

Which laboratory value should the nurse review to determine whether a patient's hypothyroidism is caused by a problem with the anterior pituitary gland or with the thyroid gland? a. Thyroxine (T4) level b. Triiodothyronine (T3) level c. Thyroid-stimulating hormone (TSH) level d. Thyrotropin-releasing hormone (TRH) level

C

Which question will provide the most useful information to a nurse who is interviewing a patient about a possible thyroid disorder? a. "What methods do you use to help cope with stress?" b. "Have you experienced any blurring or double vision?" c. "Have you had a recent unplanned weight gain or loss?" d. "Do you have to get up at night to empty your bladder?"

C

After teaching a patient with interstitial cystitis about management of the condition, the nurse determines that further instruction is needed when the patient says, a. "I will have to stop having coffee and orange juice for breakfast." b. "I should start taking a high potency multiple vitamin every morning." c. "I will buy some calcium glycerophosphate (Prelief) at the pharmacy." d. "I should call the doctor about increased bladder pain or odorous urine."

B. "I should start taking a high potency multiple vitamin every morning" High-potency multiple vitamins may irritate the bladder and increase symptoms. The other patient statements indicate good understanding of the teaching.

After the home health nurse teaches a patient with a neurogenic bladder how to use intermittent catheterization for bladder emptying, which patient statement indicates that the teaching has been effective? a. "I will use a sterile catheter and gloves for each time I self-catheterize." b. "I will clean the catheter carefully before and after each catheterization." c. "I will need to buy seven new catheters weekly and use a new one every day." d. "I will need to take prophylactic antibiotics to prevent any urinary tract infections."

B. "I will clean the catheter carefully before and after each catheterization" Patients who are at home can use a clean technique for intermittent self-catheterization and change the catheter every 7 days. There is no need to use a new catheter every day, to use sterile catheters, or to take prophylactic antibiotics.

31. A newly diagnosed type 1 diabetic patient likes to run 3 miles several mornings a week. Which teaching will the nurse implement about exercise for this patient? a. "You should not take the morning NPH insulin before you run." b. "Plan to eat breakfast about an hour before your run." c. "Afternoon running is less likely to cause hypoglycemia." d. "You may want to run a little farther if your glucose is very high."

B. "Plan to eat breakfast about an hour before you run" Rationale: Blood sugar increases after meals, so this will be the best time to exercise. NPH insulin will not peak until mid-afternoon and is safe to take before a morning run. Running can be done in either the morning or afternoon. If the glucose is very elevated, the patient should postpone the run.

During a health screening event which assessment finding would alert the nurse to the possible presence of osteoporosis in a white 61-year-old female? A. The presence of bowed legs B. A measurable loss of height C. Poor appetite and aversion to dairy products D. Development of unstable, wide-gait ambulation

B. A measurable loss of height A gradual but measurable loss of height and the development of kyphosis or "dowager's hump" are indicative of the presence of osteoporosis in which the rate of bone resorption is greater than bone deposition. Bowed legs may be caused by abnormal bone development or rickets but is not indicative of osteoporosis. Lack of calcium and Vitamin D intake may cause osteoporosis but are not indicative it is present. A wide gait is used to support balance and does not indicate osteoporosis.

The nurse is teaching a wellness class to a group of women at their workplace. The nurse knows that which woman is at highest risk for developing cancer? A. A woman who obtains regular cancer screenings and consumes a high-fiber diet B. A woman who has a body mass index of 35 kg/m2 and smoked cigarettes for 20 years C. A woman who exercises five times every week and does not consume alcoholic beverages D. A woman who limits fat consumption and has regular mammography and Pap screenings

B. A woman who has a body mass index of 35 kg/m2 and smoked cigarettes for 20 years Cancer prevention and early detection are associated with the following behaviors: limited alcohol use; regular physical activity; maintaining a normal body weight; obtaining regular cancer screenings; avoiding cigarette smoking and other tobacco use; using sunscreen with SPF 15 or higher; and practicing healthy dietary habits (e.g., reduced fat and increased fruits and vegetables).

The screening criteria for assessing prostate cancer include a: A. baseline ultrasound of the prostate at age 40 B. baseline ultrasound of the prostate at age 50 C. yearly DRE for men age 30 and older D. yearly DRE for men age 50 and older

D

Which statement by a 50-year-old female patient indicates to the nurse that further assessment of thyroid function may be necessary? a. "I notice my breasts are tender lately." b. "I am so thirsty that I drink all day long." c. "I get up several times at night to urinate." d. "I feel a lump in my throat when I swallow."

D

The nurse would question the use of which cathartic agent in a patient with renal insufficiency? A. Bisacodyl (Dulcolax) B. Lubiprostone (Amitiza) C. Cascara sagrada (Senekot) D. Magnesium hydroxide (Milk of Magnesia)

D Milk of Magnesia may cause hypermagnesemia in patients with renal insufficiency. The nurse should question this order with the health care provider. Bisacodyl, lubiprostone, and cascara sagrada are safe to use in patients with renal insufficiency as long as the patient is not currently dehydrated.

The nurse is caring for a 45-year-old woman with a herniated lumbar disc. The patient realizes that weight loss is necessary to lessen back strain. The patient is 5'6" tall and weighs 186 lb (84.5 kg) with a body mass index (BMI) of 28 kg/m2. The nurse explains to the patient that this measurement places her in which of the following weight categories? a. Normal weight b. Overweight c. Obese d. Severely obese

Correct answer: b Rationale: A normal BMI is 18.5 to 24.9 kg/m2, whereas a BMI of 25 to 29.9 kg/m2 is considered overweight. A BMI of 30.0-39.9 is considered obese, and a BMI of 40 or greater is severely obese.

A woman is 5 ft, 6 in (166 cm) tall and weighs 200 lb (90.9 kg) with a waist-to-hip ratio of 0.7. The nurse counsels the patient with the knowledge that the patient is at greatest risk for a. heart disease. b. osteoporosis. c. diabetes mellitus. d. endometrial cancer.

Correct answer: b Rationale: A patient who is obese (BMI of 32.2) but has a waist-to-hip ratio of less than 0.8, indicating gynoid obesity, has an increased risk for osteoporosis. The other conditions are risks associated with android obesity.

The patient with systemic lupus erythematosus had been diagnosed with syndrome of inappropriate antidiuretic hormone (SIADH). What should the nurse expect to include in this patient's plan of care (select all that apply)? A. Obtain weekly weights. B. Limit fluids to 1000 mL per day.* C. Monitor for signs of hypernatremia. D. Minimize turning and range of motion. E. Keep the head of the bed at 10 degrees or less elevation.

B. Limit fluids to 1000 mL per day.* E. Keep the head of the bed at 10 degrees or less elevation. The care for the patient with SIADH will include limiting fluids to 1000 mL per day or less to decrease weight, increase osmolality, and improve symptoms; and keeping the head of the bed elevated at 10 degrees or less to enhance venous return to the heart and increase left atrial filling pressure, thereby reducing the release of ADH. The weights should be done daily along with intake and output. Signs of hyponatremia should be monitored, and frequent turning, positioning, and range-of-motion exercises are important to maintain skin integrity and joint mobility.

The nurse is caring for a patient admitted to the nursing unit with osteomyelitis of the tibia. Which symptom will the nurse most likely find on physical examination of the patient? A. Nausea and vomiting B. Localized pain and warmth C. Paresthesia in the affected extremity D. Generalized bone pain throughout the leg

B. Localized pain and warmth Osteomyelitis is an infection of bone and bone marrow that can occur with trauma, surgery, or spread from another part of the body. Because it is an infection, the patient will exhibit typical signs of inflammation and infection, including localized pain and warmth. Nausea and vomiting and paresthesia of the extremity are not expected to occur. Pain occurs, but it is localized, not generalized throughout the leg.

5. A pregnant woman with a history of early chronic HIV infection is seen at the clinic. Which information will the nurse include when teaching the patient? a. The antiretroviral medications used to treat HIV infection are teratogenic. b. Most infants born to HIV-positive mothers are not infected with the virus. c. Since she is at an early stage of HIV infection, the infant will not contract HIV. d. It is likely that her newborn will become infected with HIV unless she uses antiretroviral drug therapy (ART).

B. Most infants born to HIV-positive mothers are not infected with the virus Only 25% of infants born to HIV-positive mothers develop HIV infection, even when the mother does not use ART during pregnancy. The percentage drops to 2% when ART is used. Perinatal transmission can occur at any stage of HIV infection (although it is less likely to occur when the viral load is lower). ART can safely be used in pregnancy, although some ART drugs should be avoided.

The nurse interviews a 50-year-old man with a history of type 2 diabetes mellitus, chronic bronchitis, and osteoarthritis who has a fasting blood glucose of 154 mg/dL. Which medications, if taken by the patient, may raise blood glucose levels? A. Glargine (Lantus) Incorrect B. Prednisone (Deltasone) Correct C. Metformin (Glucophage) D. Acetaminophen (Tylenol)

B. Prednisone (Deltasone) Prednisone is a corticosteroid that may cause glucose intolerance in susceptible patients by increasing gluconeogenesis and insulin resistance. Insulin (e.g., glargine) and metformin (an oral hypoglycemic agent) decrease blood glucose levels. Acetaminophen has a glucose-lowering effect

Polydipsia and polyuria related to diabetes mellitus are primarily due to? a) the release of ketones from cells during fat metabolism b) fluid shifts resulting from the osmotic effect of hyperglycemia c) damage to the kidneys from exposure to high levels of glucose d) changes in RBC's resulting from attachment of excessive glucose to hemoglobin

B. fluid shifts resulting from the osmotic effect of hyperglycemia

4. A nurse is caring for a client who has undergone a non-related living donor kidney transplant. On the fifth postoperative day, the nurse notes that the client has gained 1 kg of body weight since the previous day. The nurse suspects rejection. Which of the following would also be seen in a client experiencing rejection? -Serum creatinine 0.8 mg/dL -BP of 160/90 mmHg -Pain of 4 out of 10 at the surgical incision -Urinary output of 100 mL/ hour

BP of 160/90 mmHg

A 60-year-old patient is taking spironolactone (Aldactone), a drug that blocks the action of aldosterone on the kidney, for hypertension. The nurse will monitor for a. increased serum sodium. b. decreased urinary output. c. elevated serum potassium. d. evidence of fluid overload.

C

A normal reproductive function that may be altered in a patient who undergoes a prostatectomy is: A. sperm production B. production of testosterone C. production of seminal fluid D. release of sperm from the epididymis

C

An abnormal finding noted during physical assessment of the male reproductive system is: A. descended testes B. symmetric scrotum C. slight clear urethral discharge D. the glans covered with a prepuce

C

Estrogen production by the mature ovarian follicle causes: A. decreased secretion of FSH and LH B. increased production of GnRH and FSH C. release of GnRH and increased secretion of LH D. decreased release of FSH and decreased progesterone production

C

Significant information about a patient's past health history related to the reproductive system should include: A. extent of sexual activity B. general satisfaction with sexuality C. previous STDs D. self image and relationship with others

C

A 50-year-old female patient smokes, is getting a divorce, and is reporting eye problems. On assessment of this patient, the nurse notes exophthalmos. What other abnormal assessments should the nurse expect to find in this patient? A. Puffy face, decreased sweating, and dry hair B. Muscle aches and pains and slow movements C. Decreased appetite, increased thirst, and pallor D. Systolic hypertension and increased heart rate

D. Systolic hypertension and increased heart rate The patient's manifestations point to Graves' disease or hyperthyroidism, which would also include systolic hypertension and increased heart rate and increased thirst. Puffy face, decreased sweating; dry, coarse hair; muscle aches and pains and slow movements; decreased appetite and pallor are all manifestations of hypothyroidism.

7. A 20-year-old female patient who is HIV-positive has a new prescription for efavirenz (Sustiva). Which information about the patient is most important to communicate to the prescribing physician before administering the efavirenz? a. The patient's CD4+ T cell count is 800 cells/μL. b. The patient already has etravirine (Intelence) prescribed. c. The patient states that the antiretroviral therapy (ART) frequently cause nausea. d. The patient is sexually active and does not use any contraception.

D. The patient is sexually active and does not use any contraception Efavirenz can cause fetal anomalies and should not be used in patients who may be pregnant. The patient's nausea, use of another nucleoside reverse transcriptase inhibitor, and CD4+ count also should be communicated to the physician, but the most important information is that the patient may potentially be pregnant.

18. Which information about an HIV-positive patient who is taking antiretroviral medications is most important for the nurse to address when planning care? a. The patient's blood glucose level is 168 mg/dl. b. The patient complains of feeling "constantly tired." c. The patient is unable to state the side effects of the medications. d. The patient states "sometimes I miss a dose of zidovudine (AZT)."

D. The patient states "Sometimes I miss a dose of zidovudine (AZT)." Since missing doses of ART can lead to drug resistance, this patient statement indicates the need for interventions such as teaching or changes in the drug scheduling. Elevated blood glucose and fatigue are common side effects of ART. The nurse should discuss medication side effects with the patient, but this is not as important as addressing the skipped doses of AZT.

The patient is being treated with brachytherapy for cervical cancer. What factors must the nurse be aware of to protect herself when caring for this patient? A. The medications the patient is taking B. The nutritional supplements that will help the patient C. How much time is needed to provide the patient's care D. The time the nurse spends at what distance from the patient

D. The time the nurse spends at what distance from the patient The principles of ALARA (as low as reasonably achievable) and time, distance, and shielding are essential to maintain the nurse's safety when the patient is a source of internal radiation. The patient's medications, nutritional supplements, and time needed to complete care will not protect the nurse caring for a patient with brachytherapy for cervical cancer.

15. A patient who has been treated for HIV infection for 7 years has developed fat redistribution to the trunk, with wasting of the arms, legs, and face. The nurse will anticipate teaching the patient about a. the benefits of daily exercise. b. foods that are higher in protein. c. treatment with antifungal agents. d. a change in antiretroviral therapy.

D. a change in antiretroviral therapy A frequent first intervention for metabolic disorders is a change in ART. Treatment with antifungal agents would not be appropriate because there is no indication of fungal infection. Changes in diet or exercise have not proven helpful for this problem.

27. A patient recovering from DKA asks the nurse how acidosis occurs. The best response by the nurse is that a. insufficient insulin leads to cellular starvation, and as cells rupture they release organic acids into the blood. b. when an insulin deficit causes hyperglycemia, then proteins are deaminated by the liver, causing acidic by-products. c. excess glucose in the blood is metabolized by the liver into acetone, which is acidic. d. an insulin deficit promotes metabolism of fat stores, which produces large amounts of acidic ketones.

D. an insulin deficit promotes metabolism of fat stores, which produces large amounts of acidic ketones. Rationale: Ketoacidosis is caused by the breakdown of fat stores when glucose is not available for intracellular metabolism. The other responses are inaccurate.

A 78-year-old who has been admitted to the hospital with dehydration is confused and incontinent of urine. Which nursing action will be best to include in the plan of care? a. Apply absorbent incontinent pads. b. Restrict fluids after the evening meal. c. Insert an indwelling catheter until the symptoms have resolved. d. Assist the patient to the bathroom every 2 hours during the day.

D. assist the patient to the bathroom every 2 hours during the day In older or confused patients, incontinence may be avoided by using scheduled toileting times. Indwelling catheters increase the risk for urinary tract infection (UTI). Incontinent pads increase the risk for skin breakdown. Restricting fluids is not appropriate in a patient with dehydration.

Which action will the nurse include in the plan of care for a patient who has had a ureterolithotomy and has a left ureteral catheter and a urethral catheter in place? a. Provide education about home care for both catheters. b. Apply continuous steady tension to the ureteral catheter. c. Clamp the ureteral catheter unless output from the urethral catheter stops. d. Call the health care provider if the ureteral catheter output drops suddenly.

D. call the health care provider if the ureteral catheter output drops suddenly The health care provider should be notified if the ureteral catheter output decreases since obstruction of this catheter may result in an increase in pressure in the renal pelvis. Tension on the ureteral catheter should be avoided in order to prevent catheter displacement. To avoid pressure in the renal pelvis, the catheter is not clamped. Since the patient is not usually discharged with a ureteral catheter in place, patient teaching about both catheters is not needed.

The nurse observes nursing assistive personnel (NAP) taking the following actions when caring for a patient with a retention catheter. Which action requires that the nurse intervene? a. Taping the catheter to the skin on the patient's upper inner thigh b. Cleaning around the patient's urinary meatus with soap and water c. Using an alcohol-based hand cleaner before performing catheter care d. Disconnecting the catheter from the drainage tube to obtain a specimen

D. disconnecting the catheter form the drainage tube to obtain a specimen The catheter should not be disconnected from the drainage tube because this increases the risk for urinary tract infection (UTI). The other actions are appropriate and do not require any intervention.

When the nurse is caring for a patient who has had left-sided extracorporeal shock wave lithotripsy, which assessment finding is most important to report to the health care provider? a. Blood in urine b. Left flank pain c. Left flank bruising d. Drop in urine output

D. drop in urine output Because lithotripsy breaks the stone into small sand, which could cause obstruction, it is important to report a drop in urine output. Left flank pain, bruising, and hematuria are common after lithotripsy.

An 18-year-old male patient is undergoing a growth hormone stimulation test. The nurse should monitor the patient for A. hypothermia. Incorrect B. hypertension. C. hyperreflexia. D. hypoglycemia.

D. hypoglycemia Insulin or arginine (agent that stimulates insulin secretion) is administered for a growth hormone stimulation test. The nurse should monitor the patient closely for hypoglycemia. Hypothermia and hypertension are not expected in response to insulin or arginine. Hyperreflexia is an autonomic complication of spinal cord injury.

2. A patient is admitted to the hospital with Pneumocystis jiroveci pneumonia (PCP) and HIV testing is positive. Based on diagnostic criteria established by the Centers for Disease Control and Prevention (CDC), the patient is diagnosed as having a. acute infection. b. early chronic infection. c. intermediate chronic infection. d. late chronic infection or AIDS.

D. late chronic infection or AIDS Development of PCP pneumonia meets the diagnostic criterion for AIDS. The other responses indicate an earlier stage of HIV infection than is indicated by the PCP infection.

14. A patient with HIV infection has developed Mycobacterium avium complex infection. An appropriate outcome for the patient is that the patient will a. be free from injury. b. receive immunizations. c. have adequate oxygenation. d. maintain intact perineal skin.

D. maintain intact perineal skin The major manifestation of M. avium infection is loose, watery stools, which would increase the risk for perineal skin breakdown. The other outcomes would be appropriate for other complications (pneumonia, dementia, influenza, etc.) associated with HIV infection.

When assessing the patient who has a lower urinary tract infection (UTI), the nurse will initially ask about a. nausea. b. flank pain. c. poor urine output. d. pain with urination.

D. pain with urination Pain with urination is a common symptom of a lower UTI. Urine output does not decrease, but frequency may be experienced. Flank pain and nausea are associated with an upper UTI.

19. A hospitalized diabetic patient receives 12 U of regular insulin mixed with 34 U of NPH insulin at 7:00 AM. The patient is away from the nursing unit for diagnostic testing at noon, when lunch trays are distributed. The most appropriate action by the nurse is to a. save the lunch tray to be provided upon the patient's return to the unit. b. call the diagnostic testing area and ask that a 5% dextrose IV be started. c. ensure that the patient drinks a glass of milk or orange juice at noon in the diagnostic testing area. d. request that the patient be returned to the unit to eat lunch if testing will not be completed promptly.

D. request that the patient be returned to the unit to eat lunch if the testing will not be completed promptly. Rationale: Consistency for mealtimes assists with regulation of blood glucose, so the best option is for the patient to have lunch at the usual time. Waiting to eat until after the procedure is likely to cause hypoglycemia. Administration of an IV solution is unnecessarily invasive for the patient. A glass of milk or juice will keep the patient from becoming hypoglycemic but will cause a rapid rise in blood glucose because of the rapid absorption of the simple carbohydrate in these items.

A 62-year-old asks the nurse for a perineal pad, stating that laughing or coughing causes leakage of urine. Which intervention is most appropriate to include in the care plan? a. Assist the patient to the bathroom q3hr. b. Place a commode at the patient's bedside. c. Demonstrate how to perform the Credé maneuver. d. Teach the patient how to perform Kegel exercises.

D. teach the patient how to perform Kegel exercises Exercises to strengthen the pelvic floor muscles will help reduce stress incontinence. The Credé maneuver is used to help empty the bladder for patients with overflow incontinence. Placing the commode close to the bedside and assisting the patient to the bathroom are helpful for functional incontinence.

Which statement would be correct for a patient with type 2 diabetes who was admitted to the hospital with pneumonia? a) The patient must receive insulin therapy to prevent ketoacidosis b) The patient has islet cell antibodies that have destroyed the pancreas's ability to produce insulin c) The patient has minimal or absent endogenous insulin secretion and requires daily insulin injections d) The patient may have sufficient endogenous insulin to prevent ketosis but is at risk for hyperosmolar hyperglycemic syndrome

D. the patient may have sufficient endogenous insulin to prevent ketosis but is at risk for hyperosmolar hyperglycemic syndrome

The nurse identifies a nursing diagnosis of pain related to muscle spasms for a 45-year-old patient who has low back pain from a herniated lumbar disc. What would be an appropriate nursing intervention to treat this problem? A. Provide gentle ROM to the lower extremities. B. Elevate the head of the bed 20 degrees and flex the knees. C. Place the bed in reverse Trendelenburg with the feet firmly against the footboard. D. Place a small pillow under the patient's upper back to gently flex the lumbar spine.

B. Elevate the head of the bed 20 degrees and flex the knees. The nurse should elevate the head of the bed 20 degrees and flex the knees to avoid extension of the spine and increasing the pain. The slight flexion provided by this position often is comfortable for a patient with a herniated lumbar disc. ROM to the lower extremities will be limited to prevent extremes of spinal movement. Reverse Trendelenburg and a pillow under the patient's upper back will more likely increase pain.

b. Seasonal allergic rhinitis is most commonly caused by pollens from trees, weeds, and grasses. Airborne allergies can be controlled by sleeping in an air-conditioned room, daily damp dusting, covering the mattress and pillows with hypoallergenic covers, and wearing a mask outdoors.

Which of the following statements made by the nurse is most appropriate in teaching patient interventions to minimize the effects of seasonal allergic rhinitis? A. "You will need to get rid of your pets." B. "You should sleep in an air-conditioned room." C. "You would do best to stay indoors during the winter months." D. "You will need to dust your house with a dry feather duster twice a week."

When caring for the patient with a traumatic brain injury (TBI), the nurse knows that damage to which endocrine gland can affect the hormone secretion from some of the other endocrine glands? ANTERIOR PITUITARY ADRENAL HYPOTHALAMUS PARATHYROID

With a TBI, the anterior pituitary is likely to be damaged. The anterior pituitary gland secrets tropic hormones that control the secretion of hormones by other endocrine glands (the thyroid, adrenal cortex, and reproductive organs). The parathyroids secrete parathyroid hormone that regulates serum calcium level by acting on bone, the kidneys, and indirectly the gastrointestinal tract. The pineal gland secretes melatonin that helps regulate circadian rhythm and reproduction. The thyroid glands secrete thyroxine (T4), triiodothyronine (T3) that regulates the cell processes of cell growth and tissue differentiation, and calcitonin that affects bone tissue to regulate serum calcium and phosphorus levels.

The patient with CKD is receiving dialysis, and the nurse observes excoriations on the patient's skin. What pathophysiologic changes in CKD can contribute to this finding (select all that apply)? a. Dry skin b. Sensory neuropathy c. Vascular calcifications d. Calcium-phosphate skin deposits e. Uremic crystallization from high BUN

a, b, d. Pruritus is common in patients receiving dialysis. It causes scratching from dry skin, sensory neuropathy, and calcium-phosphate deposition in the skin. Vascular calcifications contribute to cardiovascular disease, not to itching skin. Uremic frost rarely occurs without BUN levels greater than 200 mg/dL, which should not occur in a patient on dialysis; urea crystallizes on the skin and also causes pruritis.

Which complication of chronic kidney disease is treated with erythropoietin (EPO)? a. Anemia b. Hypertension c. Hyperkalemia d. Mineral and bone disorder

a. Erythropoietin is used to treat anemia, as it stimulates the bone marrow to produce red blood cells.

14. Which statement accurately describes Graves' disease? a. Exophthalmos occurs in Graves' disease. b. It is an uncommon form of hyperthyroidism. c. Manifestations of hyperthyroidism occur from tissue desensitization to the sympathetic nervous system. d. Diagnostic testing in the patient with Graves' disease will reveal an increased thyroid-stimulating hormone (TSH) level.

a. Exophthalmos occurs in Graves's disease Exophthalmos or protrusion of the eyeballs may occur in Graves' disease from increased fat deposits and fluid in the orbital tissues and ocular muscles, forcing the eyeballs outward. Graves' disease is the most common form of hyperthyroidism. Increased metabolic rate and sensitivity of the sympathetic nervous system lead to the clinical manifestations. Thyroid-stimulating hormone (TSH) level is decreased in Graves' disease.

Priority Decision: During the immediate postoperative care of a recipient of a kidney transplant, what should the nurse expect to do? a. Regulate fluid intake hourly based on urine output. b. Monitor urine-tinged drainage on abdominal dressing. c. Medicate the patient frequently for incisional flank pain. d. Remove the urinary catheter to evaluate the ureteral implant.

a. Fluid and electrolyte balance is critical in the transplant recipient patient, especially because diuresis often begins soon after surgery. Fluid replacement is adjusted hourly based on kidney function and urine output. Urine-tinged drainage on the abdominal dressing may indicate leakage from the ureter implanted into the bladder and the health care provider should be notified. The donor patient may have a flank or laparoscopic incision(s) where the kidney was removed. The recipient has an abdominal incision where the kidney was placed in the iliac fossa. The urinary catheter is usually used for 2 to 3 days to monitor urine output and kidney function.

20. As a precaution for vocal cord paralysis from damage to the recurrent laryngeal nerve during thyroidectomy surgery, what equipment should be in the room in case it is needed for this emergency situation? a. Tracheostomy tray b. Oxygen equipment c. IV calcium gluconate d. Paper and pencil for communication

a. Tracheostomy Tray A tracheostomy tray is in the room to use if vocal cord paralysis occurs from recurrent laryngeal nerve damage or for laryngeal stridor from tetany. The oxygen equipment may be useful but will not improve oxygenation with vocal cord paralysis without a tracheostomy. IV calcium salts will be used if hypocalcemia occurs from parathyroid damage. The paper and pencil for communication may be helpful, especially if a tracheostomy is performed, but will not aid in emergency oxygenation of the patient.

In a patient with AKI, which laboratory urinalysis result indicates tubular damage? a. Hematuria b. Specific gravity fixed at 1.010 c. Urine sodium of 12 mEq/L (12 mmol/L) d. Osmolality of 1000 mOsm/kg (1000 mmol/kg)

b. A urine specific gravity that is consistently 1.010 and a urine osmolality of about 300 mOsm/kg is the same specific gravity and osmolality as plasma. This indicates that tubules are damaged and unable to concentrate urine. Hematuria is more common with postrenal damage. Tubular damage is associated with a high sodium concentration (greater than 40 mEq/L).

A, D

Female orgasm is the result of (select all that apply): A. uterine and vaginal contractions B. clitoral swelling and increased vaginal lubrication C.vaginal enlargement and uterine elevation secretion D. rapid release of muscular tension in the reproductive structures

b. Genetic carrier screening should be done in families with a history of sickle cell disease. Diabetes and iron-deficiency anemia are not amenable to any form of genetic testing, whereas a family history of breast cancer suggests the need for presymptomatic testing for estimating the patient's risk of developing breast cancer.

For which of the following individuals is genetic carrier screening indicated? A. A patient with a history of type 1 diabetes B. A patient with a family history of sickle cell disease C. A patient whose mother and sister died of breast cancer D. A patient who has a long-standing history of iron-deficiency anemia

18. A nurse is caring for a client who was brought to the emergency department following an accident. The nurse suspects a ruptured bladder. Which of the following findings is consistent with this diagnosis? -Anuria -Hematuria -Pyuria -Fever

Hematuria

16. A nurse is caring for a client who has recurrent kidney stones. The provider prescribes several diagnostic studies, including IVP, urine C&S, and strain all urine. The nurse needs to inquire further if the client states which of the following? -The last time I voided, it was painful and slightly blood-tinged -My last menstrual period was a week ago, and it was normal -I probably drink at least 2 quarts of fluid every single day -I never eat shellfish because it gives me hives.

I never eat shellfish because it gives me hives

15. A nurse is caring for a client with a history of cystitis. Which of the following statements indicates that the client needs further reinforcement of the teaching about the condition? -I try to empty my bladder every 2-3 hours -I drink 2-3 quarts of fluids a day -I prefer to take baths instead of showers -I use an oral contraceptive for birth control

I prefer to take baths instead of showers

The nurse provides discharge instructions for a 64-year-old woman with ascites and peripheral edema related to cirrhosis. Which statement, if made by the patient, indicates teaching was effective? "It is safe to take acetaminophen up to four times a day for pain." "Lactulose (Cephulac) should be taken every day to prevent constipation." "Herbs and other spices should be used to season my foods instead of salt." "I will eat foods high in potassium while taking spironolactone (Aldactone)."

"Herbs and other spices should be used to season my foods instead of salt." A low-sodium diet is indicated for the patient with ascites and edema related to cirrhosis. Table salt is a well-known source of sodium and should be avoided. Alternatives to salt to season foods include the use of seasonings such as garlic, parsley, onion, lemon juice, and spices. Pain medications such as acetaminophen, aspirin, and ibuprofen should be avoided as these medications may be toxic to the liver. The patient should avoid potentially hepatotoxic over-the-counter drugs (e.g., acetaminophen) because the diseased liver is unable to metabolize these drugs. Spironolactone is a potassium-sparing diuretic. Lactulose results in the acidification of feces in bowel and trapping of ammonia, causing its elimination in feces.

In teaching a patient with malignant melanoma about this disorder, the nurse recognizes that the prognosis of the patient is most dependent on A) the thickness of the lesion B) the degree of color change in the lesion C) how much the lesion has spread superficially D) the amount of ulceration present in the lesion

A) the thickness of the lesion

In a patient admitted with cellulitis of the left foot, which of the following clinical manifestations would you expect to find on assessment of the left foot? A) Redness and swelling B) Pallor and poor turgor C) Cyanosis and coolness D) Edema and brown skin discoloration

A) Redness and swelling Cellulitis is a diffuse, acute inflammation of the skin. It is characterized by redness, swelling, and heat in the affected area. These changes accompany the processes of inflammation and infection.

A male patient returned from the operating room 6 hours ago with a cast on his right arm. He has not yet voided. Which action would be the most beneficial in assisting the patient to void? A) Suggest he stand at the bedside B) Stay with the patient C) Give him the urinal to use in bed D) Tell him that, if he doesn't urinate, he will be catheterized

A) Suggest he stand at the bedside

The nurse should teach a patient who is taking which of the following drugs to avoid prolonged sun exposure? A) Tetracycline B) Ipratropium C) Morphine sulfate D) Oral contraceptives

A) Tetracycline Several antibiotics, including tetracycline, may cause photosensitivity. This is not the case with ipratropium, morphine, or oral contraceptives.

Which of the following laboratory results is the best indicator that a patient with cellulitis is recovering from this infection? A) WBC of 8200/μl B) WBC of 2900/μl C) WBC of 16,300/μl D) WBC of 12,700/μl

A) WBC of 8200/μl The normal white blood cell count is generally 4000 to 11,000/μl. For this reason, the patient's level would be returning to normal if it was 8200/μl, indicating recovery from cellulitis.

Dermatologic manifestation(s) of Cushing syndrome can include (select all that apply) A) acne B) telangiectasia C) increased sweating D) brown pigmentation in the legs

A) acne B) telangiectasia

A mother and her two children have been diagnosed with pediculosis corporis at a health care center. An appropriate measure in teating this condition is: A) applying pyrethrins to the body B) topical application of griseofulvin C) moist compresses applied frequently D) administration of systemic antibiotics

A) applying pyrethrins to the body

Important patient teaching after a chemical peel includes A) avoidance of sun exposure B) application of firm bandages C) limitation of vigorous exercise D) use of ice to relieve discomfort

A) avoidance of sun exposure

Which individuals would be at high risk for low back pain (select all that apply)? a.A 63-year-old man who is a long-distance truck driver b.A 36-year-old 6 ft, 2 in construction worker who weighs 260 lb c.A 28-year-old female yoga instructor who is 5 ft, 6 in and weighs 130 lb d.A 30-year-old male nurse who works on an orthopedic unit and smokes e.A 44-year-old female chef with prior compression fracture of the spine

A, B, D, E Risk factors associated with low back pain include a lack of muscle tone and excess body weight, stress, poor posture, cigarette smoking, pregnancy, prior compression fractures of the spine, spinal problems since birth, and a family history of back pain. Jobs that require repetitive heavy lifting, vibration (such as a jackhammer operator), and prolonged periods of sitting are also associated with low back pain. Low back pain is most often caused by a musculoskeletal problem. The causes of low back pain of musculoskeletal origin include (1) acute lumbosacral strain, (2) instability of the lumbosacral bony mechanism, (3) osteoarthritis of the lumbosacral vertebrae, (4) degenerative disc disease, and (5) herniation of an intervertebral disc. Health care personnel are at high risk for the development of low back pain. Lifting and moving patients, excessive time being stooped over or leaning forward, and frequent twisting can result in low back pain.

Select the results (in italics) that are normal in a urinalysis. A. pH 6 B. Specific gravity 1.015 C. Protein small D. Sugar negative E. Nitrate small F. Leukocyte esterase positive G. Bilirubin negative

A, B, D, G Rationale: The abnormal values are indicative of a urinary tract infection. As a result of protein, nitrates, and leukoesterase in the urine, the nurse can expect the laboratory to analyze microscopic sediment including evaluating the sample for the presence of crystals, casts, WBCs, and RBCs.

The nurse provides instructions to a 30-year-old female office worker who has low back pain. Which statement by the patient requires an intervention by the nurse? A. "Acupuncture to the lower back would cause irreparable nerve damage." B. "Smoking may aggravate back pain by decreasing blood flow to the spine." C. "Sleeping on my side with knees and hips bent reduces stress on my back." D. "Switching between hot and cold packs provides relief of pain and stiffness."

A. "Acupuncture to the lower back would cause irreparable nerve damage." Acupuncture is a safe therapy when the practitioner has been appropriately trained. Very fine needles are inserted into the skin to stimulate specific anatomic points in the body for therapeutic purposes.

The nurse assesses a 76-year-old man with chronic myeloid leukemia receiving nilotinib (Tasigna). It is most important for the nurse to ask which question? A. "Have you had a fever?" B. "Have you lost any weight?" C. "Has diarrhea been a problem?" D. "Have you noticed any hair loss?"

A. "Have you had a fever?" An adverse effect of nilotinib is neutropenia. Infection is common in neutropenic patients and is the primary cause of death in cancer patients. Patients should report a temperature of 100.4o F or higher. Other adverse effects of nilotinib are thrombocytopenia, bleeding, nausea, fatigue, elevated lipase level, fever, rash, pruritus, diarrhea, and pneumonia.

5. When assessing the patient experiencing the onset of symptoms of type 1 diabetes, which question should the nurse ask? a. "Have you lost any weight lately?" b. "Do you crave fluids containing sugar?" c. "How long have you felt anorexic?" d. "Is your urine unusually dark-colored?"

A. "Have you lost any weight lately?" Rationale: Weight loss occurs because the body is no longer able to absorb glucose and starts to break down protein and fat for energy.

Which statement made by the patient with type 2 diabetes is accurate? a) "I am supposed to have a meal or snack if I drink alcohol." b) "I am not allowed to eat any sweets because of my diabetes." c) "I do not need to watch what I eat because mu diabetes is not the bad kind." d) "The amount of at in my diet is not important. Only carbohydrates raise my blood sugar."

A. "I am supposed to have a meal of snack if i drink alcohol"

25. Twelve hours after undergoing a gastroduodenostomy (Billroth I), a patient complains of increasing abdominal pain. The patient has absent bowel sounds and 200 mL of bright red nasogastric (NG) drainage in the last hour. The most appropriate action by the nurse at this time is to a. notify the surgeon. b. irrigate the NG tube. c. administer the prescribed morphine. d. continue to monitor the NG drainage.

ANS: A Increased pain and 200 mL of bright red NG drainage 12 hours after surgery indicate possible postoperative hemorrhage, and immediate actions such as blood transfusion and/or return to surgery are needed. Because the NG is draining, there is no indication that irrigation is needed. The patient may need morphine, but this is not the highest priority action. Continuing to monitor the NG drainage is not an adequate response.

A 20-year-old baseball pitcher has an arthroscopic repair of a rotator cuff injury performed in same-day surgery. When the nurse plans postoperative teaching for the patient, which information will be included? a. "You have an appointment with a physical therapist for tomorrow." b. "You can still play baseball but you will not be able to return to pitching." c. "The doctor will use the drop-arm test to determine the success of surgery." d. "Leave the shoulder immobilizer on for the first few days to minimize pain."

ANS: A Physical therapy after a rotator cuff repair begins on the first postoperative day to prevent "frozen shoulder." A shoulder immobilizer is used immediately after the surgery, but leaving the arm immobilized for several days would lead to loss of range of motion (ROM). The drop-arm test is used to test for rotator cuff injury, but not after surgery. The patient may be able to return to pitching after rehabilitation.

A patient with a right lower leg fracture will be discharged home with an external fixation device in place. Which information will the nurse teach? a. "You will need to check and clean the pin insertion sites daily." b. "The external fixator can be removed for your bath or shower." c. "You will need to remain on bed rest until bone healing is complete." d. "Prophylactic antibiotics are used until the external fixator is removed."

ANS: A Pin insertion sites should be cleaned daily to decrease the risk for infection at the site. An external fixator allows the patient to be out of bed and avoid the risks of prolonged immobility. The device is surgically placed and is not removed until the bone is stable. Prophylactic antibiotics are not routinely given when an external fixator is used.

A patient who has severe pain associated with terminal pancreatic cancer is being cared for at home by family members. Which finding by the nurse indicates that teaching regarding pain management has been effective? a. The patient uses the ordered opioid pain medication whenever the pain is greater than 5 (0 to 10 scale). b. The patient agrees to take the medications by the IV route in order to improve analgesic effectiveness. c. The patient takes opioids around the clock on a regular schedule and uses additional doses when breakthrough pain occurs. d. The patient states that nonopioid analgesics may be used when the maximal dose of the opioid is reached without adequate pain relief.

ANS: C For chronic cancer pain, analgesics should be taken on a scheduled basis, with additional doses as needed for breakthrough pain. Taking the medications only when pain reaches a certain level does not provide effective pain control. Although nonopioid analgesics also may be used, there is no maximum dose of opioid. Opioids are given until pain control is achieved. The IV route is not more effective than the oral route, and usually the oral route is preferred.

The nurse prepares to obtain a culture from a patient who has a possible fungal infection on the foot. Which items should the nurse gather for this procedure? a. Sterile gloves b. Patch test instruments c. Cotton-tipped applicators d. Local anesthetic, syringe, and intradermal needle

ANS: C Fungal cultures are obtained by swabbing the affected area of the skin with cotton-tipped applicators. Sterile gloves are not needed because it is not a sterile procedure. Local injection is not needed because the swabbing is not usually painful. The patch test is done to determine whether a patient is allergic to specific testing material, not for obtaining fungal specimens

After a patient has a short-arm plaster cast applied in the emergency department, which statement by the patient indicates a good understanding of the nurse's discharge teaching? a. "I can get the cast wet as long as I dry it right away with a hair dryer." b. "I should avoid moving my fingers and elbow until the cast is removed." c. "I will apply an ice pack to the cast over the fracture site for the next 24 hours." d. "I can use a cotton-tipped applicator to rub lotion on any dry areas under the cast."

ANS: C Ice application for the first 24 hours after a fracture will help reduce swelling and can be placed over the cast. Plaster casts should not get wet. The patient should be encouraged to move the joints above and below the cast. Patients should not insert objects inside the cast.

Which statement by the patient indicates a good understanding of the nurse's teaching about a new short-arm plaster cast? a. "I can get the cast wet as long as I dry it right away with a hair dryer." b. "I should avoid moving my fingers and elbow until the cast is removed." c. "I will apply an ice pack to the cast over the fracture site off and on for 24 hours." d. "I can use a cotton-tipped applicator to rub lotion on any dry areas under the cast."

ANS: C Ice application for the first 24 hours after a fracture will help reduce swelling and can be placed over the cast. Plaster casts should not get wet. The patient should be encouraged to move the joints above and below the cast. Patients should not insert objects inside the cast.

A high school teacher with ulnar drift caused by rheumatoid arthritis (RA) is scheduled for a left hand arthroplasty. Which patient statement to the nurse indicates a realistic expectation for the surgery? a. "This procedure will correct the deformities in my fingers." b. "I will not have to do as many hand exercises after the surgery." c. "I will be able to use my fingers with more flexibility to grasp things." d. "My fingers will appear more normal in size and shape after this surgery."

ANS: C The goal of hand surgery in RA is to restore function, not to correct for cosmetic deformity or treat the underlying process. Hand exercises will be prescribed after the surgery.

Following a motor vehicle accident, a patient arrives in the emergency department with massive right lower leg swelling. Which action will the nurse take first? a. Elevate the leg on pillows. b. Apply a compression bandage. c. Check leg pulses and sensation. d. Place ice packs on the lower leg.

ANS: C The initial action by the nurse will be to assess the circulation to the leg and to observe for any evidence of injury such as fractures or dislocations. After the initial assessment, the other actions may be appropriate based on what is observed during the assessment.

Following a motorcycle accident, a 58-year-old patient arrives in the emergency department with massive left lower leg swelling. Which action will the nurse take first? a. Elevate the leg on 2 pillows. b. Apply a compression bandage. c. Check leg pulses and sensation. d. Place ice packs on the lower leg.

ANS: C The initial action by the nurse will be to assess the circulation to the leg and to observe for any evidence of injury such as fractures or dislocations. After the initial assessment, the other actions may be appropriate, based on what is observed during the assessment.

During the teaching session for a patient who has a new diagnosis of acute leukemia the patient is restless and is looking away, never making eye contact. After teaching about the complications associated with chemotherapy, the patient asks the nurse to repeat all of the information. Based on this assessment, which nursing diagnosis is most appropriate for the patient? a. Risk for ineffective adherence to treatment related to denial of need for chemotherapy b. Acute confusion related to infiltration of leukemia cells into the central nervous system c. Risk for ineffective health maintenance related to anxiety about new leukemia diagnosis d. Deficient knowledge: chemotherapy related to a lack of interest in learning about treatment

ANS: C The patient who has a new cancer diagnosis is likely to have high anxiety, which may impact learning and require that the nurse repeat and reinforce information. The patient's history of a recent diagnosis suggests that infiltration of the leukemia is not a likely cause of the confusion. The patient asks for the information to be repeated, indicating that lack of interest in learning and denial are not etiologic factors.

A patient with cancer has a nursing diagnosis of imbalanced nutrition: less than body requirements related to altered taste sensation. Which nursing action is most appropriate? a. Add strained baby meats to foods such as casseroles. b. Teach the patient about foods that are high in nutrition. c. Avoid giving the patient foods that are strongly disliked. d. Add extra spice to enhance the flavor of foods that are served.

ANS: C The patient will eat more if disliked foods are avoided and foods that the patient likes are included instead. Additional spice is not usually an effective way to enhance taste. Adding baby meats to foods will increase calorie and protein levels, but does not address the issue of taste. The patient's poor intake is not caused by a lack of information about nutrition

A patient with cancer has a nursing diagnosis of imbalanced nutrition: less than body requirements related to altered taste sensation. Which nursing action is most appropriate? a. Add strained baby meats to foods such as casseroles. b. Teach the patient about foods that are high in nutrition. c. Avoid giving the patient foods that are strongly disliked. d. Add extra spice to enhance the flavor of foods that are served.

ANS: C The patient will eat more if disliked foods are avoided and foods that the patient likes are included instead. Additional spice is not usually an effective way to enhance taste. Adding baby meats to foods will increase calorie and protein levels, but does not address the issue of taste. The patient's poor intake is not caused by a lack of information about nutrition.

A checkout clerk in a grocery store has repetitive strain syndrome in the left elbow. The nurse will plan to teach the patient about a. surgical options. b. elbow injections. c. utilization of a left wrist splint. d. modifications in arm movement.

ANS: D Treatment for repetitive strain syndrome includes changing the ergonomics of the activity. Elbow injections and surgery are not initial options for this type of injury. A wrist splint might be used for hand or wrist pain.

A factory line worker has repetitive strain syndrome in the left elbow. The nurse will plan to teach the patient about a. surgical options. b. elbow injections. c. wearing a left wrist splint. d. modifying arm movements.

ANS: D Treatment for repetitive strain syndrome includes changing the ergonomics of the activity. Elbow injections and surgery are not initial options for this type of injury. A wrist splint might be used for hand or wrist pain.

A nurse observes that the client's left flank region is larger than the right flank region. What is the nurse's best action? A. Ask the client if he or she participates in contact sports and has been recently injured. B. Document the finding as the only action on the appropriate flowsheet. C. Apply a heating pad to the left flank after inspecting the site for signs of infection. D. Anticipate further diagnostic testing after sharing informing the physician of this finding

ANS: D Asymmetry of the flank or a unilateral protrusion may indicate an enlargement of a kidney. The enlargement may be benign or may be associated with a hydronephrosis or mass on the kidney.

35. Which of these laboratory values noted by the nurse when reviewing the chart of a diabetic patient indicates the need for further assessment of the patient? a. Fasting blood glucose of 130 mg/dl b. Noon blood glucose of 52 mg/dl c. Glycosylated hemoglobin of 6.9% d. Hemoglobin A1C of 5.8%

B. noon blood glucose of 52 mg/dl Rationale: The nurse should assess the patient with a blood glucose level of 52 mg/dl for symptoms of hypoglycemia, and give the patient some carbohydrate-containing beverage such as orange juice. The other values are within an acceptable range for a diabetic patient.

The patient has osteosarcoma of the right leg. The unlicensed assistive personnel (UAP) reports that the patient's vital signs are normal, but the patient says he still has pain in his leg and it is getting worse. What assessment question should the nurse ask the patient to determine treatment measures for this patient's pain? A. "Where is the pain?" B. "Is the pain getting worse?" C. "What does the pain feel like?" D. "Do you use medications to relieve the pain?"

C. "What does the pain feel like?" The unlicensed assistive personnel (UAP) told the nurse the location of the patient's pain and the worsening of pain (pattern). Asking about the quality of the pain will help in planning further treatment. The nurse should already know if the patient is using medication to relieve the pain or can check the patient's medication administration record to see if analgesics have been administered. The intensity of pain using a pain scale should also be assessed.

The female patient is having whole brain radiation for brain metastasis. She is concerned about how she will look when she loses her hair. What is the best response by the nurse to this patient? A. "When your hair grows back it will be patchy." B. "Don't use your curling iron and that will slow down the loss." C. "You can get a wig now to match your hair so you will not look different." D. "You should contact "Look Good, Feel Better" to figure out what to do about this."

C. "You can get a wig now to match your hair so you will not look different." Hair loss with radiation is usually permanent. The best response by the nurse is to suggest getting a wig before she loses her hair so she will not look or feel so different. When hair grows back after chemotherapy, it is frequently a different color or texture. Avoiding use of electric hair dryers, curlers, and curling irons may slow the hair loss but will not answer the patient's concern. The American Cancer Society's "Look Good, Feel Better" program will be helpful, but this response is avoiding the patient's immediate concern.

The patient was told that he would have intraperitoneal chemotherapy. He asks the nurse when the IV will be started for the chemotherapy. What should the nurse teach the patient about this type of chemotherapy delivery? A. It is delivered via an Ommaya reservoir and extension catheter. B. It is instilled in the bladder via a urinary catheter and retained for 1 to 3 hours. C. A Silastic catheter will be percutaneously placed into the peritoneal cavity for chemotherapy administration. D. The arteries supplying the tumor are accessed with surgical placement of a catheter connected to an infusion pump.

C. A Silastic catheter will be percutaneously placed into the peritoneal cavity for chemotherapy administration. Intraperitoneal chemotherapy is delivered to the peritoneal cavity via a temporary percutaneously inserted Silastic catheter and drained from this catheter after the dwell time in the peritoneum. The Ommaya reservoir is used for intraventricular chemotherapy. Intravesical bladder chemotherapy is delivered via a urinary catheter. Intraarterial chemotherapy is delivered via a surgically placed catheter that delivers chemotherapy via an external or internal infusion pump.

Priority Decision: The nurse is teaching a moderately obese woman interventions for the management of obesity. Initially, which strategies will support restricting dietary intake to below energy requirements (select all that apply)? a. Limit alcohol b. Rest when fatigued c. Determine portion sizes d. 1800- to 2200-calorie diet e. Attend Overeaters Anonymous

Correct answers: a, c Rationale: To restrict dietary intake so that it is below energy requirements, the moderately obese woman should limit or avoid alcohol intake because it increases caloric intake and has low nutritional value. Portion sizes have increased over the years and are larger than they should be. Teach the patient to determine portion sizes by weight or learn equivalencies such as that a serving of fruit is the size of a baseball. A progressive exercise program will increase energy requirements and a diet with an initial 800- to 1200-calorie limit would decrease calorie intake. Overeaters Anonymous would not restrict dietary intake below energy requirements, although it may offer support for the patient.

Health risks associated with obesity include (select all that apply) a. colorectal cancer. b. rheumatoid arthritis. c. polycystic ovary syndrome. d. nonalcoholic steatohepatitis. e. systemic lupus erythematosus.

Correct answers: a, c, d Rationale: Health risks associated with obesity include cardiovascular disease (related to increased low-density lipoprotein levels, increased triglyceride levels, and decreased high-density lipoprotein levels), hypertension, sleep apnea, obesity hypoventilation syndrome, reduced chest wall compliance, increased work of breathing, decreased total lung capacity and functional residual capacity, type 2 diabetes mellitus (i.e., hyperinsulinemia and insulin resistance), osteoarthritis, hyperuricemia, gout, gastroesophageal reflux disease, gallstones, nonalcoholic steatohepatitis, fatty liver and cirrhosis, cancer (mainly breast, endometrial, kidney, colorectal, pancreas, esophagus, and gallbladder), psychosocial problems (employment, education, and health care), low self-esteem, withdrawal from social interactions, and major depression.

The nurse is caring for a patient suffering from anorexia secondary to chemotherapy. Which strategy would be most appropriate for the nurse to use to increase the patient's nutritional intake? A. Increase intake of liquids at mealtime to stimulate the appetite. B. Serve three large meals per day plus snacks between each meal. C. Avoid the use of liquid protein supplements to encourage eating at mealtime. D. Add items such as skim milk powder, cheese, honey, or peanut butter to selected foods.

D. Add items such as skim milk powder, cheese, honey, or peanut butter to selected foods. The nurse can increase the nutritional density of foods by adding items high in protein and/or calories (such as peanut butter, skim milk powder, cheese, honey, or brown sugar) to foods the patient will eat. Increasing fluid intake at mealtime fills the stomach with fluid and decreases the desire to eat. Small frequent meals are best tolerated. Supplements can be helpful.

13. Which nursing action will be most useful in assisting a 21-year-old college student to adhere to a newly prescribed antiretroviral therapy (ART) regimen? a. Give the patient detailed information about possible medication side effects. b. Remind the patient of the importance of taking the medications as scheduled. c. Encourage the patient to join a support group for students who are HIV positive. d. Check the patient's class schedule to help decide when the ART should be taken.

D. Check the patient's class schedule to help decide when the ART should be taken The best approach to improve adherence is to learn about important activities in the patient's life and adjust the ART around those activities. The other actions also are useful, but they will not improve adherence as much as individualizing the ART to the patient's schedule.

When caring for the patient with cancer, what does the nurse understand as the response of the immune system to antigens of the malignant cells? A. Metastasis B. Tumor angiogenesis C. Immunologic escape D. Immunologic surveillance

D. Immunologic surveillance Immunologic surveillance is the process where lymphocytes check cell surface antigens and detect and destroy cells with abnormal or altered antigenic determinants to prevent these cells from developing into clinically detectable tumors. Metastasis is increased growth rate of the tumor, increased invasiveness, and spread of the cancer to a distant site in the progression stage of cancer development. Tumor angiogenesis is the process of blood vessels forming within the tumor itself. Immunologic escape is the cancer cells' evasion of immunologic surveillance that allows the cancer cells to reproduce.

The nurse is caring for a patient admitted with suspected hyperparathyroidism. Because of the potential effects of this disease on electrolyte balance, the nurse should assess this patient for what manifestation? A. Neurologic irritability B. Declining urine output D. Lethargy and weakness C. Hyperactive bowel sounds

D. Lethargy and weakness Hyperparathyroidism can cause hypercalcemia. Signs of hypercalcemia include muscle weakness, polyuria, constipation, nausea and vomiting, lethargy, and memory impairment. Neurologic irritability, declining urine output, and hyperactive bowel sounds do not occur with hypercalcemia.

The nurse cares for a 58-year-old woman with breast cancer who is admitted for severe back pain related to a compression fracture. The patient's laboratory values include serum potassium of 4.5 mEq/L, serum sodium of 144 mEq/L, and serum calcium of 14.3 mg/dL. Which signs and symptoms will the nurse expect the patient to exhibit? A. Anxiety, irregular pulse, and weakness B. Muscle stiffness, dysphagia, and dyspnea C. Hyperactive reflexes, tremors, and seizures D. Nausea, vomiting, and altered mental status

D. Nausea, vomiting, and altered mental status Breast cancer can metastasize to the bone. Vertebrae are a common site. Pathologic fractures at the site of metastasis are common because of a weakening of the involved bone. High serum calcium levels result as calcium is released from damaged bones. Normal serum calcium is between 8.6 to 10.2 mg/dL. Clinical manifestations of hypercalcemia include nausea, vomiting, and altered mental status (e.g., lethargy, decreased memory, confusion, personality changes, psychosis, stupor, coma). Other manifestations include weakness, depressed reflexes, anorexia, bone pain, fractures, polyuria, dehydration, and nephrolithiasis. Manifestations of hypomagnesemia include hyperactive reflexes, tremors, and seizures. Symptoms of hyperkalemia include anxiety, irregular pulse, and weakness. Symptoms of hypocalcemia include muscle stiffness, dysphagia, and dyspnea.

A 67-year-old patient hospitalized with osteomyelitis has an order for bed rest with bathroom privileges with the affected foot elevated on two pillows. The nurse would place highest priority on which intervention? A. Ambulate the patient to the bathroom every 2 hours. B. Ask the patient about preferred activities to relieve boredom. C. Allow the patient to dangle legs at the bedside every 2 to 4 hours. D. Perform frequent position changes and range-of-motion exercises.

D. Perform frequent position changes and range-of-motion exercises. The patient is at risk for atelectasis of the lungs and for contractures because of prescribed bed rest. For this reason, the nurse should place the priority on changing the patient's position frequently to promote lung expansion and performing range-of-motion (ROM) exercises to prevent contractures. Assisting the patient to the bathroom will keep the patient safe as the patient is in pain, but it may not be needed every 2 hours. Providing activities to relieve boredom will assist the patient to cope with the bed rest, and dangling the legs every 2 to 4 hours may be too painful. The priority is position changes and ROM exercises.

The patient with an adrenal hyperplasia is returning from surgery for an adrenalectomy. For what immediate postoperative risk should the nurse plan to monitor the patient? A. Vomiting B. Infection C. Thomboembolism D. Rapid BP changes

D. Rapid BP changes The risk of hemorrhage is increased with surgery on the adrenal glands as well as large amounts of hormones being released in the circulation, which may produce hypertension and cause fluid and electrolyte imbalances to occur for the first 24 to 48 hours after surgery. Vomiting, infection, and thromboembolism may occur postoperatively with any surgery.

8. A college student who has type 1 diabetes normally walks each evening as part of an exercise regimen. The student now plans to take a swimming class every day at 1:00 PM. The clinic nurse teaches the patient to a. delay eating the noon meal until after the swimming class. b. increase the morning dose of neutral protamine Hagedorn (NPH) insulin on days of the swimming class. c. time the morning insulin injection so that the peak occurs while swimming. d. check glucose level before, during, and after swimming.

D. check glucose level before, during, and after swimming. Rationale: The change in exercise will affect blood glucose, and the patient will need to monitor glucose carefully to determine the need for changes in diet and insulin administration. Because exercise tends to decrease blood glucose, patients are advised to eat before exercising. Increasing the morning NPH or timing the insulin to peak during exercise may lead to hypoglycemia, especially with the increased exercise. Cognitive Level: Application Text Reference: p. 1269 Nursing Process: Implementation NCLEX: Health Promotion and Maintenance

4. A program of weight loss and exercise is recommended for a patient with impaired fasting glucose (IFG). When teaching the patient about the reason for these lifestyle changes, the nurse will tell the patient that a. the high insulin levels associated with this syndrome damage the lining of blood vessels, leading to vascular disease. b. although the fasting plasma glucose levels do not currently indicate diabetes, the glycosylated hemoglobin will be elevated. c. the liver is producing excessive glucose, which will eventually exhaust the ability of the pancreas to produce insulin, and exercise will normalize glucose production. d. the onset of diabetes and the associated cardiovascular risks can be delayed or prevented by weight loss and exercise.

D. the onset of diabetes and the associated cardiovascular risks can be delayed or prevented by weight loss and exercise. Rationale: The patient with IFG is at risk for developing type 2 diabetes, but this risk can be decreased with lifestyle changes. Glycosylated hemoglobin levels will not be elevated in IFG and the Hb A1C test is not included in prediabetes testing. Elevated insulin levels do not cause the damage to blood vessels that can occur with IFG. The liver does not produce increased levels of glucose in IFG. Cognitive Level: Application Text Reference: p. 1255 Nursing Process: Implementation NCLEX: Physiological Integrity

When caring for a patient with liver disease, the nurse recognizes the need to prevent bleeding resulting from altered clotting factors and rupture of varices. Which nursing interventions would be appropriate to achieve this outcome (select all that apply)? Use smallest gauge needle possible when giving injections or drawing blood. Teach patient to avoid straining at stool, vigorous blowing of nose, and coughing. Advise patient to use soft-bristle toothbrush and avoid ingestion of irritating food. Apply gentle pressure for the shortest possible time period after performing venipuncture. Instruct patient to avoid aspirin and NSAIDs to prevent hemorrhage when varices are present.

Use smallest gauge needle possible when giving injections or drawing blood. Teach patient to avoid straining at stool, vigorous blowing of nose, and coughing. Advise patient to use soft-bristle toothbrush and avoid ingestion of irritating food. Instruct patient to avoid aspirin and NSAIDs to prevent hemorrhage when varices are present. Using the smallest gauge needle for injections will minimize the risk of bleeding into the tissues. Avoiding straining, nose blowing, and coughing will reduce the risk of hemorrhage at these sites. The use of a soft-bristle toothbrush and avoidance of irritating food will reduce injury to highly vascular mucous membranes. The nurse should apply gentle but prolonged pressure to venipuncture sites to minimize the risk of bleeding. Aspirin and NSAIDs should not be used in patients with liver disease because they interfere with platelet aggregation, thus increasing the risk for bleeding.

Nursing management of the patient with acute pancreatitis includes (select all that apply) a. checking for signs of hypocalcemia. b. providing a diet low in carbohydrates. c. giving insulin based on a sliding scale. d. observing stools for signs of steatorrhea. e. monitoring for infection, particularly respiratory tract infection. (Lewis 1042)

a, e Rationale: During the acute phase, it is important to monitor vital signs. Hemodynamic stability may be compromised by hypotension, fever, and tachypnea. Intravenous fluids are ordered, and the response to therapy is monitored. Fluid and electrolyte balances are closely monitored. Frequent vomiting, along with gastric suction, may result in decreased levels of chloride, sodium, and potassium. Because hypocalcemia can occur in acute pancreatitis, the nurse should observe for symptoms of tetany, such as jerking, irritability, and muscular twitching. Numbness or tingling around the lips and in the fingers is an early indicator of hypocalcemia. The patient should be assessed for Chvostek's sign or Trousseau's sign. A patient with acute pancreatitis should be observed for fever and other manifestations of infection. Respiratory infections are common because the retroperitoneal fluid raises the diaphragm, which causes the patient to take shallow, guarded abdominal breaths.

3. A patient with acromegaly is treated with a transsphenoidal hypophysectomy. What should the nurse do postoperatively? a. Ensure that any clear nasal drainage is tested for glucose. b. Maintain the patient flat in bed to prevent cerebrospinal fluid (CSF) leakage. c. Assist the patient with toothbrushing every 4 hours to keep the surgical area clean. d. Encourage deep breathing, coughing, and turning to prevent respiratory complications.

a. Ensure that any clear nasal drainage is tested for glucose A transsphenoidal hypophysectomy involves entry into the sella turcica through an incision in the upper lip and gingiva into the floor of the nose and the sphenoid sinuses. Postoperative clear nasal drainage with glucose content indicates cerebrospinal fluid (CSF) leakage from an open connection to the brain, putting the patient at risk for meningitis. After surgery, the patient is positioned with the head elevated to avoid pressure on the sella turcica. Coughing and straining are avoided to prevent increased intracranial pressure and CSF leakage. Although mouth care is required every 4 hours, toothbrushing should not be performed because injury to the suture line may occur.

A man with end-stage kidney disease is scheduled for hemodialysis following healing of an arteriovenous fistula (AVF). What should the nurse explain to him that will occur during dialysis? a. He will be able to visit, read, sleep, or watch TV while reclining in a chair. b. He will be placed on a cardiac monitor to detect any adverse effects that might occur. c. The dialyzer will remove and hold part of his blood for 20 to 30 minutes to remove the waste products. d. A large catheter with two lumens will be inserted into the fistula to send blood to and return it from the dialyzer.

a. While patients are undergoing hemodialysis, they can perform quiet activities that do not require the limb that has the vascular access. Blood pressure is monitored frequently and the dialyzer monitors dialysis function but cardiac monitoring is not usually indicated. The hemodialysis machine continuously circulates both the blood and the dialysate past the semipermeable membrane in the machine. Graft and fistula access involve the insertion of two needles into the site: one to remove blood from and the other to return blood to the dialyzer.

A patient with osteomyelitis is treated with surgical debridement with implantation of antibiotic beads. When the patient asks why the beads are used, the nurse answers (select all that apply) a."The beads are used to directly deliver antibiotics to the site of the infection." b."There are no effective oral or IV antibiotics to treat most cases of bone infection." c."This is the safest method of delivering long-term antibiotic therapy for a bone infection." d."The beads are an adjunct to debridement and oral and IV antibiotics for deep infections." e."The ischemia and bone death that occur with osteomyelitis are impenetrable to IV antibiotics."

a."The beads are used to directly deliver antibiotics to the site of the infection." d."The beads are an adjunct to debridement and oral and IV antibiotics for deep infections." Treatment of chronic osteomyelitis includes surgical removal of the poorly vascularized tissue and dead bone and the extended use of IV and oral antibiotics. Antibiotic-impregnated polymethylmethacrylate bead chains may be implanted during surgery to aid in combating the infection.

Before discharge from the same-day surgery unit, instruct the patient who has had a surgical correction of bilateral hallux valgus to a.rest frequently with the feet elevated. b.soak the feet in warm water several times a day. c.expect the feet to be numb for the next few days. d.expect continued pain in the feet, since this is not uncommon.

a.rest frequently with the feet elevated. After surgical correction of bilateral hallux valgus, the feet should be elevated with the heel off the bed to help reduce discomfort and prevent edema.

6. The patient is diagnosed with syndrome of inappropriate antidiuretic hormone (SIADH). What manifestation should the nurse expect to find? a. Decreased body weight b. Decreased urinary output c. Increased plasma osmolality d. Increased serum sodium levels

b. Decreased urinary output With increased antidiuretic hormone (ADH), the permeability of the renal distal tubules is increased, so water is reabsorbed into circulation. Decreased output of concentrated urine with increased urine osmolality and specific gravity occur. In addition, fluid retention with weight gain, serum hypoosmolality, dilutional hyponatremia, and hypochloremia occur.

28. A patient has been diagnosed with hypoparathyroidism. What manifestations should the nurse expect to observe (select all that apply)? a. Skeletal pain b. Dry, scaly skin c. Personality changes d. Abdominal cramping e. Cardiac dysrhythmias f. Muscle spasms and stiffness

b. Dry, scaly skin c. Personality changes d. Abdominal cramping e. Cardiac dysrhythmias f. Muscle spasms and stiffness In hypoparathyroidism the patient has inadequate circulating parathyroid hormone (PTH) that leads to hypocalcemia from the inability to maintain serum calcium levels. With hypocalcemia there is muscle stiffness and spasms, which can lead to cardiac dysrhythmias and abdominal cramps. There can also be personality and visual changes and dry, scaly skin.

A patient with AKI has a serum potassium level of 6.7 mEq/L (6.7 mmol/L) and the following arterial blood gas results: pH 7.28, PaCO2 30 mm Hg, PaO2 86 mm Hg, HCO3 − 18 mEq/L (18 mmol/L). The nurse recognizes that treatment of the acid-base problem with sodium bicarbonate would cause a decrease in which value? a. pH b. Potassium level c. Bicarbonate level d. Carbon dioxide level

b. During acidosis, potassium moves out of the cell in exchange for H+ ions, increasing the serum potassium level. Correction of the acidosis with sodium bicarbonate will help to shift the potassium back into the cells. A decrease in pH and the bicarbonate and PaCO2 levels would indicate worsening acidosis.

Priority Decision: A dehydrated patient is in the Injury stage of the RIFLE staging of AKI. What would the nurse first anticipate in the treatment of this patient? a. Assess daily weight b. IV administration of fluid and furosemide (Lasix) c. IV administration of insulin and sodium bicarbonate d. Urinalysis to check for sediment, osmolality, sodium, and specific gravity

b. Injury is the stage of RIFLE classification when urine output is less than 0.5 mL/kg/hr for 12 hours, the serum creatinine is increased times two or the glomerular filtration rate (GFR) is decreased by 50%. This stage may be reversible by treating the cause or, in this patient, the dehydration by administering IV fluid and a low dose of a loop diuretic, furosemide (Lasix). Assessing the daily weight will be done to monitor fluid changes but it is not the first treatment the nurse should anticipate. IV administration of insulin and sodium bicarbonate would be used for hyperkalemia. Checking the urinalysis will help to determine if the AKI has a prerenal, intrarenal, or postrenal cause by what is seen in the urine but with this patient's dehydration, it is thought to be prerenal to begin treatment.

Which assessment parameter is of highest priority when caring for a patient undergoing a water deprivation test? a. Serum glucose b. Patient weight c. Arterial blood gases d. Patient temperature

b. Patient weight A patient is at risk for severe dehydration during a water deprivation test. The test should be discontinued and the patient rehydrated if the patient's weight drops more than 2 kg at any time. The other assessment parameters do not assess fluid balance.

A patient with pancreatic cancer is admitted to the hospital for evaluation of possible treatment options. The patient asks the nurse to explain the Whipple procedure that the surgeon has described. The explanation includes the information that a Whipple procedure involves a. creating a bypass around the obstruction caused by the tumor by joining the gallbladder to the jejunum. b. resection of the entire pancreas and the distal portion of the stomach, with anastomosis of the common bile duct and the stomach into the duodenum. c. removal of part of the pancreas, part of the stomach, the duodenum, and the gallbladder, with joining of the pancreatic duct, the common bile duct, and the stomach into the jejunum. d. radical removal of the pancreas, the duodenum, and the spleen, and attachment of the stomach to the jejunum, which requires oral supplementation of pancreatic digestive enzymes and insulin replacement therapy. (Lewis 1042)

c Rationale: The classic operation for pancreatic cancer is a radical pancreaticoduodenectomy, or Whipple procedure. This entails resection of the proximal pancreas (i.e., proximal pancreatectomy), the adjoining duodenum (i.e., duodenectomy), the distal portion of the stomach (i.e., partial gastrectomy), and the distal segment of the common bile duct. The pancreatic duct, common bile duct, and stomach are anastomosed to the jejunum.

13. What characteristic is related to Hashimoto's thyroiditis? a. Enlarged thyroid gland b. Viral-induced hyperthyroidism c. Bacterial or fungal infection of thyroid gland d. Chronic autoimmune thyroiditis with antibody destruction of thyroid tissue

d. Chronic autoimmune thyroiditis with antibody destruction of thyroid tissue In Hashimoto's thyroiditis, thyroid tissue is destroyed by autoimmune antibodies. An enlarged thyroid gland is a goiter. Viral-induced hyperthyroidism is subacute granulomatous thyroiditis. Acute thyroiditis is caused by bacterial or fungal infection.

24. Priority Decision: When replacement therapy is started for a patient with long-standing hypothyroidism, what is most important for the nurse to monitor the patient for? a. Insomnia b. Weight loss c. Nervousness d. Dysrhythmias

d. Dysrhythmias All these manifestations may occur with treatment of hypothyroidism. However, as a result of the effects of hypothyroidism on the cardiovascular system, when thyroid replacement therapy is started myocardial oxygen consumption is increased and the resultant oxygen demand may cause angina, cardiac dysrhythmias, and heart failure, so monitoring for dysrhythmias is most important.

When teaching the patient with acute hepatitis C (HCV), the patient demonstrates understanding when the patient makes which statement? "I will use care when kissing my wife to prevent giving it to her." "I will need to take adofevir (Hepsera) to prevent chronic HCV." "Now that I have had HCV, I will have immunity and not get it again." "I will need to be checked for chronic HCV and other liver problems."

"I will need to be checked for chronic HCV and other liver problems." The majority of patients who acquire HCV usually develop chronic infection, which may lead to cirrhosis or liver cancer. HCV is not transmitted via saliva, but percutaneously and via high-risk sexual activity exposure. The treatment for acute viral hepatitis focuses on resting the body and adequate nutrition for liver regeneration. Adofevir (Hepsera) is taken for severe hepatitis B (HBV) with liver failure. Chronic HCV is treated with pegylated interferon with ribavirin. Immunity with HCV does not occur as it does with HAV and HBV, so the patient may be reinfected with another type of HCV.

The patient with cirrhosis is being taught self-care. Which statement indicates the patient needs more teaching? "If I notice a fast heart rate or irregular beats, this is normal for cirrhosis." "I need to take good care of my belly and ankle skin where it is swollen." "A scrotal support may be more comfortable when I have scrotal edema." "I can use pillows to support my head to help me breathe when I am in bed."

"If I notice a fast heart rate or irregular beats, this is normal for cirrhosis." Correct If the patient with cirrhosis experiences a fast or irregular heart rate, it may be indicative of hypokalemia and should be reported to the health care provider, as this is not normal for cirrhosis. Edematous tissue is subject to breakdown and needs meticulous skin care. Pillows and a semi-Fowler's or Fowler's position will increase respiratory efficiency. A scrotal support may improve comfort if there is scrotal edema.

Which of the following safe sun practices would the nurse include in the teaching care plan for a patient who has photosensitivity? (select all that apply) A) wear protective clothing B) apply sunscreen liberally and often C) tanning booths decrease the likelihood of sunburn D) avoid exposure to the sun, especially during midday E) wear any sunscreen as long as it is purchased at a drugstore

A) wear protective clothing B) apply sunscreen liberally and often D) avoid exposure to the sun, especially during midday

A stroke patient who primarily uses a wheelchair for mobility has diarrhea with fecal incontinence. What should the nurse assess first? A. Fecal impaction B. Perineal hygiene C. Dietary fiber intake D. Antidiarrheal agent use

A Patients with limited mobility are at risk for fecal impactions due to constipation that may lead to liquid stool leaking around the hardened impacted feces, so assessing for fecal impaction is the priority. Perineal hygiene can be assessed at the same time. Assessing the dietary fiber and fluid intake and antidiarrheal agent use will be assessed and considered next.

The nurse is preparing to insert a nasogastric (NG) tube into a 68-year-old female patient who is nauseated and vomiting. She has an abdominal mass and suspected small intestinal obstruction. The patient asks the nurse why this procedure is necessary. What response by the nurse is most appropriate? A. "The tube will help to drain the stomach contents and prevent further vomiting." B. "The tube will push past the area that is blocked and thus help to stop the vomiting." C. "The tube is just a standard procedure before many types of surgery to the abdomen." D. "The tube will let us measure your stomach contents so that we can plan what type of IV fluid replacement would be best."

A The NG tube is used to decompress the stomach by draining stomach contents and thereby prevent further vomiting. The NG tube will not push past the blocked area. Potential surgery is not currently indicated. The location of the obstruction will determine the type of fluid to use, not measure the amount of stomach contents

When evaluating the patient's understanding about the care of the ileostomy, what statement by the patient indicates the patient needs more teaching? A. "I will be able to regulate when I have stools." B. "I will be able to wear the pouch until it leaks." C. "Dried fruit and popcorn must be chewed very well." D. "The drainage from my stoma can damage my skin."

A The ileostomy is in the ileum and drains liquid stool frequently, unlike the colostomy which has more formed stool the further distal the ostomy is in the colon. The ileostomy pouch is usually worn 4-7 days or until it leaks. It must be changed immediately if it leaks because the drainage is very irritating to the skin. To avoid obstruction, popcorn, dried fruit, coconut, mushrooms, olives, stringy vegetables, food with skin, and meats with casings must be chewed extremely well before swallowing because of the narrow diameter of the ileostomy lumen.

24. A patient with gout tells the nurse that he takes losartan (Cozaar) for control of the condition. The nurse will need to monitor a. blood pressure. b. blood glucose. c. erythrocyte count. d. lymphocyte count.

Answer: A Rationale: Losartan, an angiotensin II receptor antagonist, will lower blood pressure. It does not affect blood glucose, red blood cell count (RBC), or lymphocytes. Cognitive Level: Comprehension Text Reference: p. 1716 Nursing Process: Planning NCLEX: Physiological Integrity

Two days following a colectomy for an abdominal mass, a patient reports gas pains and abdominal distention. The nurse plans care for the patient based on the knowledge that the symptoms are occurring as a result of A. impaired peristalsis. B. irritation of the bowel. C. nasogastric suctioning. D. inflammation of the incision site.

A Until peristalsis returns to normal following anesthesia, the patient may experience slowed gastrointestinal motility leading to gas pains and abdominal distention. Irritation of the bowel, nasogastric suctioning, and inflammation of the surgical site do not cause gas pains or abdominal distention.

Which statements will the nurse include when teaching a patient who is scheduled for oral glucose tolerance testing in the outpatient clinic (select all that apply)? a. "You will need to avoid smoking before the test." b. "Exercise should be avoided until the testing is complete." c. "Several blood samples will be obtained during the testing." d. "You should follow a low-calorie diet the day before the test." e. "The test requires that you fast for at least 8 hours before testing."

A C E

The patient had a lumbar spine arthrodesis. What should the nurse include in discharge teaching (select all that apply)? A Do not smoke cigarettes. B You should not walk for 3 weeks. C You must wear your brace at all times. D You may drive as soon as you feel like it. E Do not bend your spine until your follow-up appointment.

A Do not smoke cigarettes. E Do not bend your spine until your follow-up appointment. After a spinal fusion, the patient should not smoke cigarettes as nonunion tends to occur more often with smokers. Preventing pressure by not bending or twisting the spine or lifting more than 10 pounds will facilitate healing. The amount of time that is needed will be determined by the surgeon at follow-up appointments, but healing usually takes 6 to 9 months. An important aspect of healing is progressively increasing walking, which increases circulation of nutrients and oxygen for healing. If a brace is ordered to protect the surgical area, the surgeon will order how often the patient should wear it. Driving is not done until the surgeon allows it and the patient is no longer taking opioids for pain.

The nurse is caring for a woman recently diagnosed with viral hepatitis A. Which individual should the nurse refer for an immunoglobin (IG) injection? A caregiver who lives in the same household with the patient A friend who delivers meals to the patient and family each week A relative with a history of hepatitis A who visits the patient daily A child living in the home who received the hepatitis A vaccine 3 months ago

A caregiver who lives in the same household with the patient IG is recommended for persons who do not have anti-HAV antibodies and are exposed as a result of close contact with persons who have HAV or foodborne exposure. Persons who have received a dose of HAV vaccine more than 1 month previously or who have a history of HAV infection do not require IG.

When providing discharge teaching for the patient after a laparoscopic cholecystectomy, what information should the nurse include? A lower-fat diet may be better tolerated for several weeks. Correct Do not return to work or normal activities for 3 weeks. Bile-colored drainage will probably drain from the incision. Keep the bandages on and the puncture site dry until it heals.

A lower-fat diet may be better tolerated for several weeks. Correct Although the usual diet can be resumed, a low-fat diet is usually better tolerated for several weeks following surgery. Normal activities can be gradually resumed as the patient tolerates. Bile-colored drainage or pus, redness, swelling, severe pain, and fever may all indicate infection. The bandage may be removed the day after surgery, and the patient can shower.

Female orgasm is the result of (select all that apply): A. uterine and vaginal contractions B. clitoral swelling and increased vaginal lubrication C.vaginal enlargement and uterine elevation secretion D. rapid release of muscular tension in the reproductive structures

A, D

The nurse instructs a 28-year-old man with acromegaly resulting from an unresectable benign pituitary tumor about octreotide (Sandostatin). The nurse should intervene if the patient makes which statement? A. "I will come in to receive this medication IV every 2 to 4 weeks." B. "I will inject the medication in the subcutaneous layer of the skin." C. "The medication will decrease the growth hormone production to normal." D. "If radiation treatment is not effective, I may need to take the medication."

A. "I will come in to receive this medication IV every 2 to 4 weeks Drugs are most commonly used in patients who have had an inadequate response to or cannot be treated with surgery and/or radiation therapy. The most common drug used for acromegaly is octreotide (Sandostatin), a somatostatin analog that reduces growth hormone levels to within the normal range in many patients. Octreotide is given by subcutaneous injection three times a week. Two long-acting analogs, octreotide (Sandostatin LAR) and lanreotide SR (Somatuline Depot), are available as intramuscular (IM) injections given every 2 to 4 weeks.

A 54-year-old patient with acute osteomyelitis asks the nurse how this problem will be treated. Which response by the nurse is most appropriate? A. "IV antibiotics are usually required for several weeks." B. "Oral antibiotics are often required for several months." C. "Surgery is almost always necessary to remove the dead tissue that is likely to be present." D. "Drainage of the foot and instillation of antibiotics into the affected area is the usual therapy."

A. "IV antibiotics are usually required for several weeks." The standard treatment for acute osteomyelitis consists of several weeks of IV antibiotic therapy. This is because bone is denser and less vascular than other tissues, and it takes time for the antibiotic therapy to eradicate all of the microorganisms. Surgery may be used for chronic osteomyelitis, which may include debridement of the devitalized and infected tissue and irrigation of the affected bone with antibiotics.

Previous administrations of chemotherapy agents to a cancer patient have resulted in diarrhea. Which dietary modification should the nurse recommend? A. A bland, low-fiber diet B. A high-protein, high-calorie diet C. A diet high in fresh fruits and vegetables D. A diet emphasizing whole and organic foods

A. A bland, low-fiber diet Patients experiencing diarrhea secondary to chemotherapy and/or radiation therapy often benefit from a diet low in seasonings and roughage before the treatment. Foods should be easy to digest and low in fat. Fresh fruits and vegetables are high in fiber and should be minimized during treatment. Whole and organic foods do not prevent diarrhea.

Which nursing action will be most helpful in decreasing the risk for hospital-acquired infection (HAI) of the urinary tract in patients admitted to the hospital? a. Avoid unnecessary catheterizations. b. Encourage adequate oral fluid intake. c. Test urine with a dipstick daily for nitrites. d. Provide thorough perineal hygiene to patients.

A. Avoid unnecessary catherizations Since catheterization bypasses many of the protective mechanisms that prevent urinary tract infection (UTI), avoidance of catheterization is the most effective means of reducing HAI. The other actions will also be helpful, but are not as useful as decreasing urinary catheter use.

The nurse is admitting a patient to the nursing unit with a history of a herniated lumbar disc and low back pain. In completing a more thorough pain assessment, the nurse should ask the patient if which action aggravates the pain? A. Bending or lifting B. Application of warm moist heat C. Sleeping in a side-lying position D. Sitting in a fully extended recliner

A. Bending or lifting Back pain that is related to a herniated lumbar disc often is aggravated by events and activities that increase the stress and strain on the spine, such as bending or lifting, coughing, sneezing, and lifting the leg with the knee straight (straight leg-raising test). Application of moist heat, sleeping position, and ability to sit in a fully extended recliner do not aggravate the pain of a herniated lumbar disc.

The nurse is caring for a patient hospitalized with exacerbation of chronic bronchitis and herniated lumbar disc. Which breakfast choice would be most appropriate for the nurse to encourage the patient to check on the breakfast menu? A. Bran muffin B. Scrambled eggs C. Puffed rice cereal D. Buttered white toast

A. Bran muffin Each meal should contain one or more sources of fiber, which will reduce the risk of constipation and straining with defecation, which increases back pain. Bran is typically a high-fiber food choice and is appropriate for selection from the menu. Scrambled eggs, puffed rice cereal, and buttered white toast do not have as much fiber.

The laboratory reports that the cells from the patient's tumor biopsy are Grade II. What should the nurse know about this histologic grading? A. Cells are abnormal and moderately differentiated. B. Cells are very abnormal and poorly differentiated. C. Cells are immature, primitive, and undifferentiated. D. Cells differ slightly from normal cells and are well-differentiated.

A. Cells are abnormal and moderately differentiated. Grade II cells are more abnormal than Grade I and moderately differentiated. Grade I cells differ slightly from normal cells and are well-differentiated. Grade III cells are very abnormal and poorly differentiated. Grade IV cells are immature, primitive, and undifferentiated; the cell origin is difficult to determine.

A patient who is diagnosed with nephrotic syndrome has 3+ ankle and leg edema and ascites. Which nursing diagnosis is a priority for the patient? a. Excess fluid volume related to low serum protein levels b. Activity intolerance related to increased weight and fatigue c. Disturbed body image related to peripheral edema and ascites d. Altered nutrition: less than required related to protein restriction

A. Excess fluid volume related to low serum protein levels The patient has massive edema, so the priority problem at this time is the excess fluid volume. The other nursing diagnoses also are appropriate, but the focus of nursing care should be resolution of the edema and ascites.

The nurse is caring for a 68-year-old woman after a parathyroidectomy related to hyperparathyroidism. The nurse should administer IV calcium gluconate if the patient exhibits which clinical manifestations? A. Facial muscle spasms or laryngospasms B. Decreased muscle tone or muscle weakness C. Tingling in the hands and around the mouth D. Shortened QT interval on the electrocardiogram

A. Facial muscle spasms or laryngospasms Nursing care for the patient following a parathyroidectomy includes monitoring for a sudden decrease in serum calcium levels causing tetany, a condition of neuromuscular hyperexcitability. If tetany is severe (e.g., muscular spasms or laryngospasms develop), IV calcium gluconate should be administered. Mild tetany, characterized by unpleasant tingling of the hands and around the mouth, may be present but should decrease over time without treatment. Decreased muscle tone, muscle weakness, and shortened QT interval are clinical manifestations of hyperparathyroidism.

28. Intramuscular glucagon is administered to an unresponsive patient for treatment of hypoglycemia. Which action should the nurse take after the patient regains consciousness? a. Give the patient a snack of cheese and crackers. b. Have the patient drink a glass of orange juice or nonfat milk. c. Administer a continuous infusion of 5% dextrose for 24 hours. d. Assess the patient for symptoms of hyperglycemia.

A. Give the patient a snack of cheese and crackers. Rationale: Rebound hypoglycemia can occur after glucagon administration, but having a meal containing complex carbohydrates plus protein and fat will help prevent hypoglycemia. Orange juice and nonfat milk will elevate blood sugar rapidly, but the cheese and crackers will stabilize blood sugar. Administration of glucose intravenously might be used in patients who were unable to take in nutrition orally. The patient should be assessed for symptoms of hypoglycemia after glucagon administration.

1. When caring for a patient who has just been diagnosed with early chronic HIV infection, which prophylactic measures will the nurse anticipate being included in the plan of care (select all that apply)? a. Hepatitis B vaccine b. Pneumococcal vaccine c. Influenza virus vaccine d. Trimethoprim-sulfamethoxazole e. Varicella zoster immune globulin

A. Hepatitis B vaccine B. Pneumococcal vaccine C. Influenza virus vaccine Prevention of other infections is an important intervention in patients who are HIV positive, and these vaccines are recommended as soon as the HIV infection is diagnosed. Antibiotics and immune globulin are used to prevent and treat infections that occur later in the course of the disease, when the CD4 count has dropped or when infection has occurred.

A 32-year-old patient is diagnosed with polycystic kidney disease. Which information is most appropriate for the nurse to include in teaching at this time? a. Importance of genetic counseling b. Complications of renal transplantation c. Methods for treating chronic and severe pain d. Differences between hemodialysis and peritoneal dialysis

A. Importance of genetic counseling Because a 32-year-old patient may be considering having children, the nurse should include information about genetic counseling when teaching the patient. The well-managed patient will not need to choose between hemodialysis and peritoneal dialysis or know about the effects of transplantation for many years. There is no indication that the patient has chronic pain.

An 88-year-old with benign prostatic hyperplasia (BPH) has a markedly distended bladder and is agitated and confused. Which of the following interventions prescribed by the health care provider should the nurse implement first? a. Insert a urinary retention catheter. b. Schedule an intravenous pyelogram. c. Administer lorazepam (Ativan) 0.5 mg PO. d. Draw blood for blood urea nitrogen (BUN) and creatinine testing.

A. Insert a urinary retention catheter The patient's history and clinical manifestations are consistent with acute urinary retention, and the priority action is to relieve the retention by catheterization. The BUN and creatinine measurements can be obtained after the catheter is inserted. The patient's agitation may resolve once the bladder distention is corrected, and sedative drugs should be used cautiously in older patients. The IVP is an appropriate test, but does not need to be done urgently.

The patient and his family are upset that the patient is going through procedures to diagnose cancer. What nursing actions should the nurse use first to facilitate their coping with this situation (select all that apply)? A. Maintain hope. B. Exhibit a caring attitude. C. Plan realistic long-term goals. D. Give them antianxiety medications. E. Be available to listen to fears and concerns. F. Teach them about all the types of cancer that could be diagnosed.

A. Maintain hope. B. Exhibit a caring attitude. E. Be available to listen to fears and concerns. Maintaining hope, exhibiting a caring attitude, and being available to actively listen to fears and concerns would be the first nursing interventions to use as well as assessing factors affecting coping during the diagnostic period. Providing relief from distressing symptoms for the patient and teaching them about the diagnostic procedures would also be important. Realistic long-term goals and teaching about the type of cancer cannot be done until the cancer is diagnosed. Giving the family antianxiety medications would not be appropriate.

16. Glyburide (Micronase, DiaBeta, Glynase) is prescribed for a patient whose type 2 diabetes has not been controlled with diet and exercise. When teaching the patient about glyburide, the nurse explains that a. glyburide stimulates insulin production and release from the pancreas. b. the patient should not take glyburide for 48 hours after receiving IV contrast media. c. glyburide should be taken even when the blood glucose level is low in the morning. d. glyburide decreases glucagon secretion.

A. glyburide stimulates insulin production and release from the pancreas. Rationale: The sulfonylureas stimulate the production and release of insulin from the pancreas. If the glucose level is low, the patient should contact the health care provider before taking the glyburide, since hypoglycemia can occur with this category of medication. Metformin should be held for 48 hours after administration of IV contract, but this is not necessary for glyburide.

You are caring for a patient with newly diagnosed diabetes type 1. What information is essential to include in your patient teaching before discharge from the hospital? (Select all that apply) a) Insulin administration b) Elimination of sugar from diet c) Need to reduce physical exercise d) Use of a portable blood glucose monitor e) Hypoglycemia prevention, symptoms, and treatment

A. insulin administration D. use of a portable blood glucose monitor E. hypoglycemia prevention, symptoms, and treatment

14. A patient using a split mixed-dose insulin regimen asks the nurse about the use of intensive insulin therapy to achieve tighter glucose control. The nurse should teach the patient that a. intensive insulin therapy requires three or more injections a day in addition to an injection of a basal long-acting insulin. b. intensive insulin therapy is indicated only for newly diagnosed type 1 diabetics who have never experienced ketoacidosis. c. studies have shown that intensive insulin therapy is most effective in preventing the macrovascular complications characteristic of type 2 diabetes. d. an insulin pump provides the best glucose control and requires about the same amount of attention as intensive insulin therapy.

A. intensive insulin therapy requires three or more injections a day in addition to an injection of basal long-acting insulin Rationale: Patients using intensive insulin therapy must check their glucose level four to six times daily and administer insulin accordingly. A previous episode of ketoacidosis is not a contraindication for intensive insulin therapy. Intensive insulin therapy is not confined to type 2 diabetics and would prevent microvascular changes as well as macrovascular changes. Intensive insulin therapy and an insulin pump are comparable in glucose control.

When planning teaching for a patient with benign nephrosclerosis the nurse should include instructions regarding a. monitoring and recording blood pressure. b. obtaining and documenting daily weights. c. measuring daily intake and output amounts. d. preventing bleeding caused by anticoagulants.

A. monitoring and recording blood pressure Hypertension is the major symptom of nephrosclerosis. Measurements of intake and output and daily weights are not necessary unless the patient develops renal insufficiency. Anticoagulants are not used to treat nephrosclerosis.

12. A 24-year-old woman who uses injectable illegal drugs asks the nurse about preventing AIDS. The nurse informs the patient that the best way to reduce the risk of HIV infection from drug use is to a. participate in a needle-exchange program. b. clean drug injection equipment before use. c. ask those who share equipment to be tested for HIV. d. avoid sexual intercourse when using injectable drugs.

A. participate in a needle-exchange program Participation in needle-exchange programs has been shown to control the rate of HIV infection. Cleaning drug equipment before use also reduces risk, but it might not be consistently practiced by individuals in withdrawal. HIV antibodies do not appear for several weeks to months after exposure, so testing drug users would not be very effective in reducing risk for HIV exposure. It is difficult to make appropriate decisions about sexual activity when under the influence of drugs.

18. A patient with type 2 diabetes that is controlled with diet and metformin (Glucophage) also has severe rheumatoid arthritis (RA). During an acute exacerbation of the patient's arthritis, the health care provider prescribes prednisone (Deltasone) to control inflammation. The nurse will anticipate that the patient may a. require administration of insulin while taking prednisone. b. develop acute hypoglycemia during the RA exacerbation. c. have rashes caused by metformin-prednisone interactions. d. need a diet higher in calories while receiving prednisone.

A. require administration of insulin while taking prednisone Rationale: Glucose levels increase when patients are taking corticosteroids, and insulin may be required to control blood glucose. Hypoglycemia is not a complication of RA exacerbation or prednisone use. Rashes are not an adverse effect caused by taking metformin and prednisone simultaneously. The patient is likely to have an increased appetite when taking prednisone, but it will be important to avoid weight gain for the patient with RA.

1. A patient who has vague symptoms of fatigue and headaches is found to have a positive enzyme immunoassay (EIA) for human immunodeficiency virus (HIV) antibodies. In discussing the test results with the patient, the nurse informs the patient that a. the EIA test will need to be repeated to verify the results. b. a viral culture will be done to determine the progress of the disease. c. it will probably be 10 or more years before the patient develops acquired immunodeficiency syndrome (AIDS). d. the Western blot test will be done to determine whether AIDS has developed.

A. the EIA test will need to be repeated to verify the results After an initial positive EIA test, the EIA is repeated before more specific testing such as the Western blot is done. Viral cultures are not part of HIV testing. Because the nurse does not know how recently the patient was infected, it is not appropriate to predict the time frame for AIDS development. The Western blot tests for HIV antibodies, not for AIDS.

36. After the home health nurse has taught a patient and family about how to use glargine and regular insulin safely, which action by the patient indicates that the teaching has been successful? a. The patient disposes of the open insulin vials after 4 weeks. b. The patient draws up the regular insulin in the syringe and then draws up the glargine. c. The patient stores extra vials of both types of insulin in the freezer until needed. d. The patient's family prefills the syringes weekly and stores them in the refrigerator.

A. the patient disposes of the open insulin vials after 4 weeks. Rationale: Insulin can be stored at room temperature for 4 weeks. Glargine should not be mixed with other insulins or prefilled and stored. Freezing alters the insulin molecule and should not be done.

Following rectal surgery, a patient voids about 50 mL of urine every 30 to 60 minutes. Which nursing action is most appropriate? a. Use an ultrasound scanner to check the postvoiding residual. b. Monitor the patient's intake and output over the next few hours. c. Have the patient take small amounts of fluid frequently throughout the day. d. Reassure the patient that this is normal after rectal surgery because of anesthesia.

A. use an ultrasound scanner to check the post voiding residual An ultrasound scanner can be used to check for residual urine after the patient voids. Because the patient's history and clinical manifestations are consistent with overflow incontinence, it is not appropriate to have the patient drink small amounts. Although overflow incontinence is not unusual after surgery, the nurse should intervene to correct the physiologic problem, not just reassure the patient. The patient may develop reflux into the renal pelvis as well as discomfort from a full bladder if the nurse waits to address the problem for several hours.

36. The health care provider prescribes the following therapies for a patient who has been admitted with dehydration and hypotension after 3 days of nausea and vomiting. Which order will the nurse implement first? a. Infuse normal saline at 250 mL/hr. b. Administer IV ondansetron (Zofran). c. Provide oral care with moistened swabs. d. Insert a 16-gauge nasogastric (NG) tube.

ANS: A Because the patient has severe dehydration, rehydration with IV fluids is the priority. The other orders should be accomplished as quickly as possible after the IV fluids are initiated.

Which integumentary assessment data from an older patient admitted with bacterial pneumonia is of most concern for the nurse? a. Reports a history of allergic rashes b. Scattered macular brown areas on extremities c. Skin brown and wrinkled, skin tenting on forearm d. Longitudinal nail bed ridges noted; sparse scalp hair

ANS: A Because the patient will be receiving antibiotics to treat the pneumonia, the nurse should be most concerned about her history of allergic rashes. The nurse needs to do further assessment of possible causes of the allergic rashes and whether she has ever had allergic reactions to any drugs, especially antibiotics. The assessment data in the other response would be normal for an older patient

A patient who is diagnosed with cervical cancer that is classified as Tis, N0, M0 asks the nurse what the letters and numbers mean. Which response by the nurse is most appropriate? a. "The cancer involves only the cervix." b. "The cancer cells look almost like normal cells." c. "Further testing is needed to determine the spread of the cancer." d. "It is difficult to determine the original site of the cervical cancer."

ANS: A Cancer in situ indicates that the cancer is localized to the cervix and is not invasive at this time. Cell differentiation is not indicated by clinical staging. Because the cancer is in situ, the origin is the cervix. Further testing is not indicated given that the cancer has not spread.

A patient who is diagnosed with cervical cancer that is classified as Tis, N0, M0 asks the nurse what the letters and numbers mean. Which response by the nurse is most appropriate? a. "The cancer involves only the cervix." b. "The cancer cells look almost like normal cells." c. "Further testing is needed to determine the spread of the cancer." d. "It is difficult to determine the original site of the cervical cancer."

ANS: A Cancer in situ indicates that the cancer is localized to the cervix and is not invasive at this time. Cell differentiation is not indicated by clinical staging. Because the cancer is in situ, the origin is the cervix. Further testing is not indicated given that the cancer has not spread. DIF: Cognitive Level: Apply (application) REF: 254 TOP: Nursing Process: Implementation MSC: NCLEX: Physiological Integrity

16. Cobalamin injections have been prescribed for a patient with chronic atrophic gastritis. The nurse determines that teaching regarding the injections has been effective when the patient states, a. "The cobalamin injections will prevent me from becoming anemic." b. "These injections will increase the hydrochloric acid in my stomach." c. "These injections will decrease my risk for developing stomach cancer." d. "The cobalamin injections need to be taken until my inflamed stomach heals."

ANS: A Cobalamin supplementation prevents the development of pernicious anemia. The incidence of stomach cancer is higher in patients with chronic gastritis, but cobalamin does not reduce the risk for stomach cancer. Chronic gastritis may cause achlorhydria, but cobalamin does not correct this. The loss of intrinsic factor secretion with chronic gastritis is permanent, and the patient will need lifelong supplementation with cobalamin.

A patient arrives in the emergency department with ankle swelling and severe pain after twisting the ankle playing soccer. Which of these prescribed collaborative interventions will the nurse implement first? a. Wrap the ankle and apply an ice pack. b. Administer naproxen (Naprosyn) 500 mg PO. c. Give acetaminophen with codeine (Tylenol #3). d. Take the patient to the radiology department for x-rays.

ANS: A Immediate care after a sprain or strain injury includes the application of cold and compression to the injury to minimize swelling. The other actions should be taken after the ankle is wrapped with a compression bandage and ice is applied.

A patient with lower leg fracture has an external fixation device in place and is scheduled for discharge. Which information will the nurse include in the discharge teaching? a. "You will need to assess and clean the pin insertion sites daily." b. "The external fixator can be removed during the bath or shower." c. "You will need to remain on bed rest until bone healing is complete." d. "Prophylactic antibiotics are used until the external fixator is removed."

ANS: A Pin insertion sites should be cleaned daily to decrease the risk for infection at the site. An external fixator allows the patient to be out of bed and avoid the risks of prolonged immobility. The device is surgically placed and is not removed until the bone is stable. Prophylactic antibiotics are not routinely given when an external fixator is used.

21. The health care provider orders intravenous (IV) ranitidine (Zantac) for a patient with gastrointestinal (GI) bleeding caused by peptic ulcer disease. When teaching the patient about the effect of the medication, which information will the nurse include? a. "Ranitidine decreases secretion of gastric acid." b. "Ranitidine neutralizes the acid in the stomach." c. "Ranitidine constricts the blood vessels in the stomach and decreases bleeding." d. "Ranitidine covers the ulcer with a protective material that promotes healing."

ANS: A Ranitidine is a histamine-2 (H2) receptor blocker, which decreases the secretion of gastric acid. The response beginning, "Ranitidine constricts the blood vessels" describes the effect of vasopressin. The response beginning "Ranitidine neutralizes the acid" describes the effect of antacids. And the response beginning "Ranitidine covers the ulcer" describes the action of sucralfate (Carafate).

The occupational health nurse will teach the patient whose job involves many hours of typing about the need to a. obtain a keyboard pad to support the wrist. b. do stretching exercises before starting work. c. wrap the wrists with compression bandages every morning. d. avoid using nonsteroidal antiinflammatory drugs (NSAIDs) for pain.

ANS: A Repetitive strain injuries caused by prolonged times working at a keyboard can be prevented by the use of a pad that will keep the wrists in a straight position. Stretching exercises during the day may be helpful, but these would not be needed before starting. Use of a compression bandage is not needed, although a splint may be used for carpal tunnel syndrome. NSAIDs are appropriate to use to decrease swelling.

The nurse assesses a patient with non-Hodgkin's lymphoma who is receiving an infusion of rituximab (Rituxan). Which assessment finding would require the most rapid action by the nurse? a. Shortness of breath b. Temperature 100.2° F (37.9° C) c. Shivering and complaint of chills d. Generalized muscle aches and pains

ANS: A Rituximab (Rituxan) is a monoclonal antibody. Shortness of breath should be investigated rapidly because anaphylaxis is a possible reaction to monoclonal antibody administration. The nurse will need to rapidly take actions such as stopping the infusion, assessing the patient further, and notifying the health care provider. The other findings will also require action by the nurse, but are not indicative of life-threatening complications.

32. When counseling a patient with a family history of stomach cancer about ways to decrease risk for developing stomach cancer, the nurse will teach the patient to avoid a. smoked foods such as bacon and ham. b. foods that cause abdominal distention. c. chronic use of H2 blocking medications. d. emotionally or physically stressful situations.

ANS: A Smoked foods such as bacon, ham, and smoked sausage increase the risk for stomach cancer. Use of H2 blockers, stressful situations, and abdominal distention are not associated with an increased incidence of stomach cancer.

30. Which information will be best for the nurse to include when teaching a patient with peptic ulcer disease (PUD) about dietary management of the disease? a. "Avoid foods that cause pain after you eat them." b. "High-protein foods are least likely to cause pain." c. "You will need to remain on a bland diet indefinitely." d. "You should avoid eating many raw fruits and vegetables."

ANS: A The best information is that each individual should choose foods that are not associated with postprandial discomfort. Raw fruits and vegetables may irritate the gastric mucosa, but chewing well seems to decrease this and some patients may tolerate these well. High-protein foods help to neutralize acid, but they also stimulate hydrochloric (HCl) acid secretion and may increase discomfort for some patients. Bland diets may be recommended during an acute exacerbation of PUD, but there is little scientific evidence to support their use.

A patient with ulnar drift caused by rheumatoid arthritis (RA) is scheduled for an arthroplasty of the hand. Which patient statement to the nurse indicates realistic expectation for the surgery? a. "I will be able to use my fingers to grasp objects better." b. "I will not have to do as many hand exercises after the surgery." c. "This procedure will prevent further deformity in my hands and fingers." d. "My fingers will appear more normal in size and shape after this surgery."

ANS: A The goal of hand surgery in RA is to restore function, not to correct for cosmetic deformity or treat the underlying process. Hand exercises will be prescribed after the surgery.

A 32-year-old patient who has had an open reduction and internal fixation (ORIF) of left lower leg fractures continues to complain of severe pain in the leg 15 minutes after receiving the prescribed IV morphine. Pulses are faintly palpable and the foot is cool. Which action should the nurse take next? a. Notify the health care provider. b. Assess the incision for redness. c. Reposition the left leg on pillows. d. Check the patient's blood pressure.

ANS: A The patient's clinical manifestations suggest compartment syndrome and delay in diagnosis and treatment may lead to severe functional impairment. The data do not suggest problems with blood pressure or infection. Elevation of the leg will decrease arterial flow and further reduce perfusion.

A patient who has had an open reduction and internal fixation (ORIF) of left lower leg fractures complains of constant severe pain in the leg, which is unrelieved by the prescribed morphine. Pulses are faintly palpable and the foot is cool. Which action should the nurse take next? a. Notify the health care provider. b. Assess the incision for redness. c. Reposition the left leg on pillows. d. Check the patient's blood pressure.

ANS: A The patient's clinical manifestations suggest compartment syndrome and delay in diagnosis and treatment may lead to severe functional impairment. The data do not suggest problems with blood pressure or infection. Elevation of the leg will decrease arterial flow and further reduce perfusion.

23. A patient with a bleeding duodenal ulcer has a nasogastric (NG) tube in place, and the health care provider orders 30 mL of aluminum hydroxide/magnesium hydroxide (Maalox) to be instilled through the tube every hour. To evaluate the effectiveness of this treatment, the nurse a. periodically aspirates and tests gastric pH. b. monitors arterial blood gas values on a daily basis. c. checks each stool for the presence of occult blood. d. measures the amount of residual stomach contents hourly.

ANS: A The purpose for antacids is to increase gastric pH. Checking gastric pH is the most direct way of evaluating the effectiveness of the medication. Arterial blood gases may change slightly, but this does not directly reflect the effect of antacids on gastric pH. Because the patient has upper gastrointestinal (GI) bleeding, occult blood in the stools will appear even after the acute bleeding has stopped. The amount of residual stomach contents is not a reflection of resolution of bleeding or of gastric pH.

A widowed mother of four school-age children is hospitalized with metastatic ovarian cancer. The patient is crying and tells the nurse that she does not know what will happen to her children when she dies. Which response by the nurse is most appropriate? a. "Why don't we talk about the options you have for the care of your children?" b. "I'm sure you have friends that will take the children when you can't care for them." c. "For now you need to concentrate on getting well and not worrying about your children." d. "Many patients with cancer live for a long time, so there is still time to plan for your children."

ANS: A This response expresses the nurse's willingness to listen and recognizes the patient's concern. The responses beginning "Many patients with cancer live for a long time" and "For now you need to concentrate on getting well" close off discussion of the topic and indicate that the nurse is uncomfortable with the topic. In addition, the patient with metastatic ovarian cancer may not have a long time to plan. Although it is possible that the patient's friends will take the children, more assessment information is needed before making plans.

When caring for a patient who is pancytopenic, which action by unlicensed assistive personnel (UAP) indicates a need for the nurse to intervene? a. The UAP assists the patient to use dental floss after eating. b. The UAP adds baking soda to the patient's saline oral rinses. c. The UAP puts fluoride toothpaste on the patient's toothbrush. d. The UAP has the patient rinse after meals with a saline solution.

ANS: A Use of dental floss is avoided in patients with pancytopenia because of the risk for infection and bleeding. The other actions are appropriate for oral care of a pancytopenic patient.

The nurse at the clinic is interviewing a 64-year-old woman who is 5 feet, 3 inches tall and weighs 125 pounds (57 kg). The patient has not seen a health care provider for 20 years. She walks 5 miles most days and has a glass of wine 2 or 3 times a week. Which topics will the nurse plan to include in patient teaching about cancer screening and decreasing cancer risk (select all that apply)? a. Pap testing b. Tobacco use c. Sunscreen use d. Mammography e. Colorectal screening

ANS: A, C, D, E The patient's age, gender, and history indicate a need for screening and/or teaching about colorectal cancer, mammography, Pap smears, and sunscreen. The patient does not use excessive alcohol or tobacco, she is physically active, and her body weight is healthy.

The nurse at the clinic is interviewing a 64-year-old woman who is 5 feet, 3 inches tall and weighs 125 pounds (57 kg). The patient has not seen a health care provider for 20 years. She walks 5 miles most days and has a glass of wine 2 or 3 times a week. Which topics will the nurse plan to include in patient teaching about cancer screening and decreasing cancer risk (select all that apply)? a. Pap testing b. Tobacco use c. Sunscreen use d. Mammography e. Colorectal screening

ANS: A, C, D, E The patient's age, gender, and history indicate a need for screening and/or teaching about colorectal cancer, mammography, Pap smears, and sunscreen. The patient does not use excessive alcohol or tobacco, she is physically active, and her body weight is healthy. DIF: Cognitive Level: Analyze (analysis) REF: 255 TOP: Nursing Process: Planning MSC: NCLEX: Health Promotion and Maintenance

A patient develops neutropenia after receiving chemotherapy. Which information about ways to prevent infection will the nurse include in the teaching plan (select all that apply)? a. Cook food thoroughly before eating. b. Choose low fiber, low residue foods. c. Avoid public transportation such as buses. d. Use rectal suppositories if needed for constipation. e. Talk to the oncologist before having any dental work done.

ANS: A, C, E Eating only cooked food and avoiding public transportation will decrease infection risk. A high-fiber diet is recommended for neutropenic patients to decrease constipation. Because bacteria may enter the circulation during dental work or oral surgery, the patient may need to postpone dental work or take antibiotics.

A patient develops neutropenia after receiving chemotherapy. Which information about ways to prevent infection will the nurse include in the teaching plan (select all that apply)? a. Cook food thoroughly before eating. b. Choose low fiber, low residue foods. c. Avoid public transportation such as buses. d. Use rectal suppositories if needed for constipation. e. Talk to the oncologist before having any dental work done.

ANS: A, C, E Eating only cooked food and avoiding public transportation will decrease infection risk. A high-fiber diet is recommended for neutropenic patients to decrease constipation. Because bacteria may enter the circulation during dental work or oral surgery, the patient may need to postpone dental work or take antibiotics. DIF: Cognitive Level: Apply (application) REF: 265 | eTable 16-16 TOP: Nursing Process: Planning MSC: NCLEX: Physiological Integrity

A patient is seen at the urgent care center after falling on the right arm and shoulder. Which finding is most important for the nurse to communicate to the health care provider? a. There is bruising at the shoulder area. b. The right arm appears shorter than the left. c. There is decreased range of motion of the shoulder. d. The patient is complaining of arm and shoulder pain.

ANS: B A shorter limb after a fall indicates a possible dislocation, which is an orthopedic emergency. Bruising, pain, and decreased range of motion also should be reported, but these do not indicate that emergent treatment is needed to preserve function.

Which nursing intervention will be included in the plan of care after a patient with a right femur fracture has a hip spica cast applied? a. Avoid placing the patient in prone position. b. Ask the patient about abdominal discomfort. c. Discuss remaining on bed rest for several weeks. d. Use the cast support bar to reposition the patient.

ANS: B Assessment of bowel sounds, abdominal pain, and nausea and vomiting will detect the development of cast syndrome. To avoid breakage, the support bar should not be used for repositioning. After the cast dries, the patient can begin ambulating with the assistance of physical therapy personnel and may be turned to the prone position

A 38-year-old female patient states that she is using topical fluorouracil to treat actinic keratoses on her face. Which additional assessment information will be most important for the nurse to obtain? a. History of sun exposure by the patient b. Method of birth control used by the patient c. Length of time the patient has used fluorouracil d. Appearance of the treated areas on the patient's face

ANS: B Because fluorouracil is teratogenic, it is essential that the patient use a reliable method of birth control. The other information is also important for the nurse to obtain, but lack of reliable birth control has the most potential for serious adverse medication effects

A 24-year-old female says she wants to begin using oral contraceptives. Which information from the nursing assessment is most important to report to the health care provider? a. The patient quit smoking 5 months previously. b. The patient's blood pressure is 154/86 mm Hg. c. The patient has not been vaccinated for rubella. d. The patient has chronic iron-deficiency anemia.

ANS: B Because hypertension increases the risk for morbidity and mortality in women taking oral contraceptives, the patient's blood pressure should be controlled before oral contraceptives are prescribed. The other information also will be reported but will not affect the choice of contraceptive

A patient who has a cast in place after fracturing the radius asks when the cast can be removed. The nurse will instruct the patient that the cast will need to remain in place a. for several months. b. for at least 3 weeks. c. until swelling of the wrist has resolved. d. until x-rays show complete bony union.

ANS: B Bone healing starts immediately after the injury, but since ossification does not begin until 3 weeks postinjury, the cast will need to be worn for at least 3 weeks. Complete union may take up to a year. Resolution of swelling does not indicate bone healing.

The nurse will instruct the patient with a fractured left radius that the cast will need to remain in place a. for several months. b. for at least 3 weeks. c. until swelling of the wrist has resolved. d. until x-rays show complete bony union.

ANS: B Bone healing starts immediately after the injury, but since ossification does not begin until 3 weeks postinjury, the cast will need to be worn for at least 3 weeks. Complete union may take up to a year. Resolution of swelling does not indicate bone healing.

Which action will the nurse take in order to evaluate the effectiveness of Buck's traction for a 62-year-old patient who has an intracapsular fracture of the right femur? a. Check peripheral pulses. b. Ask about hip pain level. c. Assess for hip contractures. d. Monitor for hip dislocation.

ANS: B Buck's traction keeps the leg immobilized and reduces painful muscle spasm. Hip contractures and dislocation are unlikely to occur in this situation. The peripheral pulses will be assessed, but this does not help in evaluating the effectiveness of Buck's traction.

A hospitalized patient who has received chemotherapy for leukemia develops neutropenia. Which observation by the nurse would indicate a need for further teaching? a. The patient ambulates several times a day in the room. b. The patient's visitors bring in some fresh peaches from home. c. The patient cleans with a warm washcloth after having a stool. d. The patient uses soap and shampoo to shower every other day.

ANS: B Fresh, thinned-skin fruits are not permitted in a neutropenic diet because of the risk of bacteria being present. The patient should ambulate in the room rather than the hospital hallway to avoid exposure to other patients or visitors. Because overuse of soap can dry the skin and increase infection risk, showering every other day is acceptable. Careful cleaning after having a bowel movement will help prevent skin breakdown and infection.

The nurse assesses a circular, flat, reddened lesion about 5 cm in diameter on a middle-aged patient's ankle. How should the nurse determine if the lesion is related to intradermal bleeding? a. Elevate the patient's leg. b. Press firmly on the lesion. c. Check the temperature of the skin around the lesion. d. Palpate the dorsalis pedis and posterior tibial pulses.

ANS: B If the lesion is caused by intradermal or subcutaneous bleeding or a nonvascular cause, the discoloration will remain when direct pressure is applied to the lesion. If the lesion is caused by blood vessel dilation, blanching will occur with direct pressure. The other assessments will assess circulation to the leg, but will not be helpful in determining the etiology of the lesion

A young man arrives in the emergency department with ankle swelling and severe pain after twisting his ankle playing basketball. Which of these prescribed collaborative interventions will the nurse implement first? a. Take the patient to have x-rays. b. Wrap the ankle and apply an ice pack. c. Administer naproxen (Naprosyn) 500 mg PO. d. Give acetaminophen with codeine (Tylenol #3).

ANS: B Immediate care after a sprain or strain injury includes the application of cold and compression to the injury to minimize swelling. The other actions should be taken after the ankle is wrapped with a compression bandage and ice is applied

A patient is being discharged 4 days after hip replacement surgery using the posterior approach. Which patient action requires immediate intervention by the nurse? a. The patient uses crutches with a swing-to gait. b. The patient leans over to pull shoes and socks on. c. The patient sits straight up on the edge of the bed. d. The patient bends over the sink while brushing teeth.

ANS: B Leaning over would flex the hip at greater than 90 degrees and predispose the patient to hip dislocation. The other patient actions are appropriate and do not require any immediate action by the nurse to protect the patient

A patient has hip replacement surgery using the posterior approach. Which patient action requires rapid intervention by the nurse? a. The patient uses crutches with a swing-to gait. b. The patient leans over to pull shoes and socks on. c. The patient sits straight up on the edge of the bed. d. The patient bends over the sink while brushing the teeth.

ANS: B Leaning over would flex the hip at greater than 90 degrees and predispose the patient to hip dislocation. The other patient actions are appropriate and do not require any immediate action by the nurse to protect the patient.

A 22-year-old tennis player has an arthroscopic repair of a rotator cuff injury performed in same-day surgery. When the nurse plans postoperative teaching for the patient, which information will be included? a. "You will not be able to serve a tennis ball again." b. "You will work with a physical therapist tomorrow." c. "The doctor will use the drop-arm test to determine the success of surgery." d. "Leave the shoulder immobilizer on for the first 4 days to minimize pain."

ANS: B Physical therapy after a rotator cuff repair begins on the first postoperative day to prevent "frozen shoulder." A shoulder immobilizer is used immediately after the surgery, but leaving the arm immobilized for several days would lead to loss of range of motion (ROM). The drop-arm test is used to test for rotator cuff injury, but not after surgery. The patient may be able to return to pitching after rehabilitation.

A patient who is being treated for stage IV lung cancer tells the nurse about new-onset back pain. Which action should the nurse take first? a. Give the patient the prescribed PRN opioid. b. Assess for sensation and strength in the legs. c. Notify the health care provider about the symptoms. d. Teach the patient how to use relaxation to reduce pain.

ANS: B Spinal cord compression, an oncologic emergency, can occur with invasion of tumor into the epidural space. The nurse will need to assess the patient further for symptoms such as decreased leg sensation and strength and then notify the health care provider. Administration of opioids or use of relaxation may be appropriate but only after the nurse has assessed for possible spinal cord compression.

When assessing for Tinel's sign in a patient with possible right-sided carpal tunnel syndrome, the nurse will ask the patient about a. weakness in the right little finger. b. tingling in the right thumb and fingers. c. burning in the right elbow and forearm. d. tremor when gripping with the right hand.

ANS: B Testing for Tinel's sign will cause tingling in the thumb and first three fingers of the affected hand in patients who have carpal tunnel syndrome. The median nerve does not innervate the right little finger or elbow and forearm. Tremor is not associated with carpal tunnel syndrome.

A patient with a complex pelvic fracture from a motor vehicle crash is on bed rest. Which nursing assessment finding is important to report to the health care provider? a. The patient states that the pelvis feels unstable. b. Abdomen is distended and bowel sounds are absent. c. There are ecchymoses across the abdomen and hips. d. The patient complains of pelvic pain with palpation.

ANS: B The abdominal distention and absent bowel sounds may be due to complications of pelvic fractures such as paralytic ileus or hemorrhage or trauma to the bladder, urethra, or colon. Pelvic instability, abdominal pain with palpation, and abdominal bruising would be expected with this type of injury.

When the nurse is caring for a patient who is on bed rest after having a complex pelvic fracture, which assessment finding is most important to report to the health care provider? a. The patient states that the pelvis feels unstable. b. Abdominal distention is present and bowel tones are absent. c. There are ecchymoses on the abdomen and hips. d. The patient complains of pelvic pain with palpation.

ANS: B The abdominal distention and absent bowel tones may be due to complications of pelvic fractures such as paralytic ileus or hemorrhage or trauma to the bladder, urethra, or colon. Pelvic instability, abdominal pain with palpation, and abdominal bruising would be expected with this type of injury.

17. A patient with peptic ulcer disease associated with the presence of Helicobacter pylori is treated with triple drug therapy. The nurse will plan to teach the patient about a. sucralfate (Carafate), nystatin (Mycostatin), and bismuth (Pepto-Bismol). b. amoxicillin (Amoxil), clarithromycin (Biaxin), and omeprazole (Prilosec). c. famotidine (Pepcid), magnesium hydroxide (Mylanta), and pantoprazole (Protonix). d. metoclopramide (Reglan), bethanechol (Urecholine), and promethazine (Phenergan).

ANS: B The drugs used in triple drug therapy include a proton pump inhibitor such as omeprazole and the antibiotics amoxicillin and clarithromycin. The other combinations listed are not included in the protocol for H. pylori infection.

A patient who has had an open reduction and internal fixation (ORIF) of a hip fracture tells the nurse that he is ready to get out of bed for the first time. Which action should the nurse take? a. Use a mechanical lift to transfer the patient from the bed to the chair. b. Check the postoperative orders for the patient's weight-bearing status. c. Avoid administration of pain medications before getting the patient up. d. Delegate the transfer of the patient to nursing assistive personnel (NAP).

ANS: B The nurse should be familiar with the weight-bearing orders for the patient before attempting the transfer. Mechanical lifts are not typically needed after this surgery. Pain medications should be given because the movement is likely to be painful for the patient. The registered nurse (RN) should supervise the patient during the initial transfer to evaluate how well the patient is able to accomplish this skill.

The nurse is preparing to assist a patient who has had an open reduction and internal fixation (ORIF) of a hip fracture out of bed for the first time. Which action should the nurse take? a. Use a mechanical lift to transfer the patient from the bed to the chair. b. Check the postoperative orders for the patient's weight-bearing status. c. Avoid administration of pain medications before getting the patient up. d. Delegate the transfer of the patient out of bed to nursing assistive personnel (NAP).

ANS: B The nurse should be familiar with the weight-bearing orders for the patient before attempting the transfer. Mechanical lifts are not typically needed after this surgery. Pain medications should be given, since the movement is likely to be painful for the patient. The RN should supervise the patient during the initial transfer to evaluate how well the patient is able to accomplish this skill.

A patient undergoes a left above-the-knee amputation with an immediate prosthetic fitting. When the patient arrives on the orthopedic unit after surgery, the nurse should a. place the patient in a prone position. b. check the surgical site for hemorrhage. c. remove the prosthesis and wrap the site. d. keep the residual leg elevated on a pillow.

ANS: B The nurse should monitor for hemorrhage after the surgery. The prosthesis will not be removed. To avoid flexion contracture of the hip, the leg will not be elevated on a pillow. The patient is placed in a prone position after amputation to prevent hip flexion, but this would not be done during the immediate postoperative period.

A chemotherapy drug that causes alopecia is prescribed for a patient. Which action should the nurse take to maintain the patient's self-esteem? a. Tell the patient to limit social contacts until regrowth of the hair occurs. b. Encourage the patient to purchase a wig or hat and wear it once hair loss begins. c. Teach the patient to gently wash hair with a mild shampoo to minimize hair loss. d. Inform the patient that hair usually grows back once the chemotherapy is complete.

ANS: B The patient is taught to anticipate hair loss and to be prepared with wigs, scarves, or hats. Limiting social contacts is not appropriate at a time when the patient is likely to need a good social support system. The damage occurs at the hair follicle and will occur regardless of gentle washing or use of a mild shampoo. The information that the hair will grow back is not immediately helpful in maintaining the patient's self-esteem.

Which statement by a 62-year-old patient who has had an above-the-knee amputation indicates that the nurse's discharge teaching has been effective? a. "I should elevate my residual limb on a pillow 2 or 3 times a day." b. "I should lay flat on my abdomen for 30 minutes 3 or 4 times a day." c. "I should change the limb sock when it becomes soiled or each week." d. "I should use lotion on the stump to prevent skin drying and cracking."

ANS: B The patient lies in the prone position several times daily to prevent flexion contractures of the hip. The limb sock should be changed daily. Lotion should not be used on the stump. The residual limb should not be elevated because this would encourage flexion contracture

Before assisting a patient with ambulation 2 days after a total hip replacement, which action is most important for the nurse to take? a. Observe the status of the incisional drain device. b. Administer the ordered oral opioid pain medication. c. Instruct the patient about the benefits of ambulation. d. Change the hip dressing and document the wound appearance.

ANS: B The patient should be adequately medicated for pain before any attempt to ambulate. Instructions about the benefits of ambulation may increase the patient's willingness to ambulate, but decreasing pain with ambulation is more important. The presence of an incisional drain or timing of dressing change will not affect ambulation.

24. A patient with a peptic ulcer who has a nasogastric (NG) tube develops sudden, severe upper abdominal pain, diaphoresis, and a very firm abdomen. Which action should the nurse take next? a. Irrigate the NG tube. b. Obtain the vital signs. c. Listen for bowel sounds. d. Give the ordered antacid.

ANS: B The patient's symptoms suggest acute perforation, and the nurse should assess for signs of hypovolemic shock. Irrigation of the NG tube, administration of antacids, or both would be contraindicated because any material in the stomach will increase the spillage into the peritoneal cavity. The nurse should assess the bowel sounds, but this is not the first action that should be taken.

After being hospitalized for 3 days with a right femur fracture, a 32-year-old patient suddenly develops shortness of breath and tachypnea. The patient tells the nurse, "I feel like I am going to die!" Which action should the nurse take first? a. Stay with the patient and offer reassurance. b. Administer the prescribed PRN oxygen at 4 L/min. c. Check the patient's legs for swelling or tenderness. d. Notify the health care provider about the symptoms.

ANS: B The patient's clinical manifestations and history are consistent with a pulmonary embolus, and the nurse's first action should be to ensure adequate oxygenation. The nurse should offer reassurance to the patient, but meeting the physiologic need for oxygen is a higher priority. The health care provider should be notified after the oxygen is started and pulse oximetry and assessment for fat embolus or venous thromboembolism (VTE) are obtained

A patient who has been hospitalized for 3 days with a hip fracture has sudden onset shortness of breath and tachypnea. The patient tells the nurse, "I feel like I am going to die!" Which action should the nurse take first? a. Stay with the patient and offer reassurance. b. Administer the prescribed PRN oxygen at 4 L/min. c. Check the patient's legs for swelling or tenderness. d. Notify the health care provider about the symptoms.

ANS: B The patient's clinical manifestations and history are consistent with a pulmonary embolus, and the nurse's first action should be to ensure adequate oxygenation. The nurse should offer reassurance to the patient, but meeting the physiologic need for oxygen is a higher priority. The health care provider should be notified after the oxygen is started and pulse oximetry and assessment for fat embolus or venous thromboembolism (VTE) are obtained.

The nurse should include which food choice when providing dietary teaching for a patient scheduled to receive external beam radiation for abdominal cancer? a. Fresh fruit salad b. Roasted chicken c. Whole wheat toast d. Cream of potato soup

ANS: B To minimize the diarrhea that is commonly associated with bowel radiation, the patient should avoid foods high in roughage, such as fruits and whole grains. Lactose intolerance may develop secondary to radiation, so dairy products should also be avoided

The nurse should include which food choice when providing dietary teaching for a patient scheduled to receive external beam radiation for abdominal cancer? a. Fresh fruit salad b. Roasted chicken c. Whole wheat toast d. Cream of potato soup

ANS: B To minimize the diarrhea that is commonly associated with bowel radiation, the patient should avoid foods high in roughage, such as fruits and whole grains. Lactose intolerance may develop secondary to radiation, so dairy products should also be avoided.

When caring for a patient who is using Buck's traction after a hip fracture, which action can the nurse delegate to unlicensed assistive personnel (UAP)? a. Monitor the skin under the traction boot for redness. b. Ensure that the weight for the traction is off the floor. c. Check for intact sensation and movement in the affected leg. d. Offer reassurance that hip and leg pain are normal after hip fracture.

ANS: B UAP can be responsible for maintaining the integrity of the traction once it has been established. Assessment of skin integrity and circulation should be done by the registered nurse (RN). UAP should notify the RN if the patient experiences hip and leg pain because pain and effectiveness of pain relief measures should be assessed by the RN.

A patient has a long-arm plaster cast applied for immobilization of a fractured left radius. Until the cast has completely dried, the nurse should a. keep the left arm in dependent position. b. avoid handling the cast using fingertips. c. place gauze around the cast edge to pad any roughness. d. cover the cast with a small blanket to absorb the dampness.

ANS: B Until a plaster cast has dried, using the palms rather than the fingertips to handle the cast helps prevent creating protrusions inside the cast that could place pressure on the skin. The left arm should be elevated to prevent swelling. The edges of the cast may be petaled once the cast is dry, but padding the edges before that may cause the cast to be misshapen. The cast should not be covered until it is dry because heat builds up during drying.

A patient with Hodgkin's lymphoma who is undergoing external radiation therapy tells the nurse, "I am so tired I can hardly get out of bed in the morning." Which intervention should the nurse add to the plan of care? a. Minimize activity until the treatment is completed. b. Establish time to take a short walk almost every day. c. Consult with a psychiatrist for treatment of depression. d. Arrange for delivery of a hospital bed to the patient's home.

ANS: B Walking programs are used to keep the patient active without excessive fatigue. Having a hospital bed does not necessarily address the fatigue. The better option is to stay as active as possible while combating fatigue. Fatigue is expected during treatment and is not an indication of depression. Minimizing activity may lead to weakness and other complications of immobility.

Which of the following conditions are associated with oversecretion of rennin? A. Alzheimer's disease B. Hypertension C. Diabetes mellitus D. Diabetes insipidus

ANS: B Renin is secreted when special cells in the DCT, called the macula densa, sense changes in blood volume and pressure. When the macula densa cells sense that blood volume is low, blood pressure is low, or blood sodium levels are low, renin is secreted. Renin then converts angiotensinogen into angiotensin I. This leads to a series of reactions that cause the secretion of the hormone aldosterone. This hormone increases kidney reabsorption of sodium and water, increasing blood pressure, blood volume, and blood sodium levels. Inappropriate or excessive renin secretion is a major cause of persistent hypertension.

1. A patient with deep partial-thickness burns experiences severe pain associated with nausea during dressing changes. Which action will be most useful in decreasing the patient's nausea? a. The patient NPO for 2 hours before and after dressing changes. b. Avoid performing dressing changes close to the patient's mealtimes. c. Administer the prescribed morphine sulfate before dressing changes. d. Give the ordered prochlorperazine (Compazine) before dressing changes.

ANS: C Because the patient's nausea is associated with severe pain, it is likely that it is precipitated by stress and pain. The best treatment will be to provide adequate pain medication before dressing changes. The nurse should avoid doing painful procedures close to mealtimes, but nausea/vomiting that occur at other times also should be addressed. Keeping the patient NPO does not address the reason for the nausea and vomiting and will have an adverse effect on the patient's nutrition. Administration of antiemetics is not the best choice for a patient with nausea caused by pain.

26. The nurse implements discharge teaching for a patient following a gastroduodenostomy for treatment of a peptic ulcer. Which patient statement indicates that the teaching has been effective? a. "Persistent heartburn is expected after surgery." b. "I will try to drink liquids along with my meals." c. "Vitamin supplements may be needed to prevent problems with anemia." d. "I will need to choose foods that are low in fat and high in carbohydrate."

ANS: C Cobalamin deficiency may occur after partial gastrectomy, and the patient may need to receive cobalamin via injections or nasal spray. Foods that have moderate fat and low carbohydrate should be chosen to prevent dumping syndrome. Ingestion of liquids with meals is avoided to prevent dumping syndrome. Although peptic ulcer disease may recur, persistent heartburn is not expected after surgery and the patient should call the health care provider if this occurs.

When teaching seniors at a community recreation center, which information will the nurse include about ways to prevent fractures? a. Tack down scatter rugs in the home. b. Most falls happen outside the home. c. Buy shoes that provide good support and are comfortable to wear. d. Range-of-motion exercises should be taught by a physical therapist.

ANS: C Comfortable shoes with good support will help decrease the risk for falls. Scatter rugs should be eliminated, not just tacked down. Activities of daily living provide range of motion exercise; these do not need to be taught by a physical therapist. Falls inside the home are responsible for many injuries

When counseling an older patient about ways to prevent fractures, which information will the nurse include? a. Tack down scatter rugs in the home. b. Most falls happen outside the home. c. Buy shoes that provide good support and are comfortable to wear. d. Range-of-motion exercises should be taught by a physical therapist.

ANS: C Comfortable shoes with good support will help decrease the risk for falls. Scatter rugs should be eliminated, not just tacked down. Activities of daily living provide range of motion exercise; these do not need to be taught by a physical therapist. Falls inside the home are responsible for many injuries.

Which question should the nurse ask when assessing a 60-year-old patient who has a history of benign prostatic hyperplasia (BPH)? a. "Have you noticed any unusual discharge from your penis?" b. "Has there been any change in your sex life in the last year?" c. "Has there been a decrease in the force of your urinary stream?" d. "Have you been experiencing any difficulty in achieving an erection?"

ANS: C Enlargement of the prostate blocks the urethra, leading to urinary changes such as a decrease in the force of the urinary stream. The other questions address possible problems with infection or sexual difficulties, but they would not be helpful in determining whether there were functional changes caused by BPH.

The nurse is caring for a patient who has been diagnosed with stage I cancer of the colon. When assessing the need for psychologic support, which question by the nurse will provide the most information? a. "How long ago were you diagnosed with this cancer?" b. "Do you have any concerns about body image changes?" c. "Can you tell me what has been helpful to you in the past when coping with stressful events?" d. "Are you familiar with the stages of emotional adjustment to a diagnosis like cancer of the colon?"

ANS: C Information about how the patient has coped with past stressful situations helps the nurse determine usual coping mechanisms and their effectiveness. The length of time since the diagnosis will not provide much information about the patient's need for support. The patient's knowledge of typical stages in adjustment to a critical diagnosis does not provide insight into patient needs for assistance. Because surgical interventions for stage I cancer of the colon may not cause any body image changes, this question is not appropriate at this time

The nurse is caring for a patient with left-sided lung cancer. Which finding would be most important for the nurse to report to the health care provider? a. Hematocrit 32% b. Pain with deep inspiration c. Serum sodium 126 mEq/L d. Decreased breath sounds on left side

ANS: C Syndrome of inappropriate antidiuretic hormone (and the resulting hyponatremia) is an oncologic metabolic emergency and will require rapid treatment in order to prevent complications such as seizures and coma. The other findings also require intervention, but are common in patients with lung cancer and not immediately life threatening.

The nurse is caring for a patient who has been diagnosed with stage I cancer of the colon. When assessing the need for psychologic support, which question by the nurse will provide the most information? a. "How long ago were you diagnosed with this cancer?" b. "Do you have any concerns about body image changes?" c. "Can you tell me what has been helpful to you in the past when coping with stressful events?" d. "Are you familiar with the stages of emotional adjustment to a diagnosis like cancer of the colon?"

ANS: C Information about how the patient has coped with past stressful situations helps the nurse determine usual coping mechanisms and their effectiveness. The length of time since the diagnosis will not provide much information about the patient's need for support. The patient's knowledge of typical stages in adjustment to a critical diagnosis does not provide insight into patient needs for assistance. Because surgical interventions for stage I cancer of the colon may not cause any body image changes, this question is not appropriate at this time.

A 42-year-old patient is admitted to the emergency department with a left femur fracture. Which information obtained by the nurse is most important to report to the health care provider? a. Ecchymosis of the left thigh b. Complaints of severe thigh pain c. Slow capillary refill of the left foot d. Outward pointing toes on the left foot

ANS: C Prolonged capillary refill may indicate complications such as arterial damage or compartment syndrome. The other findings are typical with a left femur fracture.

External-beam radiation is planned for a patient with cervical cancer. What instructions should the nurse give to the patient to prevent complications from the effects of the radiation? a. Test all stools for the presence of blood. b. Maintain a high-residue, high-fiber diet. c. Clean the perianal area carefully after every bowel movement. d. Inspect the mouth and throat daily for the appearance of thrush.

ANS: C Radiation to the abdomen will affect organs in the radiation path, such as the bowel, and cause frequent diarrhea. Careful cleaning of this area will help decrease the risk for skin breakdown and infection. Stools are likely to have occult blood from the inflammation associated with radiation, so routine testing of stools for blood is not indicated. Radiation to the abdomen will not cause stomatitis. A low-residue diet is recommended to avoid irritation of the bowel when patients receive abdominal radiation.

29. The health care provider prescribes antacids and sucralfate (Carafate) for treatment of a patient's peptic ulcer. The nurse will teach the patient to take a. antacids 30 minutes before the sucralfate. b. sucralfate at bedtime and antacids before meals. c. antacids after eating and sucralfate 30 minutes before eating. d. sucralfate and antacids together 30 minutes before each meal.

ANS: C Sucralfate is most effective when the pH is low and should not be given with or soon after antacid. Antacids are most effective when taken after eating. Administration of sucralfate 30 minutes before eating and antacids just after eating will ensure that both drugs can be most effective. The other regimens will decrease the effectiveness of the medications.

The nurse is caring for a patient who smokes 2 packs/day. To reduce the patient's risk of lung cancer, which action by the nurse is best? a. Teach the patient about the seven warning signs of cancer. b. Plan to monitor the patient's carcinoembryonic antigen (CEA) level. c. Discuss the risks associated with cigarettes during every patient encounter. d. Teach the patient about the use of annual chest x-rays for lung cancer screening.

ANS: C Teaching about the risks associated with cigarette smoking is recommended at every patient encounter because cigarette smoking is associated with multiple health problems. A tumor must be at least 0.5 cm large before it is detectable by current screening methods and may already have metastasized by that time. Oncofetal antigens such as CEA may be used to monitor therapy or detect tumor reoccurrence, but are not helpful in screening for cancer. The seven warning signs of cancer are actually associated with fairly advanced disease

After change-of-shift report on the oncology unit, which patient should the nurse assess first? a. Patient who has a platelet count of 82,000/µL after chemotherapy b. Patient who has xerostomia after receiving head and neck radiation c. Patient who is neutropenic and has a temperature of 100.5° F (38.1° C) d. Patient who is worried about getting the prescribed long-acting opioid on time

ANS: C Temperature elevation is an emergency in neutropenic patients because of the risk for rapid progression to severe infections and sepsis. The other patients also require assessments or interventions, but do not need to be assessed as urgently. Patients with thrombocytopenia do not have spontaneous bleeding until the platelets are 20,000/µL. Xerostomia does not require immediate intervention. Although breakthrough pain needs to be addressed rapidly, the patient does not appear to have breakthrough pain.

A patient who has a proximal humerus fracture that is immobilized with a left-sided long-arm cast and a sling is admitted to the medical-surgical unit. Which nursing intervention will be included in the plan of care? a. Use surgical net dressing to hang the arm from an IV pole. b. Immobilize the fingers on the left hand with gauze dressings. c. Assess the left axilla and change absorbent dressings as needed. d. Assist the patient in passive range of motion (ROM) for the right arm.

ANS: C The axilla can become excoriated when a sling is used to support the arm, and the nurse should check the axilla and apply absorbent dressings to prevent this. A patient with a sling would not have traction applied by hanging. The patient will be encouraged to move the fingers on the injured arm to maintain function and to help decrease swelling. The patient will do active ROM on the uninjured side.

A patient who slipped and fell in the shower at home has a proximal humerus fracture immobilized with a left-sided long-arm cast and a sling. Which nursing intervention will be included in the plan of care? a. Use surgical net dressing to hang the arm from an IV pole. b. Immobilize the fingers of the left hand with gauze dressings. c. Assess the left axilla and change absorbent dressings as needed. d. Assist the patient in passive range of motion (ROM) for the right arm.

ANS: C The axilla can become excoriated when a sling is used to support the arm, and the nurse should check the axilla and apply absorbent dressings to prevent this. A patient with a sling would not have traction applied by hanging. The patient will be encouraged to move the fingers on the injured arm to maintain function and to help decrease swelling. The patient will do active ROM on the uninjured side.

A pedestrian who was hit by a car is admitted to the emergency department with possible right lower leg fractures. The initial action by the nurse should be to a. elevate the right leg. b. splint the lower leg. c. check the pedal pulses. d. verify tetanus immunizations.

ANS: C The initial nursing action should be assessment of the neurovascular status of the injured leg. After assessment, the nurse may need to splint and elevate the leg, based on the assessment data. Information about tetanus immunizations should be done if there is an open wound.

After the health care provider has recommended amputation for a patient who has nonhealing ischemic foot ulcers, the patient tells the nurse that he would rather die than have an amputation. Which response by the nurse is best? a. "You are upset, but you may lose the foot anyway." b. "Many people are able to function with a foot prosthesis." c. "Tell me what you know about your options for treatment." d. "If you do not want an amputation, you do not have to have it."

ANS: C The initial nursing action should be to assess the patient's knowledge level and feelings about the options available. Discussion about the patient's option to not have the procedure, the seriousness of the condition, or rehabilitation after the procedure may be appropriate after the nurse knows more about the patient's current level of knowledge and emotional state.

After the health care provider has recommended an amputation for a patient who has ischemic foot ulcers, the patient tells the nurse, "If they want to cut off my foot, they should just shoot me instead." Which response by the nurse is best? a. "Many people are able to function normally with a foot prosthesis." b. "I understand that you are upset, but you may lose the foot anyway." c. "Tell me what you know about what your options for treatment are." d. "If you do not want the surgery, you do not have to have an amputation."

ANS: C The initial nursing action should be to assess the patient's knowledge level and feelings about the options available. Discussion about the patient's option to not have the procedure, the seriousness of the condition, or rehabilitation after the procedure may be appropriate after the nurse knows more about the patient's current level of knowledge and emotional state.

A patient who is scheduled for a right breast biopsy asks the nurse the difference between a benign tumor and a malignant tumor. Which answer by the nurse is correct? a. "Benign tumors do not cause damage to other tissues." b. "Benign tumors are likely to recur in the same location." c. "Malignant tumors may spread to other tissues or organs." d. "Malignant cells reproduce more rapidly than normal cells."

ANS: C The major difference between benign and malignant tumors is that malignant tumors invade adjacent tissues and spread to distant tissues and benign tumors never metastasize. The other statements are inaccurate. Both types of tumors may cause damage to adjacent tissues. Malignant cells do not reproduce more rapidly than normal cells. Benign tumors do not usually recur.

During a routine health examination, a 40-year-old patient tells the nurse about a family history of colon cancer. Which action should the nurse take next? a. Teach the patient about the need for a colonoscopy at age 50. b. Teach the patient how to do home testing for fecal occult blood. c. Obtain more information from the patient about the family history. d. Schedule a sigmoidoscopy to provide baseline data about the patient

ANS: C The patient may be at increased risk for colon cancer, but the nurse's first action should be further assessment. The other actions may be appropriate, depending on the information that is obtained from the patient with further questioning.

During a routine health examination, a 40-year-old patient tells the nurse about a family history of colon cancer. Which action should the nurse take next? a. Teach the patient about the need for a colonoscopy at age 50. b. Teach the patient how to do home testing for fecal occult blood. c. Obtain more information from the patient about the family history. d. Schedule a sigmoidoscopy to provide baseline data about the patient.

ANS: C The patient may be at increased risk for colon cancer, but the nurse's first action should be further assessment. The other actions may be appropriate, depending on the information that is obtained from the patient with further questioning.

11. A 62-year-old patient who has been diagnosed with esophageal cancer tells the nurse, "I know that my chances are not very good, but I do not feel ready to die yet." Which response by the nurse is most appropriate? a. "You may have quite a few years still left to live." b. "Thinking about dying will only make you feel worse." c. "Having this new diagnosis must be very hard for you." d. "It is important that you be realistic about your prognosis."

ANS: C This response is open-ended and will encourage the patient to further discuss feelings of anxiety or sadness about the diagnosis. Patients with esophageal cancer have only a low survival rate, so the response "You may have quite a few years still left to live" is misleading. The response beginning, "Thinking about dying" indicates that the nurse is not open to discussing the patient's fears of dying. And the response beginning, "It is important that you be realistic," discourages the patient from feeling hopeful, which is important to patients with any life-threatening diagnosis.

A patient with metastatic cancer of the colon experiences severe vomiting following each administration of chemotherapy. Which action, if taken by the nurse, is most appropriate? a. Have the patient eat large meals when nausea is not present. b. Offer dry crackers and carbonated fluids during chemotherapy. c. Administer prescribed antiemetics 1 hour before the treatments. d. Give the patient two ounces of a citrus fruit beverage during treatments.

ANS: C Treatment with antiemetics before chemotherapy may help prevent nausea. The patient should eat small, frequent meals. Offering food and beverages during chemotherapy is likely to cause nausea. The acidity of citrus fruits may be further irritating to the stomach.

Two hours after a closed percutaneous kidney biopsy, the client reports a dramatic increase in pain. What is the nurse's best first action? A. Reposition the client on the operative side. B. Administer prescribed opioid analgesic. C. Assess pulse rate and blood pressure. D. Check the Foley catheter for kinks.

ANS: C An increase in the intensity of pain after a percutaneous kidney biopsy is a symptom of internal hemorrhage.

With a renal threshold for glucose of 220 mg/dL, what is the expected response when a client has a blood glucose level of 400 mg/dL? A. 400 mg/dL of excreted glucose in the urine B. 220 mg/dL of excreted glucose in the urine C. 180 mg/dL of glucose is excreted in the urine D. No excreted glucose in the urine

ANS: C Blood glucose is freely filtered at the glomerulus. Therefore, if a client has a blood sugar level of 400 mg/dl, the filtrate in the proximal convoluted tubule will have a glucose concentration of 400 mg/dL. With a renal threshold of 220 mg/dl, a total of 220 mg/dL of the 400 mg/dL will be reabsorbed back into the systemic circulation, and the final urine will have a glucose concentration of 180 mg/dL.

The client is going home after urography. Which instruction or precaution should the nurse teach this client? A. "Avoid direct contact with the urine for 24 hours until the radioisotope clears." B. "You are likely to experience some dribbling of urine for several weeks after this procedure." C. "Be sure to drink at least 3 L of fluids today to help eliminate the dye faster." D. "Your skin may become slightly yellow-tinged from the dye used in this procedure."

ANS: C Dyes used in urography are potentially nephrotoxic.

6. What is the result of stimulation of erythropoietin production in the kidney tissue? A. Increased blood flow to the kidney B. Inhibition of vitamin D and loss of bone density C. Increased bone marrow production of red blood cells D. Inhibition of the active transport of sodium, leading to hyponatremi

ANS: C Erythropoietin is produced in the kidney and released in response to decreased oxygen tension in the renal blood supply. Erythropoietin stimulates red blood cell (RBC) production in the bone marrow.

Confirmed by palpation and x-ray study, the client's right kidney is lower than the left kidney. What is the nurse's interpretation of this finding? A. The client has a problem involving the right kidney. B. The client has a problem involving the left kidney. C. The client has both kidneys in the normal position. D. The client is at increased risk for kidney impairment.

ANS: C Normally, the right kidney is positioned somewhat lower than the left kidney. This anatomic difference in otherwise symmetric organs is caused by liver displacement. The significance of this difference is that the right kidney is easier to palpate in an adult than is the left kidney.

The nurse assesses a patient who is receiving interleukin-2. Which finding should the nurse report immediately to the health care provider? a. Generalized muscle aches b. Complaints of nausea and anorexia c. Oral temperature of 100.6° F (38.1° C) d. Crackles heard at the lower scapular border

ANS: D Capillary leak syndrome and acute pulmonary edema are possible toxic effects of interleukin-2. The patient may need oxygen and the nurse should rapidly notify the health care provider. The other findings are common side effects of interleukin-2.

6. The nurse is assessing a patient with gastroesophageal reflux disease (GERD) who is experiencing increasing discomfort. Which patient statement indicates that additional patient education about GERD is needed? a. "I take antacids between meals and at bedtime each night." b. "I sleep with the head of the bed elevated on 4-inch blocks." c. "I quit smoking several years ago, but I still chew a lot of gum." d. "I eat small meals throughout the day and have a bedtime snack."

ANS: D GERD is exacerbated by eating late at night, and the nurse should plan to teach the patient to avoid eating at bedtime. The other patient actions are appropriate to control symptoms of GERD.

When giving home care instructions to a patient who has multiple forearm fractures and a long-arm cast on the right arm, which information should the nurse include? a. Keep the hand immobile to prevent soft tissue swelling. b. Keep the right shoulder elevated on a pillow or cushion. c. Avoid the use of nonsteroidal anti-inflammatory drugs (NSAIDs) for the first 48 hours after the injury. d. Call the health care provider for increased swelling or numbness.

ANS: D Increased swelling or numbness may indicate increased pressure at the injury, and the health care provider should be notified immediately to avoid damage to nerves and other tissues. The patient should be encouraged to move the joints above and below the cast to avoid stiffness. There is no need to elevate the shoulder, although the forearm should be elevated to reduce swelling. NSAIDs are appropriate to treat pain after a fracture.

28. A patient who requires daily use of a nonsteroidal anti-inflammatory drug (NSAID) for management of severe rheumatoid arthritis has recently developed melena. The nurse will anticipate teaching the patient about a. substitution of acetaminophen (Tylenol) for the NSAID. b. use of enteric-coated NSAIDs to reduce gastric irritation. c. reasons for using corticosteroids to treat the rheumatoid arthritis. d. the benefits of misoprostol (Cytotec) in protecting the gastrointestinal (GI) mucosa.

ANS: D Misoprostol, a prostaglandin analog, reduces acid secretion and incidence of upper GI bleeding associated with NSAID use. Enteric coating of NSAIDs does not reduce the risk for GI bleeding. Corticosteroids increase the risk for ulcer development and will not be substituted for NSAIDs for this patient. Acetaminophen will not be effective in treating the patient's rheumatoid arthritis.

After a 26-year-old patient has been treated for pelvic inflammatory disease, the nurse will plan to teach about a. use of hormone therapy (HT). b. irregularities in the menstrual cycle. c. changes in secondary sex characteristics. d. possible difficulty with becoming pregnant.

ANS: D Pelvic inflammatory disease may cause scarring of the fallopian tubes and result in difficulty in fertilization or implantation of the fertilized egg. Because ovarian function is not affected, the patient will not require HT, have irregular menstrual cycles, or experience changes in secondary sex characteristics.

Which abnormality on the skin of an older patient is the priority to discuss immediately with the health care provider? a. Several dry, scaly patches on the face b. Numerous varicosities noted on both legs c. Dilation of small blood vessels on the face d. Petechiae present on the chest and abdomen

ANS: D Petechiae are caused by pinpoint hemorrhages and are associated with a variety of serious disorders such as meningitis and coagulopathies. The nurse should contact the patient's health care provider about this finding for further diagnostic follow-up. The other skin changes are associated with aging. Although the other changes will also require ongoing monitoring or intervention by the nurse, they do not indicate a need for urgent action

An older adult patient who has colorectal cancer is receiving IV fluids at 175 mL/hour in conjunction with the prescribed chemotherapy. Which finding by the nurse is most important to report to the health care provider? a. Patient complains of severe fatigue. b. Patient needs to void every hour during the day. c. Patient takes only 50% of meals and refuses snacks. d. Patient has audible crackles to the midline posterior chest.

ANS: D Rapid fluid infusions may cause heart failure, especially in older patients. The other findings are common in patients who have cancer and/or are receiving chemotherapy.

When working with a patient whose job involves many hours of word processing, the nurse will teach the patient about the need to a. do stretching and warm-up exercises before starting work. b. wrap the wrists with a compression bandage every morning. c. use acetaminophen (Tylenol) instead of nonsteroidal anti-inflammatory drugs (NSAIDs) for wrist pain. d. obtain a keyboard pad to support the wrist while word processing.

ANS: D Repetitive strain injuries caused by prolonged times working at a keyboard can be prevented by the use of a pad that will keep the wrists in a straight position. Stretching exercises during the day may be helpful, but these would not be needed before starting. Use of a compression bandage is not needed, although a splint may be used for carpal tunnel syndrome. NSAIDs are appropriate to use to decrease swelling.

Which finding in a patient with a Colles' fracture of the left wrist is most important to communicate to the health care provider? a. Swelling is noted around the wrist. b. The patient is reporting severe pain. c. The wrist has a deformed appearance. d. Capillary refill to the fingers is prolonged.

ANS: D Swelling, pain, and deformity are common findings with a Colles' fracture. Prolonged capillary refill indicates decreased circulation and risk for ischemia. This is not an expected finding and should be immediately reported

A patient who has ovarian cancer is crying and tells the nurse, "My husband rarely visits. He just doesn't care." The husband indicates to the nurse that he never knows what to say to help his wife. Which nursing diagnosis is most appropriate for the nurse to add to the plan of care? a. Compromised family coping related to disruption in lifestyle b. Impaired home maintenance related to perceived role changes c. Risk for caregiver role strain related to burdens of caregiving responsibilities d. Dysfunctional family processes related to effect of illness on family members

ANS: D The data indicate that this diagnosis is most appropriate because poor communication among the family members is affecting family processes. No data suggest a change in lifestyle or its role as an etiology. The data do not support impairment in home maintenance or a burden caused by caregiving responsibilities.

During assessment of the patient's skin, the nurse observes a similar pattern of small, raised lesions on the left and right upper back areas. Which term should the nurse use to document these lesions? a. Confluent b. Zosteriform c. Generalized d. Symmetric

ANS: D The description of the lesions indicates that they are grouped. The other terms are inconsistent with the description of the lesions

A patient in the emergency department who is experiencing severe pain is diagnosed with a patellar dislocation. The initial patient teaching by the nurse will focus on the need for a. a knee immobilizer. b. gentle knee flexion. c. activity restrictions. d. monitored anesthesia care (conscious sedation).

ANS: D The first goal of collaborative management is realignment of the knee to its original anatomic position, which will require anesthesia or monitored anesthesia care (MAC), formerly called conscious sedation. Immobilization, gentle range of motion (ROM) exercises, and discussion about activity restrictions will be implemented after the knee is realigned.

A patient is admitted to the emergency department with possible left lower leg fractures. The initial action by the nurse should be to a. elevate the left leg. b. splint the lower leg. c. obtain information about the tetanus immunization status. d. check the popliteal, dorsalis pedis, and posterior tibial pulses.

ANS: D The initial nursing action should be assessment of the neurovascular status of the injured leg. After assessment, the nurse may need to splint and elevate the leg, based on the assessment data. Information about tetanus immunizations should be done if there is an open wound.

A 68-year-old male patient tells the nurse that he is worried because he does not respond to sexual stimulation the same way he did when he was younger. The nurse's best response to the patient's concern is which of the following? a. "Interest in sex frequently decreases as men get older." b. "Many men need additional sexual stimulation with aging." c. "Erectile dysfunction is a common problem with older men." d. "Tell me more about how your sexual response has changed."

ANS: D The initial response by the nurse should be further assessment of the problem. The other statements by the nurse are accurate but may not respond to the patient's concerns

The nurse reviews the laboratory results of a patient who is receiving chemotherapy. Which laboratory result is most important to report to the health care provider? a. Hematocrit of 30% b. Platelets of 95,000/µL c. Hemoglobin of 10 g/L d. White blood cell (WBC) count of 2700/µL

ANS: D The low WBC count places the patient at risk for severe infection and is an indication that the chemotherapy dose may need to be lower or that WBC growth factors such as filgrastim (Neupogen) are needed. Although the other laboratory data indicate decreased levels, they do not indicate any immediate life-threatening adverse effects of the chemotherapy.

A 48-year-old patient with a comminuted fracture of the left femur has Buck's traction in place while waiting for surgery. To assess for pressure areas on the patient's back and sacral area and to provide skin care, the nurse should a. loosen the traction and help the patient turn onto the unaffected side. b. place a pillow between the patient's legs and turn gently to each side. c. turn the patient partially to each side with the assistance of another nurse. d. have the patient lift the buttocks by bending and pushing with the right leg.

ANS: D The patient can lift the buttocks off the bed by using the left leg without changing the right-leg alignment. Turning the patient will tend to move the leg out of alignment. Disconnecting the traction will interrupt the weight needed to immobilize and align the fracture.

A patient is to be discharged from the hospital 4 days after insertion of a femoral head prosthesis using a posterior approach. A statement by the patient that indicates a need for additional discharge instructions is a. "I should not cross my legs while sitting." b. "I will use a toilet elevator on the toilet seat." c. "I will have someone else put on my shoes and socks." d. "I can sleep in any position that is comfortable for me."

ANS: D The patient needs to sleep in a position that prevents excessive internal rotation or flexion of the hip. The other patient statements indicate that the patient has understood the teaching.

A patient with leukemia is considering whether to have hematopoietic stem cell transplantation (HSCT). The nurse will include which information in the patient's teaching plan? a. Transplant of the donated cells is painful because of the nerves in the tissue lining the bone. b. Donor bone marrow cells are transplanted through an incision into the sternum or hip bone. c. The transplant procedure takes place in a sterile operating room to minimize the risk for infection. d. Hospitalization will be required for several weeks after the stem cell transplant procedure is performed.

ANS: D The patient requires strict protective isolation to prevent infection for 2 to 4 weeks after HSCT while waiting for the transplanted marrow to start producing cells. The transplanted cells are infused through an IV line, so the transplant is not painful, nor is an operating room or incision required.

A patient receiving head and neck radiation for larynx cancer has ulcerations over the oral mucosa and tongue and thick, ropey saliva. Which instructions should the nurse give to this patient? a. Remove food debris from the teeth and oral mucosa with a stiff toothbrush. b. Use cotton-tipped applicators dipped in hydrogen peroxide to clean the teeth. c. Gargle and rinse the mouth several times a day with an antiseptic mouthwash. d. Rinse the mouth before and after each meal and at bedtime with a saline solution.

ANS: D The patient should rinse the mouth with a saline solution frequently. A soft toothbrush is used for oral care. Hydrogen peroxide may damage tissues. Antiseptic mouthwashes may irritate the oral mucosa and are not recommended.

8. A patient with recurring heartburn receives a new prescription for esomeprazole (Nexium). In teaching the patient about this medication, the nurse explains that this drug a. neutralizes stomach acid and provides relief of symptoms in a few minutes. b. reduces the reflux of gastric acid by increasing the rate of gastric emptying. c. coats and protects the lining of the stomach and esophagus from gastric acid. d. treats gastroesophageal reflux disease by decreasing stomach acid production.

ANS: D The proton pump inhibitors decrease the rate of gastric acid secretion. Promotility drugs such as metoclopramide (Reglan) increase the rate of gastric emptying. Cryoprotective medications such as sucralfate (Carafate) protect the stomach. Antacids neutralize stomach acid and work rapidly.

The nurse teaches a postmenopausal patient with stage III breast cancer about the expected outcomes of cancer treatment. Which patient statement indicates that the teaching has been effective? a. "After cancer has not recurred for 5 years, it is considered cured." b. "The cancer will be cured if the entire tumor is surgically removed." c. "Cancer is never considered cured, but the tumor can be controlled with surgery, chemotherapy, and radiation." d. "I will need to have follow-up examinations for many years after I have treatment before I can be considered cured."

ANS: D The risk of recurrence varies by the type of cancer. Some cancers are considered cured after a shorter time span or after surgery, but stage III breast cancer will require additional therapies and ongoing follow-up.

Which change in renal or urinary functioning as a result of the normal aging process increases the older client's risk for infection? A. Decreased glomerular filtration B. Decreased filtrate reabsorption C. Weakened sphincter muscles D. Urinary retention

ANS: D Incomplete bladder emptying for whatever reason increases the client's risk for urinary tract infections as a result of urine stasis providing an excellent culture medium that promotes the growth of microorganisms.

The client reports the regular use of all the following medications. Which one alerts the nurse to the possibility of renal impairment when used consistently? A. Antacids B. Penicillin C. Antihistamine nasal sprays D. Nonsteroidal anti-inflammatory drug

ANS: D NSAIDs inhibit prostaglandin production and decrease blood flow to the nephrons. They can cause an interstitial nephritis and renal impairment.

When taking the health history of an older adult, the nurse discovers that the patient has worked in the landscaping business for 40 years. The nurse will plan to teach the patient about how to self-assess for which clinical manifestations (select all that apply)? a. Vitiligo b. Alopecia c. Intertrigo d. Erythema e. Actinic keratosis

ANS: D, E A patient who has worked as a landscaper is at risk for skin lesions caused by sun exposure such as erythema and actinic keratosis. Vitiligo, alopecia, and intertrigo are not associated with excessive sun exposure

40. A patient with an acute attack of gout in the left great toe has a new prescription for probenecid (Benemid). Which information about the patient's home routine indicates a need for teaching regarding gout management? a. The patient takes one aspirin a day prophylactically to prevent angina. b. The patient sleeps about 8 to 10 hours every night. c. The patient generally drinks about 3 quarts of juice and water daily. d. The patient usually eats beef once or twice a week.

Answer: A Rationale: Aspirin interferes with the effectiveness of probenecid and should not be taken when the patient is taking probenecid. The patient's sleep pattern will not affect gout management. Drinking 3 quarts of water and eating beef only once or twice a week are appropriate for the patient with gout. Cognitive Level: Application Text Reference: p. 1716 Nursing Process: Assessment NCLEX: Physiological Integrity

3. The health care provider has prescribed naproxen (Naprosyn) twice daily for a patient with osteoarthritis (OA) of the hands. The patient tells the nurse after 3 weeks of use that the drug does not seem to be effective in controlling the pain. The nurse should teach the patient that a. another type of nonsteroidal antiinflammatory drug (NSAID) may be indicated because of variations in individual response to the drugs. b. it may take up to 4 to 6 weeks for NSAIDs to reach therapeutic levels in the blood. c. if NSAIDs are not effective in controlling symptoms, corticosteroids are the next drug of choice. d. adding a twice-daily aspirin to the naproxen may improve the effectiveness of the drug.

Answer: A Rationale: Individual responses to NSAIDs can vary, so the health care provider may prescribe a different NSAID. Full effectiveness of NSAIDs occurs in 2 to 3 weeks. Corticosteroids are usually reserved for use in RA. Patients are instructed to avoid aspirin when taking NSAIDs because of the increased risk for bleeding and gastrointestinal irritation. Cognitive Level: Application Text Reference: p. 1697 Nursing Process: Implementation NCLEX: Physiological Integrity

16. The biologic agent anakinra (Kineret) is prescribed for a patient who has moderately severe rheumatoid arthritis (RA). When teaching the patient about this drug, the nurse will include information about a. symptoms of gastrointestinal (GI) irritation or bleeding. b. self-administration of subcutaneous injections. c. taking the medication with at least 8 oz of fluid. d. avoiding concurrently taking aspirin or NSAIDs.

Answer: B Rationale: Anakinra is administered by subcutaneous injection. GI bleeding is not a side effect of this medication. Because the medication is injected, instructions to take it with 8 oz of fluid would not be appropriate. The patient is likely to be concurrently taking aspirin or NSAIDs and these should not be discontinued. Cognitive Level: Application Text Reference: pp. 1700, 1705-1706 Nursing Process: Implementation NCLEX: Physiological Integrity

14. When the nurse is reviewing laboratory data for a patient who is taking methotrexate (Rheumatrex), which information is most important to communicate to the health care provider? a. The platelet count is 130,000/μl. b. The white blood cell count (WBC) is 1500/μl. c. The blood glucose is 130 mg/dl. d. The potassium is 5.2 mEq/L.

Answer: B Rationale: Bone marrow suppression is a possible side effect of methotrexate, and the patient's low WBC count places the patient at high risk for infection. The other laboratory values are also abnormal but are not far from normal values and would not have any immediate serious consequences. Cognitive Level: Application Text Reference: p. 1699 Nursing Process: Evaluation NCLEX: Physiological Integrity

6. A patient with hip pain is diagnosed with osteoarthritis (OA). The nurse may need to teach the patient about the use of a. prednisone (Deltasone). b. capsaicin cream (Zostrix). c. sulfasalazine (Azulfidine). d. doxycycline (Vibramycin).

Answer: B Rationale: Capsaicin cream blocks the transmission of pain impulses and is helpful for some patients in treating OA. The other medications would be used for patients with RA. Cognitive Level: Application Text Reference: p. 1696 Nursing Process: Planning NCLEX: Physiological Integrity

11. When teaching range-of-motion exercises to a patient who is having an acute exacerbation of rheumatoid arthritis (RA) with joint pain and swelling in both hands, the nurse teaches the patient that a. affected joints should not be exercised when pain is present. b. cold applications before exercise will decrease joint pain. c. exercises should be performed passively by someone other than the patient. d. regular walking may substitute for range-of-motion (ROM) exercises on some days.

Answer: B Rationale: Cold application is helpful in reducing pain during periods of exacerbation of RA. Because the joint pain is chronic, patients are instructed to exercise even when joints are painful. ROM exercises are intended to strengthen joints as well as improve flexibility, so passive ROM alone is not sufficient. Recreational exercise is encouraged but is not a replacement for ROM exercises. Cognitive Level: Application Text Reference: pp. 1707, 1710 Nursing Process: Implementation NCLEX: Physiological Integrity

37. The health care provider plans to prescribe methotrexate (Rheumatrex) to a patient with newly diagnosed rheumatoid arthritis (RA). The patient tells the nurse, "That drug has too many side effects; I would rather wait until my joint problems are worse before beginning any drugs." The most appropriate response by the nurse is a. "You should tell the doctor how you feel so the two of you can make a decision together." b. "It is important to start methotrexate early in order to decrease the joint damage." c. "Methotrexate is not expensive and will be cheaper to take than other possible drugs." d. "Methotrexate is very effective and has no more side effects than the other available drugs."

Answer: B Rationale: Disease-modifying anti-rheumatic drugs (DMARDs) are prescribed early to prevent the joint degeneration that occurs as soon as the first year with RA. The other statements are accurate, but the most important point for the patient to understand is that it is important to start DMARDs as quickly as possible. Cognitive Level: Application Text Reference: pp. 1699, 1705 Nursing Process: Implementation NCLEX: Physiological Integrity

35. A patient with fibromyalgia syndrome (FMS) tells the nurse, "I don't know why the doctor has prescribed amitriptyline (Elavil) for me. I don't feel depressed, just tired and achy." The most appropriate response by the nurse is, "The Elavil a. is ordered to prevent depression from occurring." b. will improve the quality of your sleep at night." c. relaxes your muscles and helps prevent spasm." d. has antiinflammatory actions to reduce joint pain."

Answer: B Rationale: Elavil is ordered to improve sleep, to decrease stress and fatigue, and as an adjuvant medication for pain control. It would not be ordered to prevent depression, although it might be ordered to treat depression in a patient with FMS. Elavil is not a muscle relaxant or an antiinflammatory drug, although medications from these categories are used in treating FMS. Cognitive Level: Application Text Reference: p. 1728 Nursing Process: Implementation NCLEX: Physiological Integrity

30. A 19-year-old patient who is taking azathioprine (Imuran) for systemic lupus erythematosus has a check-up before leaving home for college. The health care provider writes all of these orders. Which one should the nurse question? a. Naproxen (Aleve) 200 mg BID b. Give measles-mumps-rubella (MMR) immunization c. Draw anti-DNA titer d. Famotidine (Pepcid) 20 mg daily

Answer: B Rationale: Live virus vaccines, such as rubella, are contraindicated in a patient taking immunosuppressive drugs. The other orders are appropriate for the patient. Cognitive Level: Application Text Reference: p. 1718 Nursing Process: Implementation NCLEX: Psychosocial Integrity

13. When teaching a patient who has rheumatoid arthritis (RA) about how to manage activities of daily living, the nurse instructs the patient to a. stand rather than sit when performing household chores. b. avoid activities that require continuous use of the same muscles. c. strengthen small hand muscles by wringing sponges or washcloths. d. protect the knee joints by sleeping with a small pillow under the knees.

Answer: B Rationale: Patients are advised to avoid repetitious movements. Sitting during household chores is recommended to decrease stress on joints. Wringing water out of sponges would increase the joint stress. Patients are encouraged to position joints in the extended position, and sleeping with a pillow behind the knees would decrease the ability of the knee to extend and also decrease knee ROM. Cognitive Level: Application Text Reference: p. 1710 Nursing Process: Implementation NCLEX: Health Promotion and Maintenance

28. A patient with polyarthralgia with joint swelling and pain is being evaluated for systemic lupus erythematosus (SLE). The nurse knows that the serum test result that is the most specific for SLE is the presence of a. rheumatoid factor. b. anti-Smith antibody (Anti-Sm). c. antinuclear antibody (ANA). d. lupus erythematosus (LE) cell prep.

Answer: B Rationale: The anti-Sm is antibody found almost exclusively in SLE. The other blood tests are also used in screening but are not as specific to SLE. Cognitive Level: Comprehension Text Reference: pp. 1718-1719 Nursing Process: Assessment NCLEX: Physiological Integrity

36. A patient who has had fatigue and muscle weakness for several years is diagnosed with chronic fatigue syndrome. The patient expresses anger at the health care professional for not offering relief of the symptoms and also anger at family members for saying "snap out of it and get busy." Based on the patient's statements, the nurse identifies a nursing diagnosis of a. activity intolerance related to fatigue. b. powerlessness related to lack of control over illness. c. altered family process related to illness of family member. d. situational low self-esteem related to inability to meet role expectation.

Answer: B Rationale: The patient's statements support the problem and etiology of powerlessness related to lack of control. The patient does not complain about activity intolerance. Although the patient may have risk for altered family process, but there are not enough data to support this diagnosis. The patient is not expressing low self-esteem or feelings of inadequacy regarding meeting role expectations. Cognitive Level: Application Text Reference: p. 1729 Nursing Process: Diagnosis NCLEX: Psychosocial Integrity

26. A patient with an acute exacerbation of systemic lupus erythematosus (SLE) is hospitalized with incapacitating fatigue, acute hand and wrist pain, and proteinuria. The health care provider prescribes prednisone (Deltasone) 40 mg twice daily. Which nursing action should be included in the plan of care? a. Institute seizure precautions. b. Reorient to time and place PRN. c. Monitor intake and output. d. Place on cardiac monitor.

Answer: C Rationale: Lupus nephritis is a common complication of SLE, and when the patient is taking corticosteroids, it is especially important to monitor renal function. There is no indication that the patient is experiencing any nervous system or cardiac problems with the SLE. Cognitive Level: Application Text Reference: pp. 1718, 1720 Nursing Process: Planning NCLEX: Physiological Integrity

18. In teaching a patient with ankylosing spondylitis (AS) about the management of the condition, the nurse instructs the patient to a. sleep on the side with hips flexed. b. take slow, long walks as a form of exercise. c. perform daily deep-breathing exercises. d. take frequent naps during the day.

Answer: C Rationale: Deep-breathing exercises are used to decrease the risk for pulmonary complications that may occur with the reduced chest expansion that can occur with ankylosing spondylitis (AS). Patients should sleep on the back and avoid flexed positions. Prolonged standing and walking should be avoided. There is no need for frequent naps. Cognitive Level: Comprehension Text Reference: p. 1712 Nursing Process: Implementation NCLEX: Physiological Integrity

38. A patient with an exacerbation of rheumatoid arthritis (RA) is taking prednisone (Deltasone) 40 mg daily. Which of these assessment data obtained by the nurse indicate that the patient is experiencing a side effect of the medication? a. The patient has experienced a recent 5-pound weight loss. b. The patient's erythrocyte sedimentation rate (ESR) has increased. c. The patient's blood glucose is 166 mg/dl. d. The patient has no improvement in symptoms.

Answer: C Rationale: Hyperglycemia is a side effect of prednisone. Corticosteroids increase appetite and lead to weight gain. An elevated ESR and no improvement in symptoms would indicate that the prednisone was not effective but would not be side effects of the medication. Cognitive Level: Application Text Reference: p. 1699 Nursing Process: Evaluation NCLEX: Physiological Integrity

8. The health care provider prescribes methotrexate (Rheumatrex) for a 28-year-old woman with stage II moderate rheumatoid arthritis (RA). When obtaining a health history from the patient, the most important information for the nurse to communicate to the health care provider is that the patient has a. a history of infectious mononucleosis as a teenager. b. a family history of age-related macular degeneration of the retina. c. been trying to have a baby before her disease becomes more severe. d. been using large doses of vitamins and health foods to treat the RA.

Answer: C Rationale: Methotrexate is teratogenic, and the patient should be taking contraceptives during methotrexate therapy. The other information will not impact the choice of methotrexate as therapy. Cognitive Level: Application Text Reference: p. 1699 Nursing Process: Assessment NCLEX: Physiological Integrity

20. A 22-year-old patient hospitalized with severe pain in the knees and a fever and shaking chills is suspected of having septic arthritis. Information obtained during the nursing history that indicates a risk factor for septic arthritis is that the patient a. has a parent who has reactive arthritis. b. recently returned from a trip to South America. c. is sexually active and has multiple partners. d. had several sports-related knee injuries as a teenager.

Answer: C Rationale: Neisseria gonorrhoeae is the most common cause for septic arthritis in sexually active young adults. The other information does not point to any risk for septic arthritis. Cognitive Level: Application Text Reference: p. 1713 Nursing Process: Assessment NCLEX: Physiological Integrity

2. When screening patients at a community center, the nurse will plan to teach ways to reduce risk factors for osteoarthritis to a a. 24-year-old man who participates in a summer softball team. b. 36-year-old woman who is newly diagnosed with diabetes mellitus. c. 49-year-old woman who works on an automotive assembly line. d. 56-year-old man who is a member of a construction crew.

Answer: C Rationale: OA is more likely to occur in women as a result of estrogen reduction at menopause and in individuals whose work involves repetitive movements and lifting. Moderate exercise, such as softball, reduces risk for OA. Diabetes is not a risk factor for OA. Working on a construction crew would involve nonrepetitive work and thus would not be as risky. Cognitive Level: Application Text Reference: p. 1694 Nursing Process: Planning NCLEX: Physiological Integrity

4. When teaching a patient with osteoarthritis (OA) of the left hip and lower lumbar vertebrae about management of the condition, the nurse determines that additional instruction is needed when the patient says, a. "I can use a cane if I find it helpful in relieving the pressure on my back and hip." b. "A warm shower in the morning will help relieve the stiffness I have when I get up." c. "I should try to stay active throughout the day to keep my joints from becoming stiff." d. "I should take no more than 1 g of acetaminophen four times a day to control the pain."

Answer: C Rationale: Protection and avoidance of joint stressors are recommended for patients with OA, so this patient should alternate periods of rest with necessary activity. The other patient statements indicate that teaching has been effective. Cognitive Level: Application Text Reference: pp. 1696, 1701 Nursing Process: Evaluation NCLEX: Health Promotion and Maintenance

25. A 26-year-old woman has been diagnosed with early systemic lupus erythematosus (SLE) involving her joints. In teaching the patient about the disease, the nurse includes the information that SLE is a(n) a. hereditary disorder of women but usually does not show clinical symptoms unless a woman becomes pregnant. b. autoimmune disease of women in which antibodies are formed that destroy all nucleated cells in the body. c. disorder of immune function, but it is extremely variable in its course, and there is no way to predict its progression. d. disease that causes production of antibodies that bind with cellular estrogen receptors, causing an inflammatory response.

Answer: C Rationale: SLE has an unpredictable course, even with appropriate treatment. Women are more at risk for SLE, but it is not confined exclusively to women. Clinical symptoms may worsen during pregnancy but are not confined to pregnancy or the perinatal period. All nucleated cells are not destroyed by the antinuclear antibodies. The inflammation in SLE is not caused by antibody binding to cellular estrogen receptors. Cognitive Level: Comprehension Text Reference: pp. 1717, 1719 Nursing Process: Implementation NCLEX: Physiological Integrity

9. A patient with an acute exacerbation of rheumatoid arthritis (RA) has localized pain and inflammation of the fingers, wrists, and feet with swelling, redness, and limited movement of the joints. When developing the plan of care, the nurse recognizes that the most appropriate patient outcome at this time is to a. maintain a positive self-image. b. perform activities of daily living independently. c. achieve satisfactory control of pain. d. make a successful adjustment to disease progression.

Answer: C Rationale: The focus during an acute exacerbation of RA is to manage pain effectively. The other outcomes are appropriate long-term outcomes. Cognitive Level: Application Text Reference: p. 1707 Nursing Process: Planning NCLEX: Health Promotion and Maintenance

27. A patient with systemic lupus erythematosus (SLE) who has a facial rash and alopecia tells the nurse, "I hate the way I look! I never go anyplace except here to the health clinic." An appropriate nursing diagnosis for the patient is a. activity intolerance related to fatigue and inactivity. b. impaired skin integrity related to itching and skin sloughing. c. social isolation related to embarrassment about the effects of SLE. d. impaired social interaction related to lack of social skills.

Answer: C Rationale: The patient's statement about not going anyplace because of hating the way he or she looks supports the diagnosis of social isolation because of embarrassment about the effects of the SLE. Activity intolerance is a possible problem for patients with SLE, but the information about this patient does not support this as a diagnosis. The rash with SLE is nonpruritic. There is no evidence of lack of social skills for this patient. Cognitive Level: Application Text Reference: p. 1722 Nursing Process: Diagnosis NCLEX: Psychosocial Integrity

7. A 71-year-old obese patient has bilateral osteoarthritis (OA) of the hips. The nurse teaches the patient that the most beneficial measure to protect the joints is to a. use a wheelchair to avoid walking as much as possible. b. sit in chairs that do not cause the hips to be lower than the knees. c. use a walker for ambulation to relieve the pressure on the hips. d. eat according to a weight-reduction diet to obtain a healthy body weight.

Answer: D Rationale: Because the patient's major risk factor is obesity, the nurse should teach the patient that weight loss is the best way to reduce stress on the hips. Avoiding activity by sitting in a wheelchair would likely increase the patient's weight; moderate activity is recommended for patients with OA. Sitting with the hips higher than the knees and using a walker would be recommended but are not as helpful as weight loss for this obese patient. Cognitive Level: Application Text Reference: p. 1701 Nursing Process: Implementation NCLEX: Physiological Integrity

12. Prednisone (Deltasone) is prescribed for a patient with an acute exacerbation of rheumatoid arthritis. When the patient has a follow-up visit 1 month later, the nurse recognizes that the patient's response to the treatment may be best evaluated by a. blood glucose testing. b. liver function tests. c. serum electrolyte levels. d. C-reactive protein level.

Answer: D Rationale: C-reactive protein is a marker for inflammation, and a decrease would indicate that the corticosteroid therapy was effective. Blood glucose and serum electrolyte levels will also be monitored to check for side effects of prednisone. Liver function is not routinely monitored for patients receiving steroids. Cognitive Level: Application Text Reference: pp. 1698-1699 Nursing Process: Evaluation NCLEX: Physiological Integrity

5. A 58-year-old patient has been diagnosed with osteoarthritis (OA) of the hands and feet. The patient tells the nurse, "I am afraid that I will be hopelessly crippled in just a few years!" The best response by the nurse is that a. progression of OA can be prevented with a regimen of exercise, diet, and drugs. b. OA is an inflammatory process with periods of exacerbation and remission. c. joint degeneration with pain and deformity occurs with OA by age 60 to 70. d. OA is common with aging, but usually it is localized and does not cause deformity.

Answer: D Rationale: OA is localized to joints that have been injured or have high use. Although exercise, diet, and drugs can help to decrease symptoms and slow disease progression, they will not prevent progression of the disease. OA is progressive and does not have exacerbations and remissions, which are typical of RA. Joint degeneration and pain occur later in OA; joint deformity is not a common symptom. Cognitive Level: Application Text Reference: pp. 1701, 1704 Nursing Process: Implementation NCLEX: Physiological Integrity

31. A patient has systemic sclerosis manifested by the CREST syndrome. During assessment of the patient, the nurse would expect to find a. bony ankylosis of the small joints in the feet. b. a recent history of significant weight gain. c. burning, itching, and photosensitivity of the eyes. d. a history of numbness and tingling in the fingers.

Answer: D Rationale: Raynaud's phenomenon is one aspect of the CREST syndrome. Bony ankylosis is not a symptom of systemic sclerosis, which does not affect bone. Weight loss occurs with CREST syndrome as a result of esophageal scarring. Burning, itching, and photosensitivity of the eyes are not associated with systemic sclerosis. Cognitive Level: Application Text Reference: p. 1723 Nursing Process: Assessment NCLEX: Physiological Integrity

17. A 35-year-old patient with three school-age children who has recently been diagnosed with rheumatoid arthritis (RA) tells the nurse that the inability to be involved in many family activities is causing stress at home. Which response by the nurse is most appropriate?" a. "Your family may need some help to understand the impact of your rheumatoid arthritis." b. "You may need to see a family therapist for some help." c. "Perhaps it would be helpful for you and your family to get involved in a support group." d. "Tell me more about the situations that are causing stress."

Answer: D Rationale: The initial action by the nurse should be further assessment. The other three responses might be appropriate based on the information the nurse obtains with further assessment. Cognitive Level: Application Text Reference: p. 1711 Nursing Process: Implementation NCLEX: Health Promotion and Maintenance

10. A home health patient with rheumatoid arthritis (RA) complains to the nurse about having chronically dry eyes and a dry mouth. Which action by the nurse is most appropriate? a. Have the patient withhold the daily methotrexate (Rheumatrex) until talking with the health care provider. b. Reassure the patient that dry eyes and mouth are very common with RA. c. Teach the patient to use an antiseptic mouth wash tid. d. Suggest that the patient start using over-the-counter (OTC) artificial tears.

Answer: D Rationale: The patient's dry eyes and oral mucous membranes are consistent with Sjögren's syndrome, a common extraarticular manifestation of RA. Symptomatic therapy such as OTC eye drops is recommended. Dry eyes and mouth are not side effects of methotrexate. Although dry eyes and mouth are common in RA, it is more helpful to offer a suggestion to relieve these symptoms than to offer reassurance. Rinsing the mouth to decrease oral dryness is appropriate, but the frequent use of antiseptic mouthwashes is not appropriate unless the patient has oral symptoms that require this. Cognitive Level: Application Text Reference: p. 1726 Nursing Process: Implementation NCLEX: Physiological Integrity

The nurse is planning care for a 68-year-old patient with an abdominal mass and suspected bowel obstruction. Which factor in the patient's history increases the patient's risk for colorectal cancer? A. Osteoarthritis B. History of colorectal polyps C. History of lactose intolerance D. Use of herbs as dietary supplements

B A history of colorectal polyps places this patient at risk for colorectal cancer. This tissue can degenerate over time and become malignant. Osteoarthritis, lactose intolerance, and the use of herbs do not pose additional risk to the patient.

What information would have the highest priority to be included in preoperative teaching for a 68-year-old patient scheduled for a colectomy? A. How to care for the wound B. How to deep breathe and cough C. The location and care of drains after surgery D. Which medications will be used during surgery

B Because anesthesia, an abdominal incision, and pain can impair the patient's respiratory status in the postoperative period, it is of high priority to teach the patient to cough and deep breathe. Otherwise, the patient could develop atelectasis and pneumonia, which would delay early recovery from surgery and hospital discharge. Care for the wound and location and care of the drains will be briefly discussed preoperatively, but done again with higher priority after surgery. Knowing which drugs will be used during surgery may not be meaningful to the patient and should be reviewed with the patient by the anesthesiologist.

21. When designing a program to decrease the incidence of HIV infection in the community, the nurse will prioritize teaching about a. methods to prevent perinatal HIV transmission. b. how to prevent transmission between sexual partners. c. ways to sterilize needles used by injectable drug users. d. means to prevent transmission through blood transfusions.

B. how to prevent transmission between sexual partners Sexual transmission is the most common way that HIV is transmitted. The nurse should also provide education about perinatal transmission, needle sterilization, and blood transfusion, but the rate of HIV infection associated with these situations is lower.

15. A diabetic patient is started on intensive insulin therapy. The nurse will plan to teach the patient about mealtime coverage using _____ insulin. a. NPH b. lispro c. detemir d. glargine

B. lispro Rationale: Rapid or short acting insulin is used for mealtime coverage for patients receiving intensive insulin therapy. NPH, glargine, or detemir will be used as the basal insulin.

A patient with renal calculi is hospitalized with gross hematuria and severe colicky left flank pain. Which nursing action will be of highest priority at this time? a. Encourage oral fluid intake. b. Administer prescribed analgesics. c. Monitor temperature every 4 hours. d. Give antiemetics as needed for nausea.

B. Administer prescribed analgesics Although all of the nursing actions may be used for patients with renal lithiasis, the patient's presentation indicates that management of pain is the highest priority action. If the patient has urinary obstruction, increasing oral fluids may increase the symptoms. There is no evidence of infection or nausea.

The 24-year-old male patient who was successfully treated for Paget's disease has come to the clinic with a gradual onset of pain and swelling around the left knee. The patient is diagnosed with osteosarcoma without metastasis. The patient wants to know why he will be given chemotherapy before the surgery. What is the best rationale the nurse should tell the patient? A. The chemotherapy is being used to save your left leg. B. Chemotherapy is being used to decrease the tumor size. C. The chemotherapy will increase your 5-year survival rate. D. Chemotherapy will help decrease the pain before and after surgery.

B. Chemotherapy is being used to decrease the tumor size. Preoperative chemotherapy is used to decrease tumor size before surgery. The chemotherapy will not save his leg if the lesion is too big or there is neurovascular or muscle involvement. Adjunct chemotherapy after amputation or limb salvage has increased 5-year survival rate in people without metastasis. Chemotherapy is not used to decrease pain before or after surgery.

The patient is brought to the ED following a car accident and is wearing medical identification that says she has Addison's disease. What should the nurse expect to be included in the collaborative care of this patient? A. Low sodium diet B. Increased glucocorticoid replacement C. Suppression of pituitary ACTH synthesis D. Elimination of mineralocorticoid replacement

B. Increased glucocorticoid replacement The patient with Addison's disease needs lifelong glucocorticoid and mineralocorticoid replacement and has an increased need with illness, injury, or stress, as this patient is experiencing. The patient with Addison's may also need a high sodium diet. Suppression of pituitary ACTH synthesis is done for Cushing syndrome. Elimination of mineralocorticoid replacement cannot be done for Addison's disease.

A patient returns to the clinic with recurrent dysuria after being treated with trimethoprim and sulfamethoxazole (Bactrim) for 3 days. Which action will the nurse plan to take? a. Remind the patient about the need to drink 1000 mL of fluids daily. b. Obtain a midstream urine specimen for culture and sensitivity testing. c. Teach the patient to take the prescribed Bactrim for at least 3 more days. d. Suggest that the patient use acetaminophen (Tylenol) to treat the symptoms.

B. Obtain a midstream urine specimen for culture and sensitivity testing Since uncomplicated urinary tract infections (UTIs) are usually successfully treated with 3 days of antibiotic therapy, this patient will need a urine culture and sensitivity to determine appropriate antibiotic therapy. Tylenol would not be as effective as other over-the-counter (OTC) medications such as phenazopyridine (Pyridium) in treating dysuria. The fluid intake should be increased to at least 1800 mL/day. Since the UTI has persisted after treatment with Bactrim, the patient is likely to need a different antibiotic. DIF: Cognitive Level: Application REF: 1123-1125

16. The nurse is preparing to give the following medications to an HIV-positive patient who is hospitalized with Pneumocystis jiroveci pneumonia (PCP). Which is most important to administer at the right time? a. Nystatin (Mycostatin) tablet for vaginal candidiasis b. Oral saquinavir (Inverase) to suppress HIV infection c. Aerosolized pentamidine (NebuPent) for PCP infection d. Oral acyclovir (Zovirax) to treat systemic herpes simplex

B. Oral saquinavir (Inverase) to suppress HIV infection It is important that antiretrovirals be taken at the prescribed time every day to avoid developing drug-resistant HIV. The other medications should also be given as close as possible to the correct time, but they are not as essential to receive at the same time every day.

20. Which action can the nurse delegate to nursing assistive personnel (NAP) who help with the care of a patient admitted with tuberculosis and placed on airborne precautions? a. Teach the patient about how to use tissues to dispose of respiratory secretions. b. Stock the patient's room with all the necessary personal protective equipment. c. Interview the patient to obtain the names of family members and close contacts. d. Tell the patient's family members the reason for the use of airborne precautions.

B. Stock the patient's room with all the necessary personal protective equipment Since all health care workers will be educated about the various types of infection precautions used in the hospital, the NAP can safely stock the room with personal protective equipment. Obtaining contact information and patient education are higher-level skills that require RN education and scope of practice.

The nurse receives a phone call from a 36-year-old woman taking cyclophosphamide (Cytoxan) for treatment of non-Hodgkin's lymphoma. The patient tells the nurse that she has muscle cramps and weakness and very little urine output. Which response by the nurse is best? A. "Start taking supplemental potassium, calcium, and magnesium." B. "Stop taking the medication now and call your health care provider." C. "These symptoms will decrease with continued use of the medication." D. "Increase fluids to 3000 mL per 24 hours to improve your urine output."

B. Stop taking the medication now and call your health care provider." Cyclophosphamide may cause syndrome of inappropriate antidiuretic hormone (SIADH). Medications that stimulate the release of ADH should be avoided or discontinued. Treatment may include restriction of fluids to 800 to 1000 mL per day. If a loop diuretic such as furosemide (Lasix) is used to promote diuresis, supplements of potassium, calcium, and magnesium may be needed.

What can the nurse do to facilitate cancer prevention for the patient in the promotion stage of cancer development? A. Teach the patient to exercise daily. B. Teach the patient promoting factors to avoid. C. Tell the patient to have the cancer surgically removed now. D. Teach the patient which vitamins will improve the immune system.

B. Teach the patient promoting factors to avoid. The promotion stage of cancer is characterized by the reversible proliferation of the altered cells. Changing the lifestyle to avoid promoting factors (dietary fat, obesity, cigarette smoking, and alcohol consumption) can reduce the chance of cancer development. Daily exercise and vitamins alone will not prevent cancer. Surgery at this stage may not be possible without a critical mass of cells, and this advice would not be the nurse's role.

The nurse is providing discharge instructions to a patient with diabetes insipidus. Which instructions regarding desmopressin acetate (DDAVP) would be most appropriate? A. The patient can expect to experience weight loss resulting from increased diuresis. B. The patient should alternate nostrils during administration to prevent nasal irritation. C. The patient should monitor for symptoms of hypernatremia as a side effect of this drug. D. The patient should report any decrease in urinary elimination to the health care provider.

B. The patient should alternate nostrils during administration to prevent nasal irritation. DDAVP is used to treat diabetes insipidus by replacing the antidiuretic hormone that the patient is lacking. Inhaled DDAVP can cause nasal irritation, headache, nausea, and other signs of hyponatremia. Diuresis will be decreased and is expected, and hypernatremia should not occur.

The patient is receiving an IV vesicant chemotherapy drug. The nurse notices swelling and redness at the site. What should the nurse do first? A. Ask the patient if the site hurts. B. Turn off the chemotherapy infusion. C. Call the ordering health care provider. D. Administer sterile saline to the reddened area.

B. Turn off the chemotherapy infusion. Because extravasation of vesicants may cause severe local tissue breakdown and necrosis, with any sign of extravasation the infusion should first be stopped. Then the protocol for the drug-specific extravasation procedures should be followed to minimize further tissue damage. The site of extravasation usually hurts, but it may not. It is more important to stop the infusion immediately. The health care provider may be notified by another nurse while the patient's nurse starts the drug-specific extravasation procedures, which may or may not include sterile saline.

A patient with nephrotic syndrome develops flank pain. The nurse will anticipate teaching the patient about treatment with a. antibiotics. b. anticoagulants. c. corticosteroids. d. antihypertensives.

B. anticoagulants Flank pain in a patient with nephrosis suggests a renal vein thrombosis, and anticoagulation is needed. Antibiotics are used to treat a patient with flank pain caused by pyelonephritis. Antihypertensives are used if the patient has high blood pressure. Corticosteroids may be used to treat nephrotic syndrome but will not resolve a thrombosis.

Following an open loop resection and fulguration of the bladder, a patient is unable to void. Which nursing action should be implemented first? a. Insert a straight catheter and drain the bladder. b. Assist the patient to take a 15-minute sitz bath. c. Encourage the patient to drink several glasses of water. d. Teach the patient how to do isometric perineal exercises.

B. assist the patient to take a 15-minute sitz bath. Sitz baths will relax the perineal muscles and promote voiding. Although the patient should be encouraged to drink fluids and Kegel exercises are helpful in the prevention of incontinence, these activities would not be helpful for a patient experiencing retention. Catheter insertion increases the risk for urinary tract infection (UTI) and should be avoided when possible

After obtaining the health history for a 25-year-old who smokes two packs of cigarettes daily, the nurse will plan to do teaching about the increased risk for a. kidney stones. b. bladder cancer. c. bladder infection. d. interstitial cystitis.

B. bladder cancer Cigarette smoking is a risk factor for bladder cancer. The patient's risk for developing interstitial cystitis, urinary tract infection (UTI), or kidney stones will not be reduced by quitting smoking.

A patient who has bladder cancer had a cystectomy with creation of an Indiana pouch. Which topic will be included in patient teaching? a. Application of ostomy appliances b. Catheterization technique and schedule c. Analgesic use before emptying the pouch d. Use of barrier products for skin protection

B. catheterization technique and schedule The Indiana pouch enables the patient to self-catheterize every 4 to 6 hours. There is no need for an ostomy device or barrier products. Catheterization of the pouch is not painful.

Which nursing action should the nurse who is caring for a patient who has had an ileal conduit for several years delegate to nursing assistive personnel (NAP)? a. Assess for symptoms of urinary tract infection (UTI). b. Change the ostomy appliance. c. Choose the appropriate ostomy bag. d. Monitor the appearance of the stoma.

B. change the ostomy appliance Changing the ostomy appliance for a stable patient could be done by NAP. Assessments of the site, choosing the appropriate ostomy bag, and assessing for (UTI) symptoms require more education and scope of practice and should be done by the RN.

Two days after surgery for an ileal conduit, the patient will not look at the stoma or participate in care. The patient insists that no one but the ostomy nurse specialist care for the stoma. The nurse identifies a nursing diagnosis of a. anxiety related to effects of procedure on lifestyle. b. disturbed body image related to change in body function. c. readiness for enhanced coping related to need for information. d. self-care deficit, toileting, related to denial of altered body function.

B. disturbed body image related to change in body function The patient's unwillingness to look at the stoma or participate in care indicates that disturbed body image is the best diagnosis. No data suggest that the impact on lifestyle is a concern for the patient. The patient does not appear to be ready for enhanced coping. The patient's insistence that only the ostomy nurse care for the stoma indicates that denial is not present.

10. A 1200-calorie diet and exercise are prescribed for a patient with newly diagnosed type 2 diabetes. The patient tells the nurse, "I hate to exercise! Can't I just follow the diet to keep my glucose under control?" The nurse teaches the patient that the major purpose of exercise for diabetics is to a. increase energy and sense of well-being, which will help with body image. b. facilitate weight loss, which will decrease peripheral insulin resistance. c. improve cardiovascular endurance, which is important for diabetics. d. set a successful pattern, which will help in making other needed changes.

B. facilitate weight loss, which will decrease peripheral insulin resistance Rationale: Exercise is essential to decrease insulin resistance and improve blood glucose control. Increased energy, improved cardiovascular endurance, and setting a pattern of success are secondary benefits of exercise, but they are not the major reason.

30. A patient with type 2 diabetes has sensory neuropathy of the feet and legs and peripheral vascular disease evidenced by decreased peripheral pulses and dependent rubor. The nurse teaches the patient that a. the feet should be soaked in warm water on a daily basis. b. flat-soled leather shoes are the best choice to protect the feet from injury. c. heating pads should always be set at a very low temperature. d. over-the-counter (OTC) callus remover may be used to remove callus and prevent pressure.

B. flat-soled leather shoes are the best choice to protect the feet from injury Rationale: The patient is taught to avoid high heels and that leather shoes are preferred. The feet should be washed, but not soaked, in warm water daily. Heating pad use should be avoided. Commercial callus and corn removers should be avoided; the patient should see a specialist to treat these problems.

25. A patient with type 1 diabetes who uses glargine (Lantus) and lispro (Humalog) insulin develops a sore throat, cough, and fever. When the patient calls the clinic to report the symptoms and a blood glucose level of 210 mg/dl, the nurse advises the patient to a. use only the lispro insulin until the symptoms of infection are resolved. b. monitor blood glucose every 4 hours and notify the clinic if it continues to rise. c. decrease intake of carbohydrates until glycosylated hemoglobin is less than 7%. d. limit intake to non-calorie-containing liquids until the glucose is within the usual range.

B. monitor blood glucose every 4 hours and notify the clinic if it continues to rise. Rationale: Infection and other stressors increase blood glucose levels and the patient will need to test blood glucose frequently, treat elevations appropriately with insulin, and call the health care provider if glucose levels continue to be elevated. Discontinuing the glargine will contribute to hyperglycemia and may lead to DKA. Decreasing carbohydrate or caloric intake is not appropriate as the patient will need more calories when ill. Glycosylated hemoglobins are not used to test for short-term alterations in blood glucose.

12. A patient with type 1 diabetes has an unusually high morning glucose measurement, and the health care provider wants the patient evaluated for possible Somogyi effect. The nurse will plan to a. administer an increased dose of NPH insulin in the evening. b. obtain the patient's blood glucose at 3:00 in the morning. c. withhold the nighttime snack and check the glucose at 6:00 AM. d. check the patient for symptoms of hypoglycemia at 2:00 to 4:00 AM.

B. obtain the patient's blood glucose at 3:00 in the morning Rationale: In the Somogyi effect, the patient's blood glucose drops in the early morning hours (in response to excess insulin administration), which causes the release of hormones that result in a rebound hyperglycemia. It is important to check the blood glucose in the early morning hours to detect the initial hypoglycemia. An increased evening NPH dose or holding the nighttime snack will further increase the risk for early morning hypoglycemia. Information about symptoms of hypoglycemia will not be as accurate as checking the patient's blood glucose in determining whether the patient has the Somogyi effect.

A patient who has had a transurethral resection with fulguration for bladder cancer 3 days previously calls the nurse at the urology clinic. Which information given by the patient is most important to report to the health care provider? a. The patient is using opioids for pain. b. The patient has noticed clots in the urine. c. The patient is very anxious about the cancer. d. The patient is voiding every 4 hours at night.

B. the patient has noticed clots in the urine Clots in the urine are not expected and require further follow-up. Voiding every 4 hours, use of opioids for pain, and anxiety are typical after this procedure.

3. During a diabetes screening program, a patient tells the nurse, "My mother died of complications of type 2 diabetes. Can I inherit diabetes?" The nurse explains that a. as long as the patient maintains normal weight and exercises, type 2 diabetes can be prevented. b. the patient is at a higher than normal risk for type 2 diabetes and should have periodic blood glucose level testing. c. there is a greater risk for children developing type 2 diabetes when the father has type 2 diabetes. d. although there is a tendency for children of people with type 2 diabetes to develop diabetes, the risk is higher for those with type 1 diabetes.

B. the patient is at higher than normal risk for type 2 diabetes and should have periodic blood glucose level testing Rationale: Offspring of people with type 2 diabetes are at higher risk for developing type 2 diabetes. The risk can be decreased, but not prevented, by maintenance of normal weight and exercising. The risk for children of a person with type 1 diabetes to develop diabetes is higher when it is the father who has the disease. Offspring of people with type 2 diabetes are more likely to develop diabetes than offspring of those with type 1 diabetes.

33. A patient with type 2 diabetes is scheduled for an outpatient coronary arteriogram. Which information obtained by the nurse when admitting the patient indicates a need for a change in the patient's regimen? a. The patient's most recent hemoglobin A1C was 6%. b. The patient takes metformin (Glucophage) every morning. c. The patient uses captopril (Capoten) for hypertension. d. The patient's admission blood glucose is 128 mg/dl.

B. the patient takes metformin (Glucophage) every morning Rationale: To avoid lactic acidosis, metformin should not be used for 48 hours after IV contrast media are administered. The other patient data indicate that the patient is managing the diabetes appropriately.

1. A patient with newly diagnosed type 2 diabetes mellitus asks the nurse what "type 2" means in relation to diabetes. The nurse explains to the patient that type 2 diabetes differs from type 1 diabetes primarily in that with type 2 diabetes a. the patient is totally dependent on an outside source of insulin. b. there is decreased insulin secretion and cellular resistance to insulin that is produced. c. the immune system destroys the pancreatic insulin-producing cells. d. the insulin precursor that is secreted by the pancreas is not activated by the liver.

B. there is a decreased insulin secretion and cellular resistance to insulin that is produced. Rationale: In type 2 diabetes, the pancreas produces insulin, but the insulin is insufficient for the body's needs or the cells do not respond to the insulin appropriately.

Following bowel resection, a patient has a nasogastric (NG) tube to suction, but complains of nausea and abdominal distention. The nurse irrigates the tube as necessary as ordered, but the irrigating fluid does not return. What should be the priority action by the nurse? A. Notify the physician. B. Auscultate for bowel sounds. C. Reposition the tube and check for placement. D. Remove the tube and replace it with a new one.

C The tube may be resting against the stomach wall. The first action by the nurse (since this is intestinal surgery and not gastric surgery) is to reposition the tube and check it again for placement. The physician does not need to be notified unless the tube function cannot be restored by the nurse. The patient does not have bowel sounds, which is why the NG tube is in place. The NG tube would not be removed and replaced unless it was no longer in the stomach or the obstruction of the tube could not be relieved.

A diabetic patient has a serum glucose level of 824 mg/dL and is unresponsive. After assessing the patient, the nurse suspects diabetic ketoacidosis rather than hyperosmolar hyperglycemic syndrome based on the finding of a) polyuria b) severe dehydration c) rapid, deep respirations d) decreased serum potassium

C. rapid, deep respirations (Kussmaul's Respirations)

The nurse formulates a nursing diagnosis of impaired physical mobility related to decreased muscle strength for a 78-year-old patient following left total knee replacement. What would be an appropriate nursing intervention for this patient? A Promote vitamin C and calcium intake in the diet. B Provide passive range of motion to all of the joints q4hr. C Encourage isometric quadriceps-setting exercises at least qid. D Keep the left leg in extension and abduction to prevent contractures.

C Encourage isometric quadriceps-setting exercises at least qid. Emphasis is placed on postoperative exercise of the affected leg, with isometric quadriceps setting beginning on the first day after surgery along with a continuous passive motion (CPM) machine. Vitamin C and calcium do not improve muscle strength, but they will facilitate healing. The patient should be able to do active range of motion to all joints. Keeping the leg in one position (extension and abduction) potentially will result in contractures.

The nurse is caring for a patient with osteoarthritis who is about to undergo total left knee arthroplasty. The nurse assesses the patient carefully to be sure that there is no evidence of what in the preoperative period? A Pain B Left knee stiffness C Left knee infection D Left knee instability

C Left knee infection It is critical that the patient be free of infection before a total knee arthroplasty. An infection in the joint could lead to even greater pain and joint instability, requiring extensive surgery. For this reason, the nurse monitors the patient for signs of infection, such as redness, swelling, fever, and elevated white blood cell count. Pain, knee stiffness, or instability may be present with osteoarthritis.

Which clinical manifestations of inflammatory bowel disease are common to both patients with ulcerative colitis (UC) and Crohn's disease (select all that apply)? A. Restricted to rectum B. Strictures are common. C. Bloody, diarrhea stools D. Cramping abdominal pain E. Lesions penetrate intestine.

C, D Clinical manifestations of UC and Crohn's disease include bloody diarrhea, cramping abdominal pain, and nutritional disorders. Intestinal lesions associated with UC are usually restricted to the rectum before moving into the colon. Lesions that penetrate the intestine or cause strictures are characteristic of Crohn's disease

The nurse determines that dietary teaching for a 75-year-old patient with osteoporosis has been successful when the patient selects which highest-calcium meal? A. Chicken stir-fry with 1 cup each onions and green peas, and 1 cup of steamed rice B. Ham and Swiss cheese sandwich on whole wheat bread, steamed broccoli, and an apple C. A sardine (3 oz) sandwich on whole wheat bread, 1 cup of fruit yogurt, and 1 cup of skim milk D. A two-egg omelet with 2 oz of American cheese, one slice of whole wheat toast, and a half grapefruit

C. A sardine (3 oz) sandwich on whole wheat bread, 1 cup of fruit yogurt, and 1 cup of skim milk The highest calcium content is present in the lunch containing milk and milk products (yogurt) and small fish with bones (sardines). Chicken, onions, green peas, rice, ham, whole wheat bread, broccoli, apple, eggs, and grapefruit each have less than 75 mg of calcium per 100 g of food. Swiss cheese and American cheese have more calcium, but not as much as the sardines, yogurt, and milk.

A patient is admitted to the hospital with new onset nephrotic syndrome. Which assessment data will the nurse expect to find related to this illness? a. Poor skin turgor b. High urine ketones c. Recent weight gain d. Low blood pressure

C. recent weight gain The patient with a nephrotic syndrome will have weight gain associated with edema. Hypertension is a clinical manifestation of nephrotic syndrome. Skin turgor is normal because of the edema. Urine protein is high.

19. Ten years after seroconversion, an HIV-infected patient has a CD4+ cell count of 800/µl and an undetectable viral load. What is the priority nursing intervention at this time? a. Monitor for symptoms of AIDS. b. Teach about the effects of antiretroviral agents. c. Encourage adequate nutrition, exercise, and sleep. d. Discuss likelihood of increased opportunistic infections.

C. Encourage adequate nutrition, exercise, and sleep The CD4+ level for this patient is in the normal range, indicating that the patient is the early chronic stage of infection, when the body is able to produce enough CD4+ cells to maintain a normal CD4+ count. AIDS and increased incidence of opportunistic infections typically develop when the CD4+ count is much lower than normal. Although initiation of ART is highly individual, it would not be likely that a patient with a normal CD4+ level would receive ART.

Which assessment finding for a patient who has had a cystectomy with an ileal conduit the previous day is most important for the nurse to communicate to the physician? a. Cloudy appearing urine b. Hypotonic bowel sounds c. Heart rate 102 beats/minute d. Continuous drainage from stoma

C. Heart rate 102 beats/minute Tachycardia may indicate infection, hemorrhage, or hypovolemia, which are all serious complications of this surgery. The urine from an ileal conduit normally contains mucus and is cloudy. Hypotonic bowel sounds are expected after bowel surgery. Continuous drainage of urine from the stoma is normal. DIF: Cognitive Level: Application REF: 1157 | 1159-1160 | 1158-1159

The nurse is caring for a patient receiving an initial dose of chemotherapy to treat a rapidly growing metastatic colon cancer. The nurse is aware that this patient is at risk for tumor lysis syndrome (TLS) and will monitor the patient closely for which abnormality associated with this oncologic emergency? A. Hypokalemia B. Hypouricemia C. Hypocalcemia D. Hypophosphatemia

C. Hypocalcemia TLS is a metabolic complication characterized by rapid release of intracellular components in response to chemotherapy. This can rapidly lead to acute renal injury. The hallmark signs of TLS are hyperuricemia, hyperphosphatemia, hyperkalemia, and hypocalcemia.

The surgeon was unable to spare a patient's parathyroid gland during a thyroidectomy. Which assessments should the nurse prioritize when providing postoperative care for this patient? A. Assessing the patient's white blood cell levels and assessing for infection B. Monitoring the patient's hemoglobin, hematocrit, and red blood cell levels C. Monitoring the patient's serum calcium levels and assessing for signs of hypocalcemia D. Monitoring the patient's level of consciousness and assessing for acute delirium or agitation

C. Monitoring the patient's serum calcium levels and assessing for signs of hypocalcemia Loss of the parathyroid gland is associated with hypocalcemia. Infection and anemia are not associated with loss of the parathyroid gland, whereas cognitive changes are less pronounced than the signs and symptoms of hypocalcemia.

The nurse is caring for a 36-year-old woman with possible hypoparathyroidism after a thyroidectomy. It is most appropriate for the nurse to assess for which clinical manifestations? A. Polyuria, polydipsia, and weight loss Incorrect B. Cardiac dysrhythmias and hypertension C. Muscle spasms and hyperactive deep tendon reflexes Correct D. Hyperpigmentation, skin ulcers, and peripheral edema

C. Muscle spasms and hyperactive deep tendon reflexes Common assessment abnormalities associated with hypoparathyroidism include tetany (muscle spasms) and increased deep tendon reflexes. Hyperpigmentation is associated with Addison's disease. Skin ulcers occur in patient with diabetes. Edema is associated with hypothyroidism. Polyuria and polydipsia occur in patients with diabetes mellitus or diabetes insipidus. Weight loss occurs in hyperthyroidism or diabetic ketoacidosis. Hypertension and cardiac dysrhythmias may be caused by hyperthyroidism, hyperparathyroidism, or pheochromocytoma.

What is a nursing priority in the care of a patient with a diagnosis of hypothyroidism? A. Providing a dark, low-stimulation environment B. Closely monitoring the patient's intake and output C. Patient teaching related to levothyroxine (Synthroid) D. Patient teaching related to radioactive iodine therapy

C. Patient teaching related to levothyroxine (Synthroid) A euthyroid state is most often achieved in patients with hypothyroidism by the administration of levothyroxine (Synthroid). It is not necessary to carefully monitor intake and output, and low stimulation and radioactive iodine therapy are indicated in the treatment of hyperthyroidism.

A 64-year-old male patient who is receiving radiation to the head and neck as treatment for an invasive malignant tumor complains of mouth sores and pain. Which intervention should the nurse add to this patient's plan of care? A. Weigh the patient every month to monitor for weight loss. B. Cleanse the mouth every 2 to 4 hours with hydrogen peroxide. C. Provide high-protein and high-calorie, soft foods every 2 hours. D. Apply palifermin (Kepivance) liberally to the affected oral mucosa.

C. Provide high-protein and high-calorie, soft foods every 2 hours. A patient with stomatitis should have soft, nonirritating foods offered frequently. The diet should be high in protein and high in calories. Saline or water should be used to cleanse the mouth (not hydrogen peroxide). Palifermin is administered intravenously as a growth factor to stimulate cells on the surface layer of the mouth to grow. Patients should be weighed at least twice each week to monitor for weight loss.

7. A patient who has just been diagnosed with type 2 diabetes is 5 ft 4 in (160 cm) tall and weighs 182 pounds (82 kg). A nursing diagnosis of imbalanced nutrition: more than body requirements is developed. Which patient outcome is most important for this patient? a. The patient will have a diet and exercise plan that results in weight loss. b. The patient will state the reasons for eliminating simple sugars in the diet. c. The patient will have a glycosylated hemoglobin level of less than 7%. d. The patient will choose a diet that distributes calories throughout the day.

C. The patient will have a glycosylated hemoglobin level of less than 7%. Rationale: The complications of diabetes are related to elevated blood glucose, and the most important patient outcome is the reduction of glucose to near-normal levels. The other outcomes are also appropriate but are not as high in priority. Cognitive Level: Application Text Reference: p. 1273 Nursing Process: Planning NCLEX: Physiological Integrity

Which information will the nurse include when teaching the patient with a urinary tract infection (UTI) about the use of phenazopyridine (Pyridium)? a. Take the medication for at least 7 days. b. Use sunscreen while taking the Pyridium. c. The urine may turn a reddish-orange color. d. Use the Pyridium before sexual intercourse.

C. The urine may turn a reddish-orange color. Patients should be taught that Pyridium will color the urine deep orange. Urinary analgesics should only be needed for a few days until the prescribed antibiotics decrease the bacterial count. Taking Pyridium before intercourse will not be helpful in reducing the risk for UTI. Pyridium does not cause photosensitivity.

When assessing a 30-year-old man who complains of a feeling of incomplete bladder emptying and a split, spraying urine stream, the nurse asks about a history of a. bladder infection. b. recent kidney trauma. c. gonococcal urethritis. d. benign prostatic hyperplasia.

C. gonococcal urethritis The patient's clinical manifestations are consistent with urethral strictures, a possible complication of gonococcal urethritis. These symptoms are not consistent with benign prostatic hyperplasia, kidney trauma, or bladder infection.

24. A diagnosis of hyperglycemic hyperosmolar nonketotic coma (HHNC) is made for a patient with type 2 diabetes who is brought to the emergency department in an unresponsive state. The nurse will anticipate the need to a. administer glargine (Lantus) insulin. b. initiate oxygen by nasal cannula. c. insert a large-bore IV catheter. d. give 50% dextrose as a bolus.

C. insert a large-bore IV catheter Rationale: HHNC is initially treated with large volumes of IV fluids to correct hypovolemia. Regular insulin is administered, not a long-acting insulin. There is no indication that the patient requires oxygen. Dextrose solutions will increase the patient's blood glucose and would be contraindicated.

A patient's renal calculus is analyzed as being very high in uric acid. To prevent recurrence of stones, the nurse teaches the patient to avoid eating a. milk and dairy products. b. legumes and dried fruits. c. organ meats and sardines. d. spinach, chocolate, and tea.

C. organ meats and sardines Organ meats and fish such as sardines increase purine levels and uric acid. Spinach, chocolate, and tomatoes should be avoided in patients who have oxalate stones. Milk, dairy products, legumes, and dried fruits may increase the incidence of calcium-containing stones.

10. When assessing an individual who has been diagnosed with early chronic HIV infection and has a normal CD4+ count, the nurse will a. check neurologic orientation. b. ask about problems with diarrhea. c. palpate the regional lymph nodes. d. examine the oral mucosa for lesions.

C. palpate the regional lymph nodes Persistent generalized lymphadenopathy is common in the early stage of chronic infection. Diarrhea, oral lesions, and neurologic abnormalities would occur in the later stages of HIV infection.

Which finding by the nurse for a patient admitted with glomerulonephritis indicates that treatment has been effective? a. The patient denies pain with voiding. b. The urine dipstick is negative for nitrites. c. Peripheral and periorbital edema is resolved. d. The antistreptolysin-O (ASO) titer is decreased.

C. peripheral and periorbital edema is resolved Since edema is a common clinical manifestation of glomerulonephritis, resolution of the edema indicates that the prescribed therapies have been effective. Antibodies to streptococcus will persist after a streptococcal infection. Nitrites will be negative and the patient will not experience dysuria since the patient does not have a urinary tract infection.

3. After having a positive rapid-antibody test for HIV, a patient is anxious and does not appear to hear what the nurse is saying. At this time, it is most important that the nurse a. teach the patient about the medications available for treatment. b. inform the patient how to protect sexual and needle-sharing partners. c. remind the patient about the need to return for retesting to verify the results. d. ask the patient to notify individuals who have had risky contact with the patient.

C. remind the patient about the need to return for retesting to verify the results After an initial positive antibody test, the next step is retesting to confirm the results. A patient who is anxious is not likely to be able to take in new information or be willing to disclose information about HIV status of other individuals.

38. A diabetic patient has a new order for inhaled insulin (Exubera). Which information about the patient indicates that the nurse should contact the patient before administering the Exubera? a. The patient has a history of a recent myocardial infarction. b. The patient's blood glucose is 224 mg/dl. c. The patient uses a bronchodilator to treat emphysema. d. The patient's temperature is 101.4° F.

C. the patient uses a bronchodilator to treat emphysema Rationale: Exubera is not recommended for patients with emphysema. The other data do not indicate any contraindication to using Exubera.

A patient undergoes a nephrectomy after having massive trauma to the kidney. Which assessment finding obtained postoperatively is most important to communicate to the surgeon? a. Blood pressure is 102/58. b. Incisional pain level is 8/10. c. Urine output is 20 mL/hr for 2 hours. d. Crackles are heard at both lung bases.

C. urine output is 20 mL/hr for 2 hours. Because the urine output should be at least 0.5 mL/kg/hr, a 40 mL output for 2 hours indicates that the patient may have decreased renal perfusion because of bleeding, inadequate fluid intake, or obstruction at the suture site. The blood pressure requires ongoing monitoring but does not indicate inadequate perfusion at this time. The patient should cough and deep breathe, but the crackles do not indicate a need for an immediate change in therapy. The incisional pain should be addressed, but this is not as potentially life threatening as decreased renal perfusion. In addition, the nurse can medicate the patient for pain. DIF: Cognitive Level: Application REF: 1154-1155

11. A nurse is reinforcing education to a client who has chronic renal failure. Which of the following should the nurse remind the client to increase in her diet? -Calcium -Phosphorous -Potassium -Sodium

Calcium

The patient with suspected pancreatic cancer is having many diagnostic studies done. Which one can be used to establish the diagnosis of pancreatic adenocarcinoma and for monitoring the response to treatment? Spiral CT scan A PET/CT scan Incorrect Abdominal ultrasound Cancer-associated antigen 19-9

Cancer-associated antigen 19-9 Correct The cancer-associated antigen 19-9 (CA 19-9) is the tumor marker used for the diagnosis of pancreatic adenocarcinoma and for monitoring the response to treatment. Although a spiral CT scan may be the initial study done and provides information on metastasis and vascular involvement, this test and the PET/CT scan or abdominal ultrasound do not provide additional information.

14. A nurse is caring for a client receiving peritoneal dialysis. The nurse notes that the client's dialysate output is less than the input, the abdomen is distended, and the client is reporting pain. Which of the following is an appropriate nursing action? -Infuse an additional amount of dialysate -Administer pain medication to the client -Change the client's position -Ask the client to ambulate

Change the client's position

The nurse notes that the patient's Foley catheter bag has been empty for 4 hours. The priority action would be to: A) Irrigate the Foley. B) Check for kinks in the tubing. C) Notify the health care provider. D) Assess the patient's intake.

Check for kinks in the tubing.

The nurse instructs an obese 22-year-old man with a sedentary job about the health benefits of an exercise program. The nurse evaluates that teaching is effective when the patient makes which statement? a. "The goal is to walk at least 10,000 steps every day of the week." b. "Weekend aerobics for 2 hours is better than exercising every day." c. "Aerobic exercise will increase my appetite and result in weight gain." d. "Exercise causes weight loss by decreasing my resting metabolic rate."

Correct answer: a Rationale: A realistic activity goal is to walk 10,000 steps a day. Increased activity does not promote an increase in appetite or lead to weight gain. Exercise should be done daily, preferably 30 minutes to an hour a day. Exercise increases metabolic rate.

What information should be included in the dietary teaching for the patient following a Roux-en-Y gastric bypass? a. Avoid sugary foods and limit fluids to prevent dumping syndrome. b. Gradually increase the amount of food ingested to preoperative levels. c. Maintain a long-term liquid diet to prevent damage to the surgical site. d. Consume foods high in complex carbohydrates, protein, and fiber to add bulk to contents.

Correct answer: a Rationale: Fluids and foods high in carbohydrates tend to promote diarrhea and symptoms of dumping syndrome in patients with gastric bypass surgery. The diet generally should be high in protein and low in carbohydrates, fat, and roughage and consists of six small feedings a day because of the small stomach size. Liquid diets are likely to be used longer for the patient with a gastroplasty.

The nurse is caring for a patient who is 5'6" tall and weighs 186 lb. The nurse has discussed reasonable weight loss goals and a low-calorie diet with the patient. Which statement made by the patient indicates a need for further teaching? a. "I will limit intake to 500 calories a day." b. "I will try to eat very slowly during mealtimes." c. "I'll try to pick foods from all of the basic food groups." d. "It's important for me to begin a regular exercise program."

Correct answer: a Rationale: Limiting intake to 500 calories per day is not indicated for this patient, and the severe calorie energy restriction would place this patient at risk for multiple nutrient deficiencies. Decreasing caloric intake at least 500 to 1000 calories a day is recommended for weight loss of one to two pounds per week. The other options show understanding of the teaching.

Priority Decision: Before selecting a weight reduction plan with an obese patient, what is most important for the nurse to first assess? a. The patient's motivation to lose weight b. The length of time that the patient has been obese c. Whether financial considerations will affect the patient's choices d. The patient's anthropometric measures of height, weight, BMI, waist-to-hip ratio, and skinfold thickness

Correct answer: a Rationale: Motivation is essential. Focus on the reasons for wanting to lose weight. The rest of the options will asset in planning the weight loss if the patient is motivated.

A 50-year-old African American woman has a BMI of 35 kg/m2, type 2 diabetes mellitus, hypercholesterolemia, and irritable bowel syndrome (IBS). She is seeking assistance in losing weight, because, "I have trouble stopping eating when I should, but I do not want to have bariatric surgery." Which drug therapy should the nurse question if it is prescribed for this patient? a. Orlistat (Xenical) b. Locaserin (Belviq) c. Phentermine (Adipex-P) d. Phentermine and topiramate (Qsymia)

Correct answer: a Rationale: Orlistat (Xenical), which blocks fat breakdown and absorption in the intestine, produces some unpleasant GI side effects. This drug would not be appropriate for someone with IBS. Locaserin (Belviq) suppresses the appetite and creates a sense of satiety that may be helpful for this patient. Phentermine (Adipex-P) needs to be used for a limited period of time (3 months or less). Qsymia is a combination of two drugs, phentermine and topiramate. Phentermine is a sympathomimetic agent that suppresses appetite and topiramate induces a sense of satiety.

A patient has been on a 1000-calorie diet with a daily exercise routine. In 2 months, the patient has lost 20 lb (9kg) toward a goal of 50 lb (23 kg) but is now discouraged that no weight has been lost in the last 2 weeks. What should the nurse tell the patient about this? a. Plateaus where no weight is lost normally occur during a weight-loss program. b. A weight considered by the body to most efficient for functioning has been reached. c. A return to former eating habits is the most common cause of not continuing to lose weight. d. A steady weight may be due to water gain from eating foods high in sodium.

Correct answer: a Rationale: Plateau periods during which no weight is lost are normal occurrences during weight reduction and may last for several days to several weeks but weight loss will resume if the prescribed weight reduction plan is continued. Weight loss may stop if former eating habits are resumed but this not the most common cause of plateaus.

A patient asks the nurse about taking phentermine and topiramate (Qsymia) for weight loss. To avoid side effects, it is important for the nurse to determine whether the patient has a history of a. glaucoma. b. hypertension. c. valvular heart disease. d. irritable bowel disease.

Correct answer: a Rationale: Qsymia is a combination of phentermine and topiramate. It must not be used in patients with glaucoma or hyperthyroidism.

A patient with hepatitis A is in the acute phase. The nurse plans care for the patient based on the knowledge that a. pruritus is a common problem with jaundice in this phase. b. the patient is most likely to transmit the disease during this phase. c. gastrointestinal symptoms are not as severe in hepatitis A as they are in hepatitis B. d. extrahepatic manifestations of glomerulonephritis and polyarteritis are common in this phase. (Lewis 1042)

Correct answer: a Rationale: The acute phase of jaundice may be icteric or anicteric. Jaundice results when bilirubin diffuses into the tissues. Pruritus sometimes accompanies jaundice. Pruritus is the result of an accumulation of bile salts beneath the skin.

During the initial postoperative period following bariatric surgery, the nurse recognizes the importance of monitoring obese patients for respiratory insufficiency based on what knowledge? a. The body stores anesthetics in adipose tissue. b. Postoperative pain may cause a decreased respiratory rate. c. Intubation may be difficult because of extra chin skinfolds. d. The patient's head must remain flat for a minimum of 2 hours postprocedure.

Correct answer: a Rationale: The body stores anesthetics in adipose tissue, placing patients with excess adipose tissue at risk for re-sedation. As adipose cells release anesthetics back into the bloodstream, the patient may become sedated after surgery, increasing the risk of hypoventilation and resultant respiratory insufficiency. Difficult intubation does not cause respiratory insufficiency. Pain usually increases respiratory rate. The patient's head should be elevated after bariatric surgery to decrease abdominal pressure and facilitate respirations.

The nurse cares for a 34-year-old woman after bariatric surgery. The nurse determines that discharge teaching related to diet is successful if the patient makes which statement? a. "A high protein diet that is low in carbohydrates and fat will prevent diarrhea." b. "Food should be high in fiber to prevent constipation from the pain medication." c. "Three meals a day with no snacks between meals will provide optimal nutrition." d. "Fluid intake should be at least 2000 mL per day with meals to avoid dehydration."

Correct answer: a Rationale: The diet generally prescribed is high in protein and low in carbohydrates, fat, and roughage and consists of six small feedings daily. Fluids should not be ingested with the meal, and in some cases, fluids should be restricted to less than 1000 mL per day. Fluids and foods high in carbohydrate tend to promote diarrhea and symptoms of the dumping syndrome. Generally, calorically dense foods (foods high in fat) should be avoided to permit more nutritionally sound food to be consumed.

The nurse teaches a 50-year-old woman who has a body mass index (BMI) of 39 kg/m2 about weight loss. Which dietary change would be appropriate for the nurse to recommend to this patient? a. Decrease fat intake and control portion size b. Increase vegetables and decrease fluid intake c. Increase protein intake and avoid carbohydrates d. Decrease complex carbohydrates and limit fiber

Correct answer: a Rationale: The safest dietary guideline for weight loss is to decrease caloric intake by maintaining a balance of nutrients and adequate hydration while controlling portion size and decreasing fat intake.

The patient with advanced cirrhosis asks why his abdomen is so swollen. The nurse's response is based on the knowledge that a. a lack of clotting factors promotes the collection of blood in the abdominal cavity. b. portal hypertension and hypoalbuminemia cause a fluid shift into the peritoneal space. c. decreased peristalsis in the GI tract contributes to gas formation and distention of the bowel. d. bile salts in the blood irritate the peritoneal membranes, causing edema and pocketing of fluid. (Lewis 1042)

Correct answer: b Rationale: Ascites is the accumulation of serous fluid in the peritoneal or abdominal cavity and is a common manifestation of cirrhosis. With portal hypertension, proteins shift from the blood vessels through the larger pores of the sinusoids (capillaries) into the lymph space. When the lymphatic system is unable to carry off the excess proteins and water, those substances leak through the liver capsule into the peritoneal cavity. Osmotic pressure of the proteins pulls additional fluid into the peritoneal cavity. A second mechanism of ascites formation is hypoalbuminemia, which results from the inability of the liver to synthesize albumin. Hypoalbuminemia results in decreased colloidal oncotic pressure. A third mechanism is hyperaldosteronism, which occurs when aldosterone is not metabolized by damaged hepatocytes. The increased level of aldosterone causes increases in sodium reabsorption by the renal tubules. Sodium retention and an increase in antidiuretic hormone levels cause additional water retention.

A patient with acute hepatitis B is being discharged in 2 days. In the discharge teaching plan the nurse should include instructions to a. avoid alcohol for the first 3 weeks. b. use a condom during sexual intercourse. c. have family members get an injection of immunoglobulin. d. follow a low-protein, moderate-carbohydrate, moderate-fat diet. (Lewis 1042)

Correct answer: b Rationale: Hepatitis B virus may be transmitted by mucosal exposure to infected blood, blood products, or other body fluids (e.g., semen, vaginal secretions, saliva). Hepatitis B is a sexually transmitted disease that is acquired through unprotected sex with an infected person. Condom use should be taught to patients to prevent transmission of hepatitis B.

Priority Decision: When medications are used in the treatment of obesity, what is most important for the nurse to teach the patient? a. Over-the-counter (OTC) diet aids are safer than other agents and con be useful in controlling appetite. b. Drugs should be used only as adjuncts to a diet and exercise program as treatment for a chronic condition. c. All drugs used for weight control are capable of altering central nervous system (CNS) function and should be used with caution. d. The primary effect of the medications is psychologic, controlling the urge to eat in response to stress or feelings of rejection.

Correct answer: b Rationale: Medications are used only as adjuncts to diet and exercise programs in the treatment of obesity. Drugs do not cure obesity; without changes in food intake and physical activity, weight gain will occur when the medications are discontinued. The medications used work in a variety of ways to control appetite but over-the-counter drugs are probably the least effective and most abused of these drugs.

A patient has been told that she has elevated liver enzymes caused by nonalcoholic fatty liver disease (NAFLD). The nursing teaching plan should include a. having genetic testing done. b. recommending a heart-healthy diet. c. the necessity to reduce weight rapidly. d. avoiding alcohol until liver enzymes return to normal. (Lewis 1042)

Correct answer: b Rationale: Nonalcoholic fatty liver disease (NAFLD) can progress to liver cirrhosis. There is no definitive treatment, and therapy is directed at reduction of risk factors, which include treatment of diabetes, reduction in body weight, and elimination of harmful medications. For patients who are overweight, weight reduction is important. Weight loss improves insulin sensitivity and reduces liver enzyme levels. No specific dietary therapy is recommended. However, a heart-healthy diet as recommended by the American Heart Association is appropriate.

Priority Decision: The nurse admitting a patient for bariatric surgery obtains the following information from the patient. Which finding should be brought to the surgeon's attention before proceeding with further patient preparation? a. History of hypertension b. History of untreated depression c. History of multiple attempts at weight loss d. History of sleep apnea treated with continuous positive airway pressure (CPAP)

Correct answer: b Rationale: Patients with histories of untreated depression or psychosis are not good candidates for surgery. All other historical information includes medical complications of severe obesity that would help to qualify the patient for the surgery.

Which teaching points are important when providing information to a patient with metabolic syndrome (select all that apply)? a. Stop smoking. b. Monitor weight daily. c. Increase level of activity. d. Decrease saturated fat intake. e. Reduce weight and maintain lower weight. f. Check blood glucose each morning prior to eating.

Correct answers: a, c, d, e Rationale: Patients with metabolic syndrome need to lower their risk factors by reducing and maintaining weight, increasing physical activity, establishing healthy diet habits, and smoking cessation. Some patients with metabolic syndrome are diabetic and would need to monitor glucose levels frequently. When monitoring weight reduction, it is recommended to check weight weekly, not daily.

Which patient is at highest risk for complications of obesity? a. A 30-year-old woman who is 5 ft (151 cm) tall, weighs 140 lb (63 kg), and carries weight in her thighs. b. A 56-year-old woman with a BMI of 38 kg/m2, a waist measurement of 38 in (96 cm), and a hip measurement of 36 in (91 cm) c. A 42-year-old man with a waist measurement of 36 in (91 cm) and a hip measurement of 36 in (91 cm) who is 5 ft, 6 in (166 cm) tall and weighs 150 lb (68.2 kg) d. A 68-year-old man with a waist measurement of 38 in (96 cm) and a hip measurement of 42 in (76 cm) who is 5 ft, 11 in (179 cm) tall and weighs 200 lb (90.9 kg)

Correct answer: b Rationale: The 56-year-old woman has a body mass index (BMI) of 38 kg/m2 (obese, Class II) with a waist-to-hip ratio of 1.1 with android obesity and is more at risk (very high) than the other patients. The 30-year-old woman has the least risk with a BMI of 27.3 kg/m2 (overweight) and gynoid shape. The 42-year-old man has a BMI of 24.2 kg/m2 (normal weight) with one risk factor in the waist-to-hip ratio of 1.0 and the 68-year-old man has a BMI of 27.9 kg/m2 (overweight) with a waist-to-hip ratio of 0.9.

A 40-year-old severely obese female patient with type 2 diabetes wants to lose weight. After learning about the surgical procedures, she thinks a combination of restrictive and malabsorptive surgery would be best. Which procedure should the nurse teach her about? a. Lipectomy b. Roux-en-Y gastric bypass c. Adjustable gastric banding d. Vertical sleeve gastrectomy

Correct answer: b Rationale: The Roux-en-Y gastric bypass is a common combination of restrictive (limiting the size of the stomach) and malabsorptive (less food is absorbed) surgery. Lipectomy is used to remove unsightly flabby folds of adipose tissue. Adjustable gastric banding is the most common restrictive procedure. Vertical sleeve gastrectomy is a restrictive procedure that preserves stomach function.

The nurse has completed initial instruction with a patient regarding a weight loss program. The nurse determines that the teaching has been effective when the patient makes which statement? a. "I plan to lose 4 lb a week until I have lost the 60-pound goal." b. "I will keep a diary of weekly weights to illustrate my weight loss." c. "I will restrict my carbohydrate intake to less than 30 g/day to maximize weight loss." d."I should not exercise more than my program requires since increased activity increases the appetite."

Correct answer: b Rationale: The patient should monitor and record weight once per week. This prevents frustration at the normal variations in daily weights and may help the patient to maintain motivation to stay on the prescribed diet. Weight loss should occur at a rate of 1 to 2 lb/week. The diet should be well balanced rather than lacking in specific components that may cause an initial weight loss but is not usually sustainable. Exercise is a necessary component of any successful weight loss program.

Which female patient is most likely to have metabolic syndrome? a. BP 128/78 mm Hg, triglycerides 160 mg/dL, fasting blood glucose 102 mg/dL b. BP 142/90 mm Hg, high-density lipoproteins 45 mg/dL, fasting blood glucose 130 mg/dL c. Waist circumference 36 in, triglycerides 162 mg/dL, high-density lipoproteins 55 mg/dL d. Waist circumference 32 in, high-density lipoproteins 38 mg/dL, fasting blood glucose 122 mg/dL

Correct answer: b Rationale: Three of the following five measures are needed for a woman to be diagnosed with metabolic syndrome: waist circumference >35 in, triglycerides >150 mg/dL, high-density lipoproteins <50 mg/dL, BP >130 mm Hg systolic or >85 mm Hg diastolic, fasting blood glucose >110 mg/dL. Although the other options have some abnormal measures, none has all three measures in the diagnostic ranges. The criteria for metabolic syndrome for both women and men are listed in Table 41-10.

Which statement about obesity is explained by genetics? a. Older obese patients have exacerbated changes of aging. b. Android body shape and weight gain are influenced by genetics. c. White Americans have a higher incidence of obesity than African Americans. d. Men have a harder time losing weight, as they have more muscle mass than women.

Correct answer: b Rationale: Twin studies and studies with adopted children have shown that body shape and weight gain are influenced by genetics but more research is needed. Older obese people do have exacerbated aging problems related to declines in physical function. African Americans and Hispanics have a higher incidence of obesity than whites. Women have a higher incidence of obesity and more difficulty losing weight than men because women have a higher percentage of metabolically less-active fat.

In developing a weight reduction program with a 45-year-old female patient who weighs 197 lb, the nurse encourages the patient to set a weight loss goal of how many pounds in 4 weeks? a. 1-2 b. 3-5 c. 4-8 d. 5-10

Correct answer: c Rationale: A realistic weight loss goal for patients is 1 to 2 lb/wk, which prevents the patient from becoming frustrated at not meeting weight loss goals.

In developing an effective weight reduction plan for an overweight patient who states a willingness to try to lose weight, it is most important for the nurse to first assess which factor? a. The length of time the patient has been obese b. The patient's current level of physical activity c. The patient's social, emotional, and behavioral influences on obesity d. Anthropometric measurements, such as body mass index and skinfold thickness

Correct answer: c Rationale: Eating patterns are established early in life, and eating has many meanings for people. To establish a weight reduction plan that will be successful for the patient, the nurse should first explore the social, emotional, and behavioral influences on the patient's eating patterns. The duration of obesity, current physical activity level, and current anthropometric measurements are not as important for the weight reduction plan.

Which statement best describes the etiology of obesity? a. Obesity primarily results from a genetic predisposition. b. Psychosocial factors can override the effects of genetics in the etiology of obesity. c. Obesity is the result of complex interactions between genetic and environmental factors. d. Genetic factors are more important than environmental factors in the etiology of obesity.

Correct answer: c Rationale: The cause of obesity involves significant genetic and biologic susceptibility factors that are highly influenced by environmental and psychosocial factors.

A community health nurse is conducting an initial assessment of a new patient. Which assessments should the nurse include when screening the patient for metabolic syndrome (select all that apply)? a. Blood pressure b. Resting heart rate c. Physical endurance d. Waist circumference e. Fasting blood glucose

Correct answers: a, d, e Rationale: The diagnostic criteria for metabolic syndrome include elevated blood pressure, fasting blood glucose, waist circumference, triglycerides, and HDL cholesterol. Resting heart rate and physical endurance are not part of the diagnostic criteria.

Which explanation about weight reduction should be included when teaching the obese patient and her obese husband? a. Weight gain is caused by psychologic factors. b. Daily weighing is recommended to monitor weight loss. c. Fat is not burned until the glycogen-water pool is depleted. d. Men lose weight less quickly than women because they have a higher percentage of metabolically less-active fat.

Correct answer: c Rationale: With reducing diets that severely restrict carbohydrates, the body's glycogen stores become depleted within a few days. The glycogen normally binds to water in fat cells and it is this water loss that causes weight loss in the first few days. Fat is not burned until the glycogen-water pool is depleted. Although psychosocial components (i.e., using food for comfort or reward and inability to buy high-nutritional quality food) may have an influence on weight gain, these factors along with lack of physical exercise, underestimation of portion size, and genetics contribute to weight gain. Weekly weighing is recommended as a more reliable indicator of weight loss because daily weighing shows frequent fluctuation from retained water (including urine) and elimination of feces. Men are able to lose weight more quickly than women because women have a higher percentage of metabolically less-active fat.

When teaching a patient about weight reduction diets, the nurse teaches the patient that an appropriate single serving of a food is a. a 6-inch bagel. b. 1 cup of chopped vegetables. c. a piece of cheese the size of three dice. d. a chicken breast the size of a deck of cards.

Correct answer: d Rationale: A chicken breast the size of a deck of cards is about 3 oz, a recommended portion size of meat. Other normal portions include a 3-inch bagel, 1/2 cup of chopped vegetables, and a piece of cheese the size of six dice.

Which patient is at highest risk for developing metabolic syndrome? a. A 62-year-old white man who has coronary artery disease with chronic stable angina b. A 54-year-old Hispanic woman who is sedentary and has nephrogenic diabetes insipidus c. A 27-year-old Asian American woman who has preeclampsia and gestational diabetes mellitus d. A 38-year-old Native American man who has diabetes mellitus and elevated hemoglobin A1C

Correct answer: d Rationale: African Americans, Hispanics, Native Americans, and Asians are at an increased risk for development of metabolic syndrome. Other risk factors include individuals who have diabetes that cannot maintain a normal glucose level, have hypertension, and secrete a large amount of insulin, or who have survived a heart attack and have hyperinsulinemia.

In the immediate postoperative period a nurse cares for a severely obese 72-year-old man who had surgery for repair of a lower leg fracture. Which assessment would be most important for the nurse to make? a. Cardiac rhythm b. Surgical dressing c. Postoperative pain d. Oxygen saturation

Correct answer: d Rationale: After surgery an older and/or severely obese patient should be closely monitored for oxygen desaturation. The body stores anesthetics in adipose tissue, placing patients with excess adipose tissue (e.g., obesity, older) at risk for resedation. As adipose cells release anesthetic back into the bloodstream, the patient may become sedated after surgery. This may depress the respiratory rate and result in a drop in oxygen saturation.

At the first visit to the clinic, the female patient with a BMI of 29 kg/m2 tells the nurse that she does not want to become obese. Which question used for assessing weight issues is the most important question for the nurse to ask? a. "What factors contributed to your current body weight?" b. "How is your overall health affected by your body weight?" c. "What is your history of gaining weight and losing weight?" d. "In what ways are you interested in managing your weight differently?"

Correct answer: d Rationale: Asking the patient about her desire to manage her weight in a different manner helps the nurse determine the patient's readiness for learning, degree of motivation, and willingness to change lifestyle habits. The nurse can help the patient set realistic goals. This question will also lead to discussing the patient's history of gaining and losing weight and factors that have contributed to the patient's current weight. The patient may be unaware of the overall health effects of her body weight, so this question is not helpful at this time.

The obesity classification that is most often associated with cardiovascular health problems is a. primary obesity. b. secondary obesity. c. gynoid fat distribution. d. android fat distribution.

Correct answer: d Rationale: Individuals with fat located primarily in the abdominal area (i.e., whose body is apple-shaped) are at greater risk for obesity-related complications (e.g., heart disease) than are those whose fat is primarily located in the upper legs (i.e., whose body is pear-shaped). Individuals whose fat is distributed over the abdomen and upper body (i.e., neck, arms, and shoulders) are classified as having android obesity.

The nurse has completed initial instruction with a patient regarding a weight-loss program. Which patient comment indicates to the nurse that the teaching has been effective? a. "I will keep a diary of daily weight to illustrate my weight loss." b. "I plan to lose 4 lb a week until I have lost the 60 lb I want to lose." c. "I should not exercise more than what is required so I don't increase my appetite." d. "I plan to join a behavior modification group to help establish long-term behavior changes."

Correct answer: d Rationale: People who have undergone behavior therapy are more successful in maintaining weight losses over time because most programs deemphasize the diet, focus on how and when the person eats and education, and provide support from others. Weighing daily is not recommended and plateaus may not allow for consistent weight loss. A goal for weight loss must be set and 1 to 2 pounds a week is realistic. A more rapid loss often causes skin and underlying tissue to lose elasticity and become flabby folds of tissue. Exercising more often depresses appetite and exercise need not be limited.

The best nutritional therapy plan for a person who is obese is a. the Zone diet. b. the Atkins diet. c. Sugar Busters. d. foods from the basic food groups.

Correct answer: d Rationale: Restricted food intake is a cornerstone for any weight loss or maintenance program. A good weight loss plan should include foods from the basic food groups.

The severely obese patient has elected to have the Roux-en-Y gastric bypass (RYGB) procedure. The nurse will know the patient understands the preoperative teaching when the patient makes which statement? a. "This surgery will preserve the function of my stomach." b. "This surgery will remove the fat cells from my abdomen." c. "This surgery can be modified whenever I need it to be changed." d. "This surgery decreases how much I can eat and how many calories I can absorb."

Correct answer: d Rationale: The RYGB decreases the size of the stomach to a gastric pouch and attaches it directly to the small intestine so food bypasses 90% of the stomach, the duodenum, and a small segment of the jejunum. The vertical sleeve gastrectomy removes 85% of the stomach, but preserves the function of the stomach. Lipectomy and liposuction remove fat tissue from the abdomen or other areas. Adjustable gastric banding can be modified or reversed at a later date.

Which patient has the highest morbidity risk? a. Male 6 ft. 1 in. tall, BMI 29 kg/m2 b. Female 5 ft. 6 in. tall, weight 150 lb. c. Male with waist circumference 46 in. d. Female 5 ft. 10 in. tall, obesity Class III

Correct answer: d Rationale: The patient in Class III obesity has the highest risk for disease because Class III denotes severe obesity or a BMI greater than 40 kg/m2. The patient with waist circumference 46 in. has a high risk for disease, but without the BMI or obesity class, a more precise determination cannot be made. The female who is 5 ft. 6 in. tall has a normal weight for her height. The male patient who is over 6 ft. tall is overweight, which increases his risk of disease, but a more precise determination cannot be made without the waist circumference.

What is a postoperative nursing intervention for the obese patient who has undergone bariatric surgery? a. Irrigating and repositioning the nasogastric (NG) tube as needed b. Delaying ambulation until the patient has enough strength to support self c. Keeping the patient positioned on the side to facilitate respiratory function d. Providing adequate support to the incision during coughing, deep breathing, and turning

Correct answer: d Rationale: Turning, coughing, and deep breathing are essential to prevent postoperative complications. Protecting the incision from strain is important since wound dehiscence is a problem for obese patients. If a nasogastric (NG) tube that is present following gastric surgery for severe obesity becomes blocked or needs repositioning, the health care provider should be notified. Ambulation is usually started on the evening of surgery and addition help will be needed to support the patient. Respiratory function is promoted by keeping the head of the bed elevated at an angle of 35 to 40 degrees.

This bariatric surgical procedure involves creating a stoma and gastric pouch that is reversible, and no malabsorption occurs. What surgical procedure is this? a. Vertical gastric banding b. Biliopancreatic diversion c. Roux-en-Y gastric bypass d. Adjustable gastric banding

Correct answer: d Rationale: With adjustable gastric banding (AGB), the stomach size is limited by an inflatable band placed around the fundus of the stomach. The band is connected to a subcutaneous port and can be inflated or deflated to change the stoma size to meet the patient's needs as weight is lost. The procedure is performed laparoscopically and, if necessary, can be modified or reversed after the initial procedure.

Wich of the following criteria must be met for a diagnosis of metabolic syndrome (select all that apply)? a. Hypertension b. Elevated triglycerides c. Elevated plasma glucose d. Increased waist circumference e. Decreased low-density lipoproteins

Correct answers: a, b, c, d Rationale: Three of the following five criteria must be met for a diagnosis of metabolic syndrome: • Waist circumference of 40 inches or more in men and 35 inches or more in women • Triglyceride levels higher than 150 mg/dL, or need for drug treatment for elevated triglyceride levels • High-density lipoprotein (HDL) cholesterol levels lower than 40 mg/dL in men and lower than 50 mg/dL in women, or need for drug treatment for reduced HDL cholesterol levels • Blood pressure: 130 mm Hg or higher systolic or 85 mm Hg or higher diastolic, or need for drug treatment for hypertension • Fasting blood glucose level of 110 mg/dL or higher, or need for drug treatment for elevated glucose levels

In preparing to care for the obese patient with cancer, what physiologic problems is this patient at a greater risk for having (select all that apply)? a. Tinnitus b. Fractures c. Sleep apnea d. Trousseau's sign e. Type 2 diabetes mellitus f. Gastroesophageal reflux disease (GERD)

Correct answers: c, e, f Rationale: Obese patients are at a higher risk for cancer, sleep apnea and sleep deprivation, type 2 diabetes mellitus, gastroesophageal reflux disease (GERD), nonalcoholic steatohepatits, osteoarthritis, and cardiovascular problems. The other options are not related to obesity.

The nurse is caring for a 68-year-old patient admitted with abdominal pain, nausea, and vomiting. The patient has an abdominal mass, and a bowel obstruction is suspected. The nurse auscultating the abdomen listens for which type of bowel sounds that are consistent with the patient's clinical picture? A. Low-pitched and rumbling above the area of obstruction B. High-pitched and hypoactive below the area of obstruction C. Low-pitched and hyperactive below the area of obstruction D. High-pitched and hyperactive above the area of obstruction

D Early in intestinal obstruction, the patient's bowel sounds are hyperactive and high-pitched, sometimes referred to as "tinkling" above the level of the obstruction. This occurs because peristaltic action increases to "push past" the area of obstruction. As the obstruction becomes complete, bowel sounds decrease and finally become absent.

The nurse is caring for a postoperative patient with a colostomy. The nurse is preparing to administer a dose of famotidine (Pepcid) when the patient asks why the medication was ordered since the patient does not have a history of heartburn or gastroesophageal reflux disease (GERD). What response by the nurse would be the most appropriate? A. "This will prevent air from accumulating in the stomach, causing gas pains." B. "This will prevent the heartburn that occurs as a side effect of general anesthesia." C. "The stress of surgery is likely to cause stomach bleeding if you do not receive it." D. "This will reduce the amount of HCl in the stomach until the nasogastric tube is removed and you can eat a regular diet again

D Famotidine is an H2-receptor antagonist that inhibits gastric HCl secretion and thus minimizes damage to gastric mucosa while the patient is not eating a regular diet after surgery. Famotidine does not prevent air from accumulating in the stomach or stop the stomach from bleeding. Heartburn is not a side effect of general anesthesia.

The nurse asks a 68-year-old patient scheduled for colectomy to sign the operative permit as directed in the physician's preoperative orders. The patient states that the physician has not really explained very well what is involved in the surgical procedure. What is the most appropriate action by the nurse? A. Ask family members whether they have discussed the surgical procedure with the physician. B. Have the patient sign the form and state the physician will visit to explain the procedure before surgery. C. Explain the planned surgical procedure as well as possible and have the patient sign the consent form. D. Delay the patient's signature on the consent and notify the physician about the conversation with the patient

D The patient should not be asked to sign a consent form unless the procedure has been explained to the satisfaction of the patient. The nurse should notify the physician, who has the responsibility for obtaining consent.

The nurse is preparing to administer a scheduled dose of docusate sodium (Colace) when the patient reports an episode of loose stool and does not want to take the medication. What is the appropriate action by the nurse? A. Write an incident report about this untoward event. B. Attempt to have the family convince the patient to take the ordered dose. C. Withhold the medication at this time and try to administer it later in the day. D. Chart the dose as not given on the medical record and explain in the nursing progress notes.

D Whenever a patient refuses medication, the dose should be charted as not given with an explanation of the reason documented in the nursing progress notes. In this instance, the refusal indicates good judgment by the patient, and the patient should not be encouraged to take it today

This morning a 21-year-old male patient had a long leg cast applied and wants to get up and try out his crutches before dinner. The nurse will not allow this. What is the best rationale that the nurse should give the patient for this decision? A The cast is not dry yet, and it may be damaged while using crutches. B The nurse does not have anyone available to accompany the patient. C Rest, ice, compression, and elevation are in process to decrease pain. D Excess edema and other problems are prevented when the leg is elevated for 24 hours

D Excess edema and other problems are prevented when the leg is elevated for 24 hours For the first 24 hours after a lower extremity cast is applied, the leg will be elevated on pillows above the heart level to avoid excessive edema and compartment syndrome. The cast will also be drying during this 24-hour period. RICE is used for soft tissue injuries, not with long leg casts.

The nurse is caring for a 75-year-old woman who underwent left total knee arthroplasty and has a new order to be "up in chair today before noon." What action should the nurse take to protect the knee joint while carrying out the order? A Administer a dose of prescribed analgesic before completing the order. B Ask the physical therapist for a walker to limit weight bearing while getting out of bed. C Keep the continuous passive motion machine in place while lifting the patient from bed to chair. D Put on a knee immobilizer before moving the patient out of bed and keep the surgical leg elevated while sitting.

D Put on a knee immobilizer before moving the patient out of bed and keep the surgical leg elevated while sitting. The nurse should apply a knee immobilizer for stability before assisting the patient to get out of bed. This is a standard measure to protect the knee during movement following surgery. Although an analgesic should be given before the patient gets up in the chair for the first time, it will not protect the knee joint. Full weight bearing is begun before discharge, so a walker will not be used if the patient did not need one before the surgery. The CPM machine is not kept in place while the patient is getting up to the chair.

Which of the following practices should the nurse teach a patient to follow when the patient is applying topical medication? A) Avoid applying medications directly on to dressings B) Use a tongue blade whenever the patient's skin integrity allows C) Avoid covering skin regions that have topical medication in place D) Apply a layer of medication that is just thick enough to ensure coverage

D) Apply a layer of medication that is just thick enough to ensure coverage Patients should be directed to avoid applying topical medications too thickly. Medications may be applied directly on to dressings, and regions with medications may be covered. A tongue blade is not normally necessary for application.

A patient with psoriasis tells the nurse that she has quit her job as a receptionist because she feels her appearance is disgusting to customers. The nursing diagnosis that best describes this patient response is: A) ineffective coping related to lack of social support B) impaired skin integrity related to presence of lesions C) anxiety related to lack of knowledge of the disease process D) social isolation related to decreased activities secondary to fear of rejection

D) social isolation related to decreased activities secondary to fear of rejection

The nurse is caring for an 18-year-old female patient with acute lymphocytic leukemia who is scheduled to receive hematopoietic stem cell transplantation (HSCT). Which statement, if made by the patient, indicates a correct understanding of the procedure? A. "After the transplant I will feel better and can go home in 5 to 7 days." B. "I understand the transplant procedure has no dangerous side effects." C. "My brother will be a 100% match for the cells used during the transplant." D. "Before the transplant I will have chemotherapy and possibly full body radiation."

D. "Before the transplant I will have chemotherapy and possibly full body radiation." Hematopoietic stem cell transplantation (HSCT) requires eradication of diseased or cancer cells. This is accomplished by administering higher-than-usual dosages of chemotherapy with or without radiation therapy. A relative such as a brother would not be a perfect match with human leukocyte antigens; only identical twins are an exact match. HSCT is an intensive procedure with adverse effects and possible death. HSCT recipients can expect a 2- to 4-week hospitalization after the transplant.

4. A patient who is diagnosed with AIDS tells the nurse, "I have lots of thoughts about dying. Do you think I am just being morbid?" Which response by the nurse is best? a. "Thinking about dying will not improve the course of AIDS." b. "It is important to focus on the good things about your life now." c. "Do you think that taking an antidepressant might be helpful to you?" d. "Can you tell me more about the kind of thoughts that you are having?

D. "Can you tell me more about the kind of thoughts you are having?" More assessment of the patient's psychosocial status is needed before taking any other action. The statements, "Thinking about dying will not improve the course of AIDS" and "It is important to focus on the good things in life" discourage the patient from sharing any further information with the nurse and decrease the nurse's ability to develop a trusting relationship with the patient. Although antidepressants may be helpful, the initial action should be further assessment of the patient's feelings.

9. A patient with type 1 diabetes has received diet instruction as part of the treatment plan. The nurse determines a need for additional instruction when the patient says, a. "I may have an occasional alcoholic drink if I include it in my meal plan." b. "I will need a bedtime snack because I take an evening dose of NPH insulin." c. "I will eat meals as scheduled, even if I am not hungry, to prevent hypoglycemia." d. "I may eat whatever I want, as long as I use enough insulin to cover the calories."

D. "I may eat whatever I want, as long as I use enough insulin to cover the calories" Rationale: Most patients with type 1 diabetes need to plan diet choices very carefully. Patients who are using intensified insulin therapy have considerable flexibility in diet choices but still should restrict dietary intake of items such as fat, protein, and alcohol. The other patient statements are correct and indicate good understanding of the diet instruction. Cognitive Level: Application Text Reference: p. 1268 Nursing Process: Evaluation NCLEX: Physiological Integrity

The nurse determines that instruction regarding prevention of future urinary tract infections (UTIs) for a patient with cystitis has been effective when the patient states, a. "I can use vaginal sprays to reduce bacteria." b. "I will drink a quart of water or other fluids every day." c. "I will wash with soap and water before sexual intercourse." d. "I will empty my bladder every 3 to 4 hours during the day."

D. "I will empty my bladder every 3 to 4 hours during the day" Voiding every 3 to 4 hours is recommended to prevent UTIs. Use of vaginal sprays is dis- couraged. The bladder should be emptied before and after intercourse, but cleaning with soap and water is not necessary. A quart of fluids is insufficient to provide adequate urine output to decrease risk for UTI.

Which item would be most beneficial when providing oral care to a patient with metastatic cancer who is at risk for oral tissue injury secondary to chemotherapy? A. Firm-bristle toothbrush B. Hydrogen peroxide rinse C. Alcohol-based mouthwash D. 1 tsp salt in 1 L water mouth rinse

D. 1 tsp salt in 1 L water mouth rinse A salt-water mouth rinse will not cause further irritation to oral tissue that is fragile because of mucositis, which is a side effect of chemotherapy. A soft-bristle toothbrush will be used. One teaspoon of sodium bicarbonate may be added to the salt-water solution to decrease odor, alleviate pain, and dissolve mucin. Hydrogen peroxide and alcohol-based mouthwash are not used because they would damage the oral tissue.

A patient with a severe pounding headache has been diagnosed with hypertension. However, the hypertension is not responding to traditional treatment. What should the nurse expect as the next step in management of this patient? A. Administration of β-blocker medications B. Abdominal palpation to search for a tumor C. Administration of potassium-sparing diuretics D. A 24-hour urine collection for fractionated metanephrines

D. A 24-hour urine collection for fractionated metanephrines Pheochromocytoma should be suspected when hypertension does not respond to traditional treatment. The 24-hour urine collection for fractionated metanephrines is simple and reliable with elevated values in 95% of people with pheochromocytoma. In a patient with pheochromocytoma preoperatively an α-adrenergic receptor blocker is used to reduce BP. Abdominal palpation is avoided to avoid a sudden release of catecholamines and severe hypertension. Potassium-sparing diuretics are not needed. Most likely they would be used for hyperaldosteronism, which is another cause of hypertension.

9. Which of these patients will the nurse working in an HIV testing and treatment clinic anticipate teaching about antiretroviral therapy (ART)? a. A patient who is currently HIV negative but has unprotected sex with multiple partners b. A patient who was infected with HIV 15 years ago and now has a CD4+ count of 840/µl c. An HIV-positive patient with a CD4+ count of 120/µl who drinks a fifth of whiskey daily d. A patient who tested positive for HIV 2 years ago and has cytomegalovirus (CMV) retinitis

D. A patient who tested positive for HIV 2 years ago and has cytomegalovirus (CMV) retinitis CMV retinitis is an AIDS-defining illness and indicates that the patient is appropriate for ART even though the HIV infection period is relatively short. An HIV-negative patient would not be offered ART. A patient with a CD4+ count in the normal range would not typically be started on ART. A patient who drinks alcohol heavily would be unlikely to be able to manage the complex drug regimen and would not be appropriate for ART despite the low CD4+ count.

The patient is receiving biologic and targeted therapy for ovarian cancer. What medication should the nurse expect to administer before therapy to combat the most common side effects of these medications? A. Morphine sulfate B. Ibuprofen (Advil) C. Ondansetron (Zofran) D. Acetaminophen (Tylenol)

D. Acetaminophen (Tylenol) Acetaminophen is administered before therapy and every 4 hours to prevent or decrease the intensity of the severe flu-like symptoms, especially with interferon which is frequently used for ovarian cancer. Morphine sulfate and ibuprofen will not decrease flu-like symptoms. Ondansetron is an antiemetic, but not used first to combat flu-like symptoms of headache, fever, chills, myalgias, etc.

A 72-year-old who has benign prostatic hyperplasia is admitted to the hospital with chills, fever, and vomiting. Which finding by the nurse will be most helpful in determining whether the patient has an upper urinary tract infection (UTI)? a. Suprapubic pain b. Bladder distention c. Foul-smelling urine d. Costovertebral tenderness

D. Costovertebral tenderness Costovertebral tenderness is characteristic of pyelonephritis. The other symptoms are characteristic of lower UTI and are likely to be present if the patient also has an upper UTI. DIF: Cognitive Level: Application REF: 1128

The patient has been diagnosed with non-small cell lung cancer. Which type of targeted therapy will most likely be used for this patient to suppress cell proliferation and promote programmed tumor cell death? A. Proteasome inhibitors B. BCR-ABL tyrosine kinase inhibitors C. CD20 monoclonal antibodies (MoAb) D. Epidermal growth factor receptor-tyrosine kinase inhibitors (EGFR-TK)

D. Epidermal growth factor receptor-tyrosine kinase inhibitors (EGFR-TK) Targeted therapies are more selective for specific molecular targets. Thus they are able to kill cancer cells with less damage to normal cells than with chemotherapy. Epidermal growth factor receptor (EGFR) is a transmembrane molecule that works through activation of intracellular tyrosine kinase (TK) to suppress cell proliferation and promote apoptosis of non-small cell lung cancer and some colorectal, head and neck, and metastatic breast cancers. Proteasome inhibitors promote accumulation of proteins that promote tumor cell death for multiple myeloma. BCR-ABL tyrosine kinase inhibitors target specific oncogenes for chronic myeloid leukemia and some GI stromal tumors. CD20 monoclonal antibodies (MoAb) bind with CD20 antigen causing cytotoxicity in non-Hodgkin's lymphoma and chronic lymphocytic leukemia.

A patient has been diagnosed with Burkitt's lymphoma. In the initiation stage of cancer, the cells genetic structure is mutated. Exposure to what may have functioned as a carcinogen for this patient? A. Bacteria B. Sun exposure C. Most chemicals D. Epstein-Barr virus

D. Epstein-Barr virus Burkitt's lymphoma consistently shows evidence of the presence of Epstein-Barr virus in vitro. Bacteria do not initiate cancer. Sun exposure causes cell alterations leading to melanoma and squamous and basal cell skin carcinoma. Long-term exposure to certain chemicals (e.g., ethylene oxide, chloroform, benzene) is known to initiate cancer.

The nurse is reinforcing health teaching about osteoporosis with a 72-year-old patient admitted to the hospital. In reviewing this disorder, what should the nurse explain to the patient? A. With a family history of osteoporosis, there is no way to prevent or slow bone resorption. B. Continuous, low-dose corticosteroid treatment is effective in stopping the course of osteoporosis. C. Estrogen therapy must be maintained to prevent rapid progression of the osteoporosis. D. Even with a family history of osteoporosis, the calcium loss from bones can be slowed by increased calcium intake and exercise.

D. Even with a family history of osteoporosis, the calcium loss from bones can be slowed by increased calcium intake and exercise. The rate of progression of osteoporosis can be slowed if the patient takes calcium supplements and/or foods high in calcium and engages in regular weight-bearing exercise. Corticosteroids interfere with bone metabolism. Estrogen therapy is no longer used to prevent osteoporosis because of the associated increased risk of heart disease and breast and uterine cancer.

A 70-year-old man who has end-stage lung cancer is admitted to the hospital with confusion and oliguria for 2 days. Which finding would the nurse report immediately to the health care provider? A. Weight gain of 2 lb B. Urine specific gravity of 1.015 C. Blood urea nitrogen of 20 mg/dL D. Serum sodium level of 118 mEq/L

D. Serum sodium level of 118 mEq/L Lung cancer cells are able to manufacture and release antidiuretic hormone (ADH) with resultant water retention and hyponatremia. Hyponatremia (serum sodium levels less than 135 mEq/L) may lead to central nervous system symptoms such as confusion, seizures, coma, and death. A weight gain may be due to fluid retention. The urine specific gravity and blood urea nitrogen are normal.

When the patient is diagnosed with muscular dystrophy, what information should the nurse include in the teaching about this disorder? A. Prolonged bed rest will be used to decrease fatigue. B. An orthotic jacket will limit mobility and may contribute to deformity. C. Continuous positive airway pressure will be used to facilitate sleeping. D. Remain active to prevent skin breakdown and respiratory complications.

D. Remain active to prevent skin breakdown and respiratory complications. With muscular dystrophy, it is important for the patient to remain active for as long as possible. Prolonged bed rest should be avoided because immobility leads to further muscle wasting. An orthotic jacket may be used to provide stability and prevent further deformity. Continuous positive airway pressure (CPAP) is used as respiratory function decreases, before mechanical ventilation is needed to sustain respiratory function.

Which nursing diagnosis is most appropriate for a patient experiencing myelosuppression secondary to chemotherapy for cancer treatment? A. Acute pain B. Hypothermia C. Powerlessness D. Risk for infection

D. Risk for infection Myelosuppression is accompanied by a high risk of infection and sepsis. Hypothermia, powerlessness, and acute pain are also possible nursing diagnoses for patients undergoing chemotherapy, but the threat of infection is paramount.

The patient has been feeling tired lately and has gained weight; reports thickened, dry skin and increased cold sensitivity even though it is now summer. Which endocrine diagnostic test should be done first? a. Free thyroxine (FT4) b. Serum growth hormone (GH) c. Follicle stimulating hormone (FSH) d. Magnetic resonance imaging (MRI) of the head

a. Free thyroxine (FT4) The manifestations the patient is experiencing could be related to hypothyroidism. Free thyroxine (FT4) is considered a better indicator of thyroid function than total T4 and could be done to evaluate the patient for hypothyroidism. Growth hormone excess could cause thick, leathery, oily skin but does not demonstrate the other manifestations. FSH is manifest with menstrual irregularity and would be useful in distinguishing primary gonadal problems from pituitary insufficiency. MRI is the examination of choice for radiologic evaluation of the pituitary gland and the hypothalamus but would not be the first diagnostic study to further explore the basis of these manifestations.

19. What preoperative instruction should the nurse give to the patient scheduled for a subtotal thyroidectomy? a. How to support the head with the hands when turning in bed b. Coughing should be avoided to prevent pressure on the incision c. Head and neck will need to remain immobile until the incision heals d. Any tingling around the lips or in the fingers after surgery is expected and temporary

a. How to support the head with the hands when turning in bed To prevent strain on the suture line postoperatively, the patient's head must be manually supported while turning and moving in bed but range-of-motion exercises for the head and neck are also taught preoperatively to be gradually implemented after surgery. There is no contraindication for coughing and deep breathing and these should be carried out postoperatively. Tingling around the lips or fingers is a sign of hypocalcemia, which may occur if the parathyroid glands are inadvertently removed during surgery. This sign should be reported immediately.

31. A patient is admitted to the hospital with a diagnosis of Cushing syndrome. On physical assessment of the patient, what should the nurse expect to find? a. Hypertension, peripheral edema, and petechiae b. Weight loss, buffalo hump, and moon face with acne c. Abdominal and buttock striae, truncal obesity, and hypotension d. Anorexia, signs of dehydration, and hyperpigmentation of the skin

a. Hypertension, peripheral edema, and petechiae The effects of adrenocortical hormone excess, especially glucocorticoid excess, include weight gain from accumulation and redistribution of adipose tissue, sodium and water retention, glucose intolerance, protein wasting,loss of bone structure, loss of collagen, and capillary fragility leading to petechiae. Clinical manifestations of adrenocortical hormone deficiency include hypotension, dehydration, weight loss, and hyperpigmentation of the skin.

A patient received a kidney transplant last month. Because of the effects of immunosuppressive drugs and CKD, what complication of transplantation should the nurse be assessing the patient for to decrease the risk of mortality? a. Infection b. Rejection c. Malignancy d. Cardiovascular disease

a. Infection is a significant cause of morbidity and mortality after transplantation because the surgery, the immunosuppressive drugs, and the effects of CKD all suppress the body's normal defense mechanisms, thus increasing the risk of infection. The nurse must assess the patient as well as use aseptic technique to prevent infections. Rejection may occur but for other reasons. Malignancy occurrence increases later due to immunosuppressive therapy. Cardiovascular disease is the leading cause of death after renal transplantation but this would not be expected to cause death within the first month after transplantation.

Teaching in relation to home management after a laparoscopic cholecystectomy should include a. keeping the bandages on the puncture sites for 48 hours. b. reporting any bile-colored drainage or pus from any incision. c. using over-the-counter antiemetics if nausea and vomiting occur. d. emptying and measuring the contents of the bile bag from the T tube every day. (Lewis 1042)

b Rationale: The following discharge instructions are taught to the patient and caregiver after a laparoscopic cholecystectomy: First, remove the bandages on the puncture site the day after surgery and shower. Second, notify the surgeon if any of the following signs and symptoms occur: redness, swelling, bile-colored drainage or pus from any incision; and severe abdominal pain, nausea, vomiting, fever, or chills. Third, gradually resume normal activities. Fourth, return to work within 1 week of surgery. Fifth, resume a usual diet, but a low-fat diet is usually better tolerated for several weeks after surgery.

What is an appropriate nursing intervention for the patient with hyperparathyroidism? a. Pad side rails as a seizure precaution. b. Increase fluid intake to 3000 to 4000 mL daily. c. Maintain bed rest to prevent pathologic fractures. d. Monitor the patient for Trousseau's and Chvostek's signs.

b. Increase fluid intake to 3000 to 4000 mL daily A high fluid intake is indicated in hyperparathyroidism to dilute the hypercalcemia and flush the kidneys so that calcium stone formation is reduced. Seizures are not associated with hyperparathyroidism. Impending tetany of hypoparathyroidism after parathyroidectomy can be noted with Trousseau's and Chvostek's signs. The patient with hyperparathyroidism is at risk for pathologic fractures resulting from decreased bone density but mobility is encouraged to promote bone calcification.

8. A patient with SIADH is treated with water restriction. What does the patient experience when the nurse determines that treatment has been effective? a. Increased urine output, decreased serum sodium, and increased urine specific gravity b. Increased urine output, increased serum sodium, and decreased urine specific gravity c. Decreased urine output, increased serum sodium, and decreased urine specific gravity d. Decreased urine output, decreased serum sodium, and increased urine specific gravity

b. Increased urine output, increased serum sodium, and decreased urine specific gravity The patient with SIADH has water retention with hyponatremia, decreased urine output, and concentrated urine with high specific gravity. Improvement in the patient's condition is reflected by increased urine output, normalization of serum sodium, and more water in the urine, thus decreasing the specific gravity.

23. The nurse has identified the nursing diagnosis of fatigue for a patient who is hypothyroid. What should the nurse do while caring for this patient? a. Monitor for changes in orientation, cognition, and behavior. b. Monitor for vital signs and cardiac rhythm response to activity. c. Monitor bowel movement frequency, consistency, shape, volume, and color. d. Assist in developing well-balanced meal plans consistent with level of energy expenditure.

b. Monitor for vital signs and cardiac rhythm response to activity. Cardiorespiratory response to activity is important to monitor in this patient to determine the effect of activities and plan activity increases. Monitoring changes in orientation, cognition, and behavior are interventions for impaired memory. Monitoring bowels is needed to plan care for the patient with constipation. Assisting with meal planning will help the patient with imbalanced nutrition: more than body requirements to lose weight if needed.

To prevent the most common serious complication of PD, what is important for the nurse to do? a. Infuse the dialysate slowly. b. Use strict aseptic technique in the dialysis procedures. c. Have the patient empty the bowel before the inflow phase. d. Reposition the patient frequently and promote deep breathing.

b. Peritonitis is a common complication of peritoneal dialysis (PD) and may require catheter removal and termination of dialysis. Infection occurs from contamination of the dialysate or tubing or from progression of exit-site or tunnel infections and strict sterile technique must be used by health professionals as well as the patient to prevent contamination. Too-rapid infusion may cause shoulder pain and pain may be caused if the catheter tip touches the bowel. Difficulty breathing, atelectasis, and pneumonia may occur from pressure of the fluid on the diaphragm, which may be prevented by elevating the head of the bed and promoting repositioning and deep breathing.

25. A patient with hypothyroidism is treated with levothyroxine (Synthroid). What should the nurse include when teaching the patient about this therapy? a. Explain that alternate-day dosage may be used if side effects occur. b. Provide written instruction for all information related to the drug therapy. c. Assure the patient that a return to normal function will occur with replacement therapy. d. Inform the patient that the drug must be taken until the hormone balance is reestablished.

b. Provide written instruction for all information related to the drug therapy Because of the mental sluggishness, inattentiveness, and memory loss that occur with hypothyroidism, it is important to provide written instructions and repeat information when teaching the patient. Replacement therapy must be taken for life and alternate-day dosing is not therapeutic. Although most patients return to a normal state with treatment, cardiovascular conditions and psychoses may persist.

In caring for a patient after a spinal fusion, the nurse would immediately report to the physician which patient symptom? a.The patient experiences a single episode of emesis. b.The patient is unable to move the lower extremities. c. The patient is nauseated and has not voided in 4 hours. d. The patient complains of pain at the bone graft donor site.

b.The patient is unable to move the lower extremities. After spinal fusion surgery, the nurse should frequently monitor peripheral neurologic signs. Movement of the arms and legs and assessment of sensation should be unchanged in comparison with the preoperative status. These assessments are usually repeated every 2 to 4 hours during the first 48 hours after surgery, and findings are compared with those of the preoperative assessment. Paresthesias, such as numbness and tingling sensation, may not be relieved immediately after surgery. The nurse should document any new muscle weakness or paresthesias and report this to the surgeon immediately.

2. During assessment of the patient with acromegaly, what should the nurse expect the patient to report? a. Infertility b. Dry, irritated skin c. Undesirable changes in appearance d. An increase in height of 2 to 3 inches a year

c. Undesirable changes in appearance The increased production of GH in acromegaly causes an increase in thickness and width of bones and enlargement of soft tissues, resulting in marked changes in facial features, oily and coarse skin, and speech difficulties. Infertility is not a common finding because GH is usually the only pituitary hormone involved in acromegaly. Height is not increased in adults with GH excess because the epiphyses of the bones are closed.

What causes the gastrointestinal (GI) manifestation of stomatitis in the patient with CKD? a. High serum sodium levels b. Irritation of the GI tract from creatinine c. Increased ammonia from bacterial breakdown of urea d. Iron salts, calcium-containing phosphate binders, and limited fluid intake

c. Uremic fetor, or the urine odor of the breath, is caused by high urea content in the blood. Increased ammonia from bacterial breakdown of urea leads to stomatitis and mucosal ulcerations. Irritation of the gastrointestinal (GI) tract from urea in CKD contributes to anorexia, nausea, and vomiting. Ingestion of iron salts and calcium-containing phosphate binders, limited fluid intake, and limited activity cause constipation.

A patient rapidly progressing toward end-stage kidney disease asks about the possibility of a kidney transplant. In responding to the patient, the nurse knows that what is a contraindication to kidney transplantation? a. Hepatitis C infection b. Coronary artery disease c. Refractory hypertension d. Extensive vascular disease

d. Extensive vascular disease is a contraindication for renal transplantation, primarily because adequate blood supply is essential for the health of the new kidney. Other contraindications include disseminated malignancies, refractory or untreated cardiac disease, chronic respiratory failure, chronic infection, or unresolved psychosocial disorders. Coronary artery disease (CAD) may be treated with bypass surgery before transplantation and transplantation can relieve hypertension. Hepatitis B or C infection is not a contraindication.

4. The nurse's responsibility for a patient with a suspected disc herniation who is experiencing acute pain and muscle spasms is a.encouraging total bed rest for several days. b.teaching the principles of back strengthening exercises. c.stressing the importance of straight-leg raises to decrease pain. d.promoting the use of cold and hot compresses and pain medication.

d.promoting the use of cold and hot compresses and pain medication. If the acute muscle spasms and accompanying pain are not severe and debilitating, the patient may be treated on an outpatient basis with nonsteroidal antiinflammatory drugs (NSAIDs; e.g., acetaminophen) and muscle relaxants (e.g., cyclobenzaprine [Flexeril]). Massage and back manipulation, acupuncture, and the application of cold and hot compresses may help some patients. Severe pain may necessitate a brief course of opioid analgesics. A brief period (1 to 2 days) of rest at home may be necessary for some people; most patients do better with a continuation of their regular activities. Prolonged bed rest should be avoided. All patients during this time should refrain from activities that aggravate the pain, including lifting, bending, twisting, and prolonged sitting.


Ensembles d'études connexes

AP CSP PRACTICE TEST WRONG ANSWERS

View Set

Romeo and Juliet: Act 3-5 Questions

View Set

SAS Certification Prep Guide: Base Programming

View Set

Defensive driving at defensivedriving.com

View Set

C393 CompTIA A+ Core 1 All Topics

View Set

MCAT Discrete Practice Questions

View Set

Sport Management: Test 1 (Chapters 1-5)

View Set

State and Local Government Final

View Set

Drugs for Angina Pectoris & Management of STEMI

View Set

Live Virtual Machine Lab 2.1: Module 02 Defining Networking Devices

View Set